Vous êtes sur la page 1sur 154

CUADERNOS DE ÁLGEBRA

No. 8
Álgebra homológica

Oswaldo Lezama

Departamento de Matemáticas
Facultad de Ciencias
Universidad Nacional de Colombia
Sede de Bogotá

30 de junio de 2015
ii

Cuaderno dedicado a Patricia, mi hermana.


Contenido

Prólogo v

1. Elementos básicos de álgebra homológica 1


1.1. Sucesiones exactas . . . . . . . . . . . . . . . . . . . . . . . . . . . . 1
1.2. Módulos proyectivos . . . . . . . . . . . . . . . . . . . . . . . . . . . 10
1.3. Módulos reflexivos . . . . . . . . . . . . . . . . . . . . . . . . . . . . 17
1.4. Producto tensorial . . . . . . . . . . . . . . . . . . . . . . . . . . . . 19
1.5. Anillo de fracciones . . . . . . . . . . . . . . . . . . . . . . . . . . . . 27
1.6. Módulo de fracciones . . . . . . . . . . . . . . . . . . . . . . . . . . . 33
1.7. Módulos inyectivos . . . . . . . . . . . . . . . . . . . . . . . . . . . . 40
1.8. Módulos planos . . . . . . . . . . . . . . . . . . . . . . . . . . . . . . 50
1.9. Anillos hereditarios . . . . . . . . . . . . . . . . . . . . . . . . . . . . 55
1.10. Ejercicios . . . . . . . . . . . . . . . . . . . . . . . . . . . . . . . . . 63

2. Ext 67
2.1. Definición . . . . . . . . . . . . . . . . . . . . . . . . . . . . . . . . . 67
2.2. Propiedades básicas . . . . . . . . . . . . . . . . . . . . . . . . . . . . 73
2.3. Ejemplos . . . . . . . . . . . . . . . . . . . . . . . . . . . . . . . . . . 82
2.4. Ejercicios . . . . . . . . . . . . . . . . . . . . . . . . . . . . . . . . . 85

3. Tor 87
3.1. Definición y propiedades básicas . . . . . . . . . . . . . . . . . . . . . 87
3.2. Ejemplos . . . . . . . . . . . . . . . . . . . . . . . . . . . . . . . . . . 91
3.3. Tor y módulos planos . . . . . . . . . . . . . . . . . . . . . . . . . . . 94
3.4. Módulos planos y matrices . . . . . . . . . . . . . . . . . . . . . . . . 99
3.5. Torsión de un módulo . . . . . . . . . . . . . . . . . . . . . . . . . . . 100
3.6. Tor y torsión . . . . . . . . . . . . . . . . . . . . . . . . . . . . . . . 102
3.7. Ejercicios . . . . . . . . . . . . . . . . . . . . . . . . . . . . . . . . . 114

4. Dimensiones de módulos y anillos 115


4.1. Dimensiones proyectiva, inyectiva y plana de un módulo . . . . . . . . 115
4.2. Dimensión global de un anillo . . . . . . . . . . . . . . . . . . . . . . 122

iii
iv CONTENIDO

4.3. Dimensión global débil de un anillo . . . . . . . . . . . . . . . . . . . 124


4.4. Dimensión global de anillos noetherianos . . . . . . . . . . . . . . . . 126
4.5. Anillos con dimensión débil ≤ 1 . . . . . . . . . . . . . . . . . . . . . 128
4.6. Dimensión global y extensiones de anillos . . . . . . . . . . . . . . . . 129
4.7. Dimensión de Krull de un módulo . . . . . . . . . . . . . . . . . . . . 134
4.8. Dimensión de Krull de un anillo . . . . . . . . . . . . . . . . . . . . . 139
4.9. Ejercicios . . . . . . . . . . . . . . . . . . . . . . . . . . . . . . . . . 145

Bibliografı́a 146
Prólogo

La colección Cuadernos de álgebra consta de 10 publicaciones sobre los principales


temas de esta rama de las matemáticas, y pretende servir de material para preparar
los exámenes de admisión y de candidatura de los programas colombianos de doc-
torado en matemáticas. Los primeros cinco cuadernos cubren el material básico de
los cursos de estructuras algebraicas y álgebra lineal de los programas de maestrı́a;
los cinco cuadernos siguientes contienen algunos de los principales temas de los
exámenes de candidatura, a saber: anillos y módulos; categorı́as; álgebra homológica;
álgebra no conmutativa; álgebra conmutativa y geometrı́a algebraica. Cada cuaderno
es fruto de las clases dictadas por el autor en la Universidad Nacional de Colombia
en los últimos 25 años, y están basados en las fuentes bibliográficas consignadas en
cada uno de ellos, como también en el libro Anillos, Módulos y Categorı́as, publi-
cado por la Facultad de Ciencias de la Universidad Nacional de Colombia, y cuya
edición está totalmente agotada (véase [14]). Un material similar, pero mucho más
completo que el presentado en estas diez publicaciones, es el excelente libro de Serge
Lang, Algebra, cuya tercera edición revisada ha sido publicada por Springer en el
2004 (véase [13]). Posiblemente el valor de los Cuadernos de álgebra sea su pre-
sentación ordenada y didáctica, ası́ como la inclusión de muchas pruebas omitidas
en la literatura y suficientes ejemplos que ilustran la teorı́a. Los cuadernos son:
1. Grupos 6. Anillos y módulos
2. Anillos 7. Categorı́as
3. Módulos 8. Álgebra homológica
4. Álgebra lineal 9. Álgebra no conmutativa
5. Cuerpos 10. Álgebra conmutativa y geometrı́a algebraica
Los cuadernos están divididos en capı́tulos, los cuales a su vez se dividen en
secciones. Para cada capı́tulo se añade al final una lista de ejercicios que deberı́a ser
complementada por los lectores con las amplias listas de problemas que inluyen las
principales monografı́as relacionadas con el respectivo tema.
Cuaderno de álgebra homológica. Este cuaderno consta de cuatro capı́tu-
los: el primero presenta los instrumentos elementales del álgebra homológica tales
como los complejos y las sucesiones exactas, los módulos de presentación finita, los
módulos proyectivos e inyectivos, el producto tensorial de bimódulos, la localización

v
vi PRÓLOGO

de anillos y módulos en el caso no conmutativo, los módulos planos sobre anillos


conmutativos, la técnica particular de localización-globalización del álgebra conmu-
tativa, la noción de rango de un módulo y el teorema de Kaplansky sobre módulos
proyectivos finitamente generados sobre anillos locales. Se incluye además una intro-
ducción a los anillos hereditarios. El segundo y terecer capı́tulos permiten conocer
y aplicar otras dos técnicas centrales del álgebra homológica, a saber: el Ext y el
Tor. En el cuarto capı́tulo se estudia la teorı́a de la dimensión de anillos: dimensión
global, dimensión débil y la dimensión de Krull. Los instrumentos considerados en
este cuaderno permiten emprender estudios más profundos en álgebra no conmuta-
tiva, teorı́a de representación de grupos y álgebras, álgebra conmutativa, geometrı́a
algebraica, análisis algebraico, entre muchas otras áreas.
Para una buena compresión del presente cuaderno se recomienda al lector con-
sultar los cuadernos 2, 3, 6 y 7 (véanse [15], [16], [18] y [19]) ya que usaremos los
resultados y la notación consignados en ellos. En particular, A denotará un anillo
no ncesariamente conmutativo y con unidad 1. A∗ denota el grupo multiplicativo
de los elementos invertibles del anillo A. Si f es un homomorfismo de anillos, en-
tonces f (1) = 1. Salvo que se advierta lo contrario, los módulos serán considerados
a derecha. Si M es un A-módulo a derecha lo denotaremos también por MA . Si N
es un submódulo de M escribiremos N ≤ M . Para n ≥ 1, Mn (A) es el anillo de
matrices cuadradas de tamaño n×n con componentes en A, GLn (A) denota el grupo
lineal general de orden n sobre A, es decir, GLn (A) = Mn (A)∗ . La matriz idéntica
de tamaño n × n se denota por En . An denota el A-módulo libre derecho de vectores
columna de longitud n con entradas en A.

Oswaldo Lezama
Departamento de Matemáticas
Universidad Nacional de Colombia
Bogotá, Colombia
jolezamas@unal.edu.co
Capı́tulo 1

Elementos básicos de álgebra


homológica

Presentamos en este primer capı́tulo las construcciones básicas del álgebra homológi-
ca como son los complejos y las sucesiones exactas, el producto tensorial de bimódu-
los, los anillos y los módulos de fracciones no conmutativos. Haremos además una
introducción a los módulos proyectivos, inyectivos y planos.

1.1. Sucesiones exactas


Definición 1.1.1. Una sucesión o cadena de homomorfismos de A-módulos de la
forma
f−1 f0 f1
C : · · · −→ M−1 −→ M0 −→ M1 −→ M2 −→ · · ·

es un complejo si Im(fi ) ⊆ ker(fi+1 ) para cada i ∈ Z. El módulo cociente

ker(fi+1 )/Im(fi )

se denomina el i-ésimo módulo de homologı́a del complejo C. La sucesión C se


dice exacta si Im(fi ) = ker(fi+1 ) para cada i ∈ Z.

Son particularmente importantes las sucesiones exactas finitas. Por ejemplo,


f g
nótese que la sucesión M1 −→ M2 −→ M3 −→ 0 es exacta si, y sólo si, Im(f ) =
f g
ker(g) y g es sobreyectivo. Similarmente, la sucesión 0 −→ M1 −→ M2 −→ M3 es
exacta si, y sólo si, f es inyectivo e Im(f ) = ker(g).
Otros criterios para determinar la exactitud de las sucesiones anteriores están
dados en el siguiente teorema.

Teorema 1.1.2. Sean M1 , M2 y M3 A-módulos. Entonces,

1
2 CAPÍTULO 1. ELEMENTOS BÁSICOS DE ÁLGEBRA HOMOLÓGICA

f g
(i) La sucesión M1 −→ M2 −→ M3 −→ 0 es exacta si, y sólo si, para todo
A-módulo N la sucesión de Z-módulos
g∗ f∗
0 −→ HomA (M3 , N ) −→ HomA (M2 , N ) −→ HomA (M1 , N )

es exacta.
f g
(ii) La sucesión 0 −→ M1 −→ M2 −→ M3 es exacta si, y sólo si, para todo
A-módulo P la sucesión de Z-módulos
f∗ g∗
0 −→ HomA (P, M1 ) −→ HomA (P, M2 ) −→ HomA (P, M3 )

es exacta.
Demostración. Presentamos la prueba de la parte (i). La demostración de (ii) es
similar.
f
⇒): en primer lugar nótese que si M1 −
→ M2 es un A-homomorfismo y N es un
A-módulo, entonces se tiene el homomorfismo de grupos abelianos
f∗
HomA (M2 , N ) −→ HomA (M1 , N )
definido por
f ∗ (h) := hf, (1.1.1)
donde h ∈ HomA (M2 , N ). Obsérvemos que f ∗ invirtió el sentido de la flecha f (en
la prueba de (ii) el homomorfismo f∗ se define por

f ∗ (h) := f h, (1.1.2)

donde h ∈ HomA (N, M1 ). Notemos que en este caso f∗ no invirtió el sentido de la


flecha f ).
Veamos que g ∗ es inyectivo: si g ∗ (h) = hg = 0, debemos probar que h = 0. Sea
x ∈ M3 entonces como g es sobreyectiva existe z ∈ M2 tal que g(z) = x, luego
h (g(z)) = h (x) = 0. Esto prueba que h se anula en cada punto de su dominio, es
decir, h es nula.
Ahora probemos que Im (g ∗ ) = ker (f ∗ ): sea u ∈ Im (g ∗ ) entonces u = g ∗ (h) =
hg, aplicamos f ∗ y obtenemos f ∗ (u) = uf = h (gf ), pero como Im(f ) = ker(g),
entonces gf = 0 y f ∗ (u) = 0, de modo que u ∈ ker (f ∗ ) . Resta ver que ker (f ∗ ) ⊆
Im (g ∗ ): sea t ∈ ker (f ∗ ) entonces f ∗ (t) = 0 = tf . Buscamos un homomorfismo
h : M3 → N de tal forma g ∗ (h) = t = hg. Sea x ∈ M3 , como g es sobre existe z ∈ M2
tal que g (z) = x luego h (g (z)) = h (x); podemos entonces definir h(x) := t(z). h
es un A-homomorfismo bien definido: si z1 , z2 son tales que g (z1 ) = x = g (z2 )
entonces deberı́amos tener que t (z1 ) = t (z2 ). En efecto, g (z1 − z2 ) = 0, es decir,
z1 − z2 ∈ ker(g) = Im(f ), por tanto existe r ∈ M1 tal que z1 − z2 = f (r), y
1.1. SUCESIONES EXACTAS 3

ası́ t (f (r)) = t (z1 − z2 ) = 0, de donde t (z1 ) = t (z2 ). Es fácil ver que h es un


A-homomorfismo y es por supuesto el homomorfismo deseado.
⇐): supongamos que la sucesión
g∗ f∗
0 → HomA (M3 , N ) −
→ HomA (M2 , N ) −→ HomA (M1 , N )

es exacta para cada A-módulo N . Veamos entonces que g es sobreyectivo: en calidad


de N escogemos a M3 /Im(g) y sea j : M3 → M3 /Im(g) el homomorfismo canónico.
Entonces, g ∗ (j) = jg = 0 (en efecto, jg(x) = g(x) = 0); pero como g ∗ es inyectiva
entonces j = 0, es decir, para cada x ∈ M3 se tiene que x = 0, es decir, x ∈ Im(g),
y de esta forma M3 = Im(g).
Veamos ahora que Im(f ) = ker(g): nótese en primer lugar que Im(g ∗ ) ⊆ ker(f ∗ ),
luego para cada h ∈ HomA (M3 , N ) se tiene que f ∗ g ∗ (h) = 0, es decir, hgf = 0,
hagamos entonces N = M3 y h = iM3 la idéntica de M3 , entonces gf = 0, con lo
cual Im(f ) ⊆ ker(g). Ahora tomemos N = M2 /Im(f ) y sea j : M2 → M2 /Im(f )
la canónica. Entonces, f ∗ (j) = jf = 0, es decir, j ∈ ker (f ∗ ) ⊆ Im(g ∗ ), luego
existe un homomorfismo t ∈ HomA (M3 , N ) tal que j = g ∗ (t) = tg. Por tanto,
Im(f ) = ker(j) ⊇ ker(g).
Definición 1.1.3. Una sucesión exacta finita de la forma
f g
0 −→ M1 −→ M2 −→ M3 −→ 0
se dice que es una sucesión exacta corta. Dos sucesiones exactas cortas
f g
0 −→ M1 −→ M2 −→ M3 −→ 0
y
a b
0 −→ N1 −→ N2 −→ N3 −→ 0
se dicen equivalentes si existen A-isomorfismos h, k, l tales que el siguiente dia-
grama conmuta:
f g
0 → M1 −→ M2 −→ M3 → 0
↓ h ↓ k ↓ l
a b
0 → N1 −→ N2 −→ N3 → 0
f g
Proposición 1.1.4. Sea 0 −→ M1 −→ M2 −→ M3 −→ 0 una sucesión exacta
corta. Entonces, esta sucesión es equivalente a la sucesión
ι j
0 −→ ker(g) −→ M2 −→ M2 / ker(g) −→ 0
donde ι es la inclusión y j es el homomorfismo canónico.
4 CAPÍTULO 1. ELEMENTOS BÁSICOS DE ÁLGEBRA HOMOLÓGICA

Demostración. Debemos definir A-homomorfismos h : M1 → ker(g), k : M2 → M2


y l : M3 → M2 / ker(g). En calidad de k tomamos la idéntica iM2 ; para l sea z ∈ M3
y sea x ∈ M2 tal que g(x) = z, definimos l(z) := j(x) = x, donde j : M2 →
M2 / ker(g) es el homomorfismo canónico. Nótese que l está correctamente definida
y es además un A-isomorfismo que satisface l (g(x)) = x = j(iM2 (x)), es decir,
lg = jiM2 . Definamos ahora el A-homomorfismo h; sea v ∈ M1 , entonces f (v) ∈
Im(f ) = ker(g), de donde g(f (v)) = 0. Definimos entonces h(v) := f (v), nótese que
h es claramente un A-isomorfismo que satisface ιh = iM2 f .
Definición 1.1.5. Sea M un A-módulo; se dice que M es de presentación finita
(o también, finitamente presentado) si existe una sucesión exacta en la forma
f g
An −
→ Am −
→ M → 0, (1.1.3)

donde An y Am son A-módulos libres de bases finitas con n ≥ 1 y m ≥ 1 elementos,


respectivamente.
Notése que el módulo nulo es de presentación finita con n = m ≥ 1 cualquiera,
g = 0 y f = iAm . Otras formas de expresar que un módulo tiene una presentación
finita son las siguientes.
Proposición 1.1.6. Sea M un A-módulo. Entonces las siguientes condiciones son
equivalentes:
(i) M es de presentación finita.

(ii) Existe una sucesión excacta en la forma


ι g
→ Am −
0→K− → M → 0,

donde m ≥ 1 y K es un A-módulo finitamente generado.

(iii) Existen un entero m ≥ 1 y un A-submódulo finitamente generado K de Am


tales que M ∼
= Am /K.
Demostración. (i) ⇒ (ii): tomando K := ker(g) = Im(f ) en (1.1.3) se obtiene la
sucesión exacta corta deseada, con K generado por n elementos.
(ii) ⇒ (i): sea K generado por n elementos, entonces se tiene el homomorfismo
f0
natural sobreyectivo An − → K que envia la base canónica de An en los generadores
0
de K; tomando f := ιf resulta la exactitud de (1.1.3).
(ii) ⇒ (iii): tenemos que M ∼
= Am / ker(g) = Am /Im(ι) ∼ = Am /K.
(iii) ⇒ (ii): sea α : M → Am /K un isomorfismo, entonces esta implicación
resulta evidente tomamdo como ι a la inclusión de K en Am y g := αj, donde
j : Am → Am /K es el homomorfismo canónico.
1.1. SUCESIONES EXACTAS 5

Observemos que todo módulo de presentación finita es finitamente generado. El


recı́proco es válido para anillos noetherianos.
Proposición 1.1.7. Sea AA noetheriano. Entonces, cada A-módulo finitamente
generado M es de presentación finita.
Demostración. Para M = 0 la afirmación es trivial. Sea M = {x1 , . . . , xm i no nulo;
definimos g : Am → M por g(ei ) := xi , 1 ≤ i ≤ m, donde {ei }m
i=1 es la base canónica
de Am . Como Am es un módulo noetheriano, K := ker(g) es finitamente generado
ι g
y se tiene entonces la suceción exacta 0 → K − → Am − → M → 0, donde ι es la
inclusión.
Notemos que si M tiene la presentación finita (1.1.3), entonces podemos calcular
la matriz de f en las bases canónicas de An y Am : en efecto, sea X := {e0j }nj=1 la
base canónica de An y sea Y := {ei }m m
i=1 la base canónica de A , entonces

f (e0j ) = e1 · f1j + · · · + em · fmj , 1 ≤ j ≤ n,


y la matriz de f en las bases X y Y se define por mX,Y (f ) := F := [fij ] ∈ Mm×n (A)
(véase [17]). Se dice que F es una matriz de presentación de M . Es claro que
la matriz de presentación de un módulo M no es única ya que los sistemas de
generadores de M son diversos. Veremos enseguida cómo están relacionadas dos de
estas matrices.
Proposición 1.1.8. Sean
f g
An −
→ Am −
→M →0
f0 g0
As −
→ Ar −
→M →0

dos presentaciones finitas de M con matrices F y F 0 , respectivamente. Entonces


existen matrices P ∈ Mr×m (A) y Q ∈ Ms×n (A) tales que P F = F 0 Q.
Demostración. Sea {ei }m i=1 la base canónica de A
m
y sea xi := g(ei ) ∈ M . Como
g es sobreyectivo, existen z1 , . . . , zm ∈ A tales que g 0 (zi ) = xi . Definimos el A-
0 r

homomorfismo p : Am → Ar por p(ei ) := zi , 1 ≤ i ≤ m. Notemos entonces que


iM g = g 0 p, luego Im(pf ) ⊆ ker(g 0 ) = Im(f 0 ). Sea {e0j }nj=1 la base canónica de An ,
definimos entonces el A-homorfismo q : An → As por q(e0j ) := yj , con yj ∈ As tal que
pf (e0j ) = f 0 (yj ). Resulta, pf = f 0 q. Calculando las matrices de los homomorfismos
p, f, f 0 , q en las bases canónicas se obtiene el resultado.
Definición 1.1.9. Sea M un A-módulo; una resolución libre de M es una suce-
sión exacta en la forma
fn+1 fn fn−1 f2 f1 f0
R : · · · −−→ Mn −→ Mn−1 −−→ · · · −
→ M1 −
→ M0 −
→ M −→ 0,
6 CAPÍTULO 1. ELEMENTOS BÁSICOS DE ÁLGEBRA HOMOLÓGICA

donde Mi es libre para cada i ∈ N.


Proposición 1.1.10. Cada A-módulo M tiene una resolución libre.
f0
Demostración. Existe un módulo libre M0 y un homomorfismo sobreyectivo M0 − →
M (en calidad de M0 podemos tomar el A-módulo libre A(M ) con una base de
cardinalidad igual a la de M , y como f0 elegimos la función que envia la base
canónica de A(M ) en los elementos de M , véase [16]). Sea K0 := ker(f0 ), entonces
f1
para K0 podemos repetir este mismo razonamiento en econtrar M1 −
→ K0 con M1
f1 f0
libre. Resulta entonces la sucesión exacta M1 −
→ M0 −
→ M −→ 0. Por recurrencia
se completa la prueba.
Concluimos esta sección con dos lemas famosos relativos a sucesiones exactas
finitas.
Lema 1.1.11 (Lema de los cinco). Si el siguiente diagrama de A-módulos y
homomorfismos es conmutativo y las filas son exactas
f1 f2 f3 f4
M1 −−−→ M2 −−−→ M3 −−−→ M4 −−−→ M5
    
t t t t t
y1 y2 y3 y4 y5
1 h 2 h 3 4 h h
N1 −−−→ N2 −−−→ N3 −−−→ N4 −−−→ N5
entonces,
(i) Si t2 y t4 son sobreyectivos y t5 es inyectivo, entonces t3 es sobreyectivo.
(ii) Si t2 y t4 son inyectivos y t1 es sobreyectivo, entonces t3 es inyectivo.
(iii) Si t1 , t2 , t4 , t5 son isomorfismos, entonces t3 es un isomorfismo.
Demostración. (i) Sea n3 ∈ N3 , entonces existe m4 ∈ M4 tal que t4 (m4 ) = h3 (n3 ),
luego h4 (t4 (m4 )) = h4 (h3 (n3 )) = 0 = t5 f4 (m4 ), de donde m4 ∈ ker(f4 ) = Im(f3 ).
Por tanto, existe m3 ∈ M3 tal que f3 (m3 ) = m4 , de donde t4 (f3 (m3 )) = t4 (m4 ) =
h3 (n3 ), y en consecuencia h3 (t3 (m3 )) = h3 (n3 ). Esto último indica que n3 − t3 (m3 ) ∈
ker(h3 ) = Im(h2 ), luego existe n2 ∈ N2 tal que n3 − t3 (m3 ) = h2 (n2 ), es decir, n3 =
t3 (m3 ) + h2 (n2 ). Pero existe m2 ∈ M2 tal que t2 (m2 ) = n2 , de donde n3 = t3 (m3 ) +
h2 (t2 (m2 )) = t3 (m3 ) + t3 (f2 (m2 )) = t3 (m3 + f2 (m2 )), es decir, t3 es sobreyectivo.
(ii) Sea m3 ∈ M3 tal que t3 (m3 ) = 0, entonces h3 (t3 (m3 )) = 0 = t4 (f3 (m3 )), y por
tanto, m3 ∈ ker(f3 ) = Im(f2 ). Existe entonces m2 ∈ M2 tal que f2 (m2 ) = m3 , de
donde t3 (f2 (m2 )) = 0 = h2 (t2 (m2 )), y de esta manera t2 (m2 ) ∈ ker(h2 ) = Im(h1 ),
luego t2 (m2 ) = h1 (n1 ), con n1 ∈ N1 . Existe m1 ∈ M1 tal que t1 (m1 ) = n1 , y
por tanto t2 (m2 ) = h1 (t1 (m1 )) = t2 (f1 (m1 )), es decir, m2 = f1 (m1 ), con lo cual,
m3 = f2 (f1 (m1 )) = 0. Esto significa que t3 es inyectivo.
(iii) Esta parte es consecuencia directa de las dos anteriores.
1.1. SUCESIONES EXACTAS 7

Corolario 1.1.12. Si el siguiente diagrama de A-módulos y homomorfismos es con-


mutativo y las filas son exactas
f1 f2
0 −−−→ M1 −−−→ M2 −−−→ M3 −−−→ 0
  
f  g
y yh y
f1 f2
0 −−−→ N1 −−−→ N2 −−−→ N3 −−−→ 0
entonces,
(i) Si f y g son sobreyectivos, entonces h es sobreyectivo.
(ii) Si f y g son inyectivos , entonces h es inyectivo.
(iii) Si f, g son isomorfismos, entonces h es un isomorfismo.
Demostración. Consecuencia directa del lema anterior.
Proposición 1.1.13. Si el siguiente diagrama de A-módulos y homomorfismos es
conmutativo, las filas son exactas y además f, h, g son inyectivos,
f1 f2
0 −−−→ M1 −−−→ M2 −−−→ M3 −−−→ 0
  
f  g
y yh y
g1 g2
0 −−−→ N1 −−−→ N2 −−−→ N3 −−−→ 0
entonces, h es un isomorfismo si, y sólo si, f y g son isomorfismos.
Demostración. ⇒): Sea n1 ∈ N1 , entonces g1 (n1 ) ∈ N2 y existe m2 ∈ M2 de tal
forma que h(m2 ) = g1 (n1 ); se tiene entonces que gf2 (m2 ) = g2 h(m2 ) = g2 g1 (n1 ) = 0,
pero como g es un monomorfismo entonces f2 (m2 ) = 0 y de esta manera m2 ∈
ker(f2 ) = Im(f1 ); existe entonces m1 ∈ M1 tal que f1 (m1 ) = m2 , con lo cual
hf1 (m1 ) = h(m2 ) = g1 f (m1 ) = g1 (n1 ), pero g1 es inyectivo, luego f (m1 ) = n1 .
Ası́ pues, f es sobreyectivo, y por lo tanto, biyectivo.
Sea ahora n3 ∈ N3 , entonces existe n2 ∈ N2 tal que g2 (n2 ) = n3 ; sea m2 ∈ M2
tal que h(m2 ) = n2 , entonces g2 h(m2 ) = g2 (n2 ) = n3 = gf2 (m2 ), es decir, g es
sobreyectivo, y por lo tanto, biyectivo.
⇐): Consecuencia directa del corolario anterior.
Lema 1.1.14 (Lema de la serpiente). Supóngase que el siguiente diagrama de
A-módulos y homomorfismos es conmutativo y las filas son exactas
f1 f2
M1 −−−→ M2 −−−→ M3 −−−→ 0
  
t t t
y1 y2 y3
1 h 2 h
0 −−−→ N1 −−− → N2 −−− → N3
Entonces existe una sucesión exacta en la forma
8 CAPÍTULO 1. ELEMENTOS BÁSICOS DE ÁLGEBRA HOMOLÓGICA

d
ker(t1 ) −
→ ker(t2 ) −
→ ker(t3 ) −
→ coker(t1 ) −
→ coker(t2 ) −
→ coker(t3 ),
donde d(k3 ) := h−1 −1
1 t2 f2 (k3 ) + Im(t1 ), con k3 ∈ ker(t3 ). Además, si f1 es inyectivo,
entonces ker(t1 ) −
→ ker(t2 ) es inyectivo, y si h2 es sobreyectivo, entonces coker(t2 ) −

coker(t3 ) es sobreyectivo.
Demostración. En el siguiente diagrama los homomorfismos li son las inyecciones,
los ji son los homomorfismos canónicos y coker(ti ) = Ni /Im(ti ), i = 1, 2, 3:
g1 g2 d
ker(t1 ) −−−→ ker(t2 ) −−−→ ker(t3 ) −−−→ · · ·
  
l l l
y1 y2 y3
f1 f2
M1 −−−→ M2 −−−→ M3 −−−→ 0
  
t t t
y1 y2 y3
h
1 h2
0 −−−→ N1 −−−→ N2 −−−→ N3
  
j j j
y1 y2 y3
d p1 p2
· · · −−−→ coker(t1 ) −−−→ coker(t2 ) −−−→ coker(t3 )
Notemos que las tres columnas del diagrama anterior son exactas. Debemos definir
los homomorfismos gi , pi , i = 1, 2, y probar que la sucesión
g1 g2 d p1 p2
ker(t1 ) −
→ ker(t2 ) −
→ ker(t3 ) −
→ coker(t1 ) −
→ coker(t2 ) −
→ coker(t3 ),
es exacta. Dividiremos la prueba en varios pasos.
Paso 1. Sea k1 ∈ ker(t1 ), entonces t1 (k1 ) = 0 y h1 t1 (k1 ) = 0 = t2 f1 (k1 ), luego
f1 (k1 ) ∈ ker(t2 ). De igual manera, sea k2 ∈ ker(t2 ), entonces t2 (k2 ) = 0 y h2 t2 (k2 ) =
0 = t3 f2 (k2 ), es decir, f2 (k2 ) ∈ ker(t3 ). Esto permite definir g1 y g2 en la siguiente
forma
g1 (k1 ) := f1 (k1 ), g2 (k2 ) := f2 (k2 ).
Notemos entonces que gi es la restricción de fi a ker(ti ), i = 1, 2; además, f1 l1 = l2 g1
y f2 l2 = l3 g2 . Es claro que si f1 es inyectivo, entonces g1 es inyectivo.
Paso 2. Los homomorfismos p1 , p2 se definen en forma natural, es decir,
p1 (n1 ) := h1 (n1 ), p2 (n2 ) := h2 (n2 ),
con ni := ni + Im(ti ), ni ∈ Ni , hi (ni ) := h(ni ) + Im(ti+1 ), i = 1, 2. Observemos
que p1 j1 = j2 h1 y p2 j2 = j3 h2 . Es claro que si h2 es sobreyectivo, entonces p2 es
sobreyectivo.
Paso 3. d está bien definida: sea k3 ∈ ker(t3 ) ⊆ M3 , entonces existe m2 ∈ M2
tal que f2 (m2 ) = k3 , de donde t3 f2 (m2 ) = t3 (k3 ) = 0 = h2 t2 (m2 ), lo cual indica
que t2 (m2 ) ∈ ker(h2 ) = Im(h1 ). Por tanto, existe n1 ∈ N1 tal que t2 (m2 ) = h1 (n1 );
entonces definimos
1.1. SUCESIONES EXACTAS 9

d(k3 ) := n1 = n1 + Im(t1 ), con h1 (n1 ) = t2 (m2 ) y f2 (m2 ) = k3 .

Esta definición puede escribirse también en la forma d(k3 ) := h−1 −1


1 t2 f2 (k3 )+Im(t1 ).
Si m02 ∈ M2 es tal que f2 (m02 ) = k3 = f2 (m2 ), entonces m2 − m02 ∈ ker(f2 ) = Im(f1 ),
luego m2 − m02 = f1 (m1 ), con m1 ∈ M1 ; además, t3 f2 (m02 ) = t3 (k3 ) = 0 = h2 t2 (m02 ),
de donde t2 (m02 ) ∈ ker(h2 ) = Im(h1 ), y en consecuencia existe n01 ∈ N1 tal que
t2 (m02 ) = h1 (n01 ). Restando resulta t2 (m2 ) − t2 (m02 ) = h1 (n1 ) − h1 (n01 ), es decir,
t2 (m2 − m02 ) = h1 (n1 − n01 ) = t2 (f1 (m1 )) = h1 t1 (m1 ), pero como h1 es inyectiva,
entonces n1 − n01 = t1 (m1 ) ∈ Im(t1 ). Esto demuestra que d está bien definida.
Paso 4. Im(g1 ) = ker(g2 ): sea g1 (k1 ) ∈ Im(g1 ), con k1 ∈ ker(t1 ), entonces
g2 g1 (k1 ) = g2 (f1 (k1 )) = f2 (f1 (k1 )) = 0, es decir, Im(g1 ) ⊆ ker(g2 ). Sea ahora
k2 ∈ ker(g2 ), con k2 ∈ ker(t2 ), entonces g2 (k2 ) = 0 = f2 (k2 ), luego k2 ∈ ker(f2 ) =
Im(f1 ), de donde k2 = f1 (k1 ), con k1 ∈ M1 . Si mostramos que k1 ∈ ker(t1 ), entonces
k2 = g1 (k1 ) ∈ Im(g1 ), con lo cual ker(g2 ) ⊆ Im(g1 ). Tenemos, h1 t1 (k1 ) = t2 f1 (k1 ) =
t2 (k2 ) = 0, pero como h1 es inyectiva, entonces t1 (k1 ) = 0.
Paso 5. Im(g2 ) = ker(d) : sea k3 = g2 (k2 ) ∈ Im(g2 ), con k2 ∈ ker(t2 ) ⊆ M2 ;
según vimos arriba g2 (k2 ) = f2 (k2 ), luego en la difinición de d podemos tomar
m2 := k2 , y entonces t2 (m2 ) = 0 = h1 (n1 ), pero como h1 es inyectivo entonces
n1 = 0 y de esta manera d(k3 ) = 0, y por tanto Im(g2 ) ⊆ ker(d). Recı́procamente,
sea k3 ∈ ker(d) ⊆ ker(t3 ), entonces n1 ∈ Im(t1 ), digamos n1 = t1 (m1 ), con m1 ∈ M1 ,
luego h1 t1 (m1 ) = t2 (m2 ), con f2 (m2 ) = k3 . Resulta, t2 (m2 ) = t2 f1 (m1 ), con lo cual
k2 := m2 −f1 (m1 ) ∈ ker(t2 ). De aqui obtenemos que f2 (k2 ) = f2 (m2 )−f2 (f1 (m1 )) =
f2 (m2 ) = k3 , de lo cual resulta k3 = g2 (k2 ) ∈ Im(g2 ).
Paso 6. Im(d) = ker(p1 ): sea n1 = d(k3 ) ∈ Im(d), entonces p1 (n1 ) = h1 (n1 ) =
t2 (m2 ) = 0 ya que t2 (m2 ) ∈ Im(t2 ). Esto demuestra que Im(d) ⊆ ker(p1 ). Sea
ahora n1 ∈ ker(p1 ), con n1 ∈ N1 , entonces h1 (n1 ) = 0, es decir, h1 (n1 ) ∈ Im(t2 ),
de donde h1 (n1 ) = t2 (m2 ), con m2 ∈ M2 . Sea k3 := f2 (m2 ), pero notemos que
t3 (k3 ) = t3 f2 (m2 ) = h2 t2 (m2 ) = h2 h1 (n1 ) = 0, es decir, k3 ∈ ker(t3 ). Con esto
d(k3 ) = n1 , y entonces ker(p1 ) ⊆ Im(d).
Paso 7. Im(p1 ) = ker(p2 ): sea p1 (n1 ) ∈ Im(p1 ), con n1 ∈ N1 , entonces p2 p1 (n1 ) =
p2 (h1 (n1 )) = h2 h1 (n1 ) = 0, es decir, Im(p1 ) ⊆ ker(p2 ). Recı́procamente, sea n2 ∈
ker(p2 ), con n2 ∈ N2 , entonces h2 (n2 ) ∈ Im(t3 ) y existe m3 ∈ M3 tal que h2 (n2 ) =
t3 (m3 ); como f2 es sobreyectivo existe m2 ∈ M2 tal que f2 (m2 ) = m3 , luego h2 (n2 ) =
t3 f2 (m2 ) = h2 t2 (m − 2). Esto indica que n2 − t2 (m2 ) ∈ ker(h2 ) = Im(h1 ), con lo
cual n2 − t2 (m2 ) = h1 (n1 ), con n1 ∈ N1 . Resulta, n2 − h1 (n1 ) = t2 (m2 ) ∈ Im(t2 ),
por tanto n2 = h1 (n1 ) = p1 (n1 ) ∈ Im(p1 ). En consecuencia, ker(p2 ) ⊆ Im(p1 ). Con
esto completamos la prueba del teorema.
10 CAPÍTULO 1. ELEMENTOS BÁSICOS DE ÁLGEBRA HOMOLÓGICA

1.2. Módulos proyectivos


El teorema 1.1.2 indica que los operadores HomA (P, −) y HomA (−, N ) preservan
la exactitud de sucesiones por la izquierda, cualesquiera que sean los módulos P y
N . Consideremos ahora la sucesión exacta

0 −→ 2Z −→ Z −→ Z2 −→ 0,
al aplicar el funtor HomZ (Z2 , −) (véase [19]) resulta la sucesión
HomZ (Z2 , 0) −→ HomZ (Z2 , 2Z) −→ HomZ (Z2 , Z) −→ HomZ (Z2 , Z2 ) −→ HomZ (Z2 , 0),

es decir, resulta la sucesión

0 −→ 0 −→ 0 −→ Z2 −→ 0

la cual no es exacta en Z2 . Ası́ pues, en general, si P es un A-módulo, el fun-


tor HomA (P, −) no preserva la exactitud a derecha. En forma similar, aplicando
HomZ (−, Z2 ) a

0 −→ 2Z −→ Z −→ Z2 −→ 0
obtenemos
HomZ (0, Z2 ) −→ HomZ (Z2 , Z2 ) −→ HomZ (Z, Z2 ) −→ HomZ (2Z, Z2 ) −→ HomZ (0, Z2 ),

es decir,
i 0
0 −→ Z2 −→
Z
Z2 −→ Z2 −→ 0

la cual no es exacta en el último Z2 . Entonces, en general, si N es un A-módulo, el


funtor HomA (−, N ) no preserva la exactitud a derecha.

Definición 1.2.1. Un A-módulo P es proyectivo si HomA (P, −) preserva la exac-


titud a derecha. Se dice que un A-módulo N es inyectivo si HomA (−, N ) preserva
la exactitud a derecha.

Ası́ pues, P es proyectivo si, y sólo si, HomA (P, −) envia sucesiones exactas
cortas en sucesiones exactas cortas, y de igual manera, N es inyectivo si, y sólo
si, HomA (−, N ) envia sucesiones exactas cortas en sucesiones exactas cortas. En
el lenguaje de las categorı́as y los funtores (véase [19]), P es proyectivo si, y sólo
si, HomA (P, −) es un funtor exacto y N es inyectivo si, y sólo si, HomA (−, N ) es
un funtor exacto. A continuación estudiaremos algunas propiedades de los módulos
proyectivos. En la sección 1.7 consideraremos los módulos inyectivos.

Teorema 1.2.2. Sea P un A-módulo. Entonces las siguientes condiciones son equi-
valentes:
1.2. MÓDULOS PROYECTIVOS 11

(i) P es proyectivo.

(ii) Para cada homomorfismo sobreyectivo f y cada homomorfismo g, existe un


homomorfismo h tal que el siguiente diagrama conmuta:

Pp
pp
h p p
pp
g
p
?
M f
- N

f
(iii) Cada homomorfismo sobreyectivo M −→ P es hendido.
ι f
Demostración. (i)⇒(ii): consideremos la sucesión exacta 0 −→ ker(f ) −→ M −→
N −→ 0, como P es proyectivo, entonces se tiene la siguiente sucesión exacta

∗ ι f∗
0 −→ HomR (P, ker(f )) −→ HomR (P, M ) −→ HomR (P, N ) −→ 0
con lo cual f∗ es sobreyectivo y entonces (ii) se cumple.
g f
(ii)⇒(i): sea 0 −→ L −→ M −→ N −→ 0 una sucesión exacta. Teniendo en
cuenta (ii) y que HomR (P, ) es exacto a izquierda, resulta entonces que
g∗ f∗
0 −→ HomR (P, L) −→ HomR (P, M ) −→ HomR (P, N ) −→ 0
es exacta y P es entonces proyectivo.
f
(ii)⇒(iii): sea M −→ P un homomorfismo sobreyectivo, y consideremos el ho-
momorfismo idéntico iP : P −→ P , según (ii) existe un homomorfismo h : P −→ M
tal que f h = iP . Esto garantiza que f es hendido.
(iii)⇒(ii): sea L := {(m, p) ∈ M × P | f (m) = g(p)} y consideremos las funciones
πM : L → M , πM (m, p) := m, πP : L → P , πP (m, p) := p. Nótese que πP es
sobreyectivo y además gπP = f πM . Por la condición (iii), existe un homomorfismo
t : P −→ L tal que πP t = iP . Definimos h := πM t, y entonces f h = f πM t = gπP t =
g.

Corolario 1.2.3. Todo módulo libre es proyectivo.

Demostración. Consecuencia directa de la parte (iii) del teorema anterior.

Proposición 1.2.4. Sea {Pi }i∈C una familia de A-módulos. Entonces,


L
i∈C Pi es proyectivo ⇐⇒ ∀i ∈ C , Pi es proyectivo.
12 CAPÍTULO 1. ELEMENTOS BÁSICOS DE ÁLGEBRA HOMOLÓGICA

Demostración. ⇒): sea i ∈ C, veamos que Pi es proyectivo. Consideremos los homo-


morfismos f y g del siguiente diagrama, con f sobreyectivo:

Pi

g
?
M f
- N

Buscamos un homomorfismo h : Pi → M tal que f h = g. Para esto consideremos el


las proyecciones πi : ⊕Pi → Pi , se tiene entonces el siguiente diagrama

⊕Pi

πi
?
Pi

g
?
M f
- N

Puesto que ⊕Pi es proyectivo existe un homomorfismo t : ⊕Pi → M tal que f t =


gπi . Ahora consideremos la inyección canónica µi : Pi → ⊕Pi . Entonces la función
buscada es h := tµi : Pi → M .

⊕Pi
 6
µi πi
 ?
t


Pi

 g
 h
 ?
M f
- N

En efecto, f h = f tµi = gπi µi = g.


⇐) Tengamos en cuenta ahora en el siguiente diagrama:

⊕Pi

g
?
M f
- N
1.2. MÓDULOS PROYECTIVOS 13

y consideremos para cada i la inyección canónica,

Pi

µi
?
⊕Pi

g
?
M f
- N

Por la proyectividad de Pi existe un homomorfismo hi : Pi → M tal que f hi = gµi .


Por la propiedad universal de ⊕Pi , existe un homomorfismo h : ⊕Pi → M tal que
hµi = hi :
Pi

 µi
 ?
hi
 ⊕Pi


 g
 h
 ?
M f
- N

Entonces que para cada i se tiene que (f h) µi = gµi , y por la unicidad en la propiedad
universal se tiene que f h = g. Esto muestra que ⊕Pi es proyectivo.

Corolario 1.2.5. P es proyectivo si, y sólo si, P es sumando directo de un módulo


libre.

Demostración. ⇒): existe F libre y un homomorfismo sobreyectivo f : F → P ,


basta entonces aplicar el teorema 1.2.2.
⇐): esto es consecuencia directa de la proposición anterior y del corolario 1.2.3.

Corolario 1.2.6. Si P es un módulo proyectivo f.g., entonces P es un módulo de


presentación finita.

Demostración. Existe n ≥ 1 y un homomorfismo sobreyectivo f : An → P , luego


An ∼
= K ⊕ P , con lo cual K es f.g. y P ∼
= An /K.
Ejemplo 1.2.7. No todo módulo proyectivo es libre: sea e2 = e ∈ A, un idempon-
tente no trivial de un anillo A. Entonces, A = eA ⊕ (1 − e) A. En particular, sea
A = Z6 , e = 3, entonces Z6 = 3Z6 ⊕ 4Z6 y 3Z6 = {0, 3} no es libre.
14 CAPÍTULO 1. ELEMENTOS BÁSICOS DE ÁLGEBRA HOMOLÓGICA

Una caracterización adicional de los módulos proyectivos es dada por el siguiente


teorema.

Teorema 1.2.8 (Teorema de la base proyectiva). Sean A un anillo y M un


A-módulo. M es proyectivo si, y sólo si, existe una colección {xi }i∈C de elementos
de M y una colección {φi : M → A}i∈C de A-homomorfismos tales que:

(i) Dado x ∈ M se tiene que φi (x) = 0, para casi todo i ∈ C.


P
(ii) Dado x ∈ M, x = i∈C xi · φi (x).

Demostración. ⇒): sea ψ : F → M un homomorfismo sobreyectivo, donde F es un


P φ : M → F tal que ψφ = iM , sea {fi }i∈C una base
R-módulo libre, entonces existe
de F ; dado x ∈ M, φ(x) = i∈C fi · ai , donde ai = 0 para casi todo i ∈ C. Definimos
xi := ψ(fi ) y los homomorfismos

φi : M → A
x 7→ φi (x) := ai .
P P P
Nótese que x = (ψφ)(x) = i∈C ψ(fi · ai ) = i∈I xi · ai = i∈I xi · φi (x).
⇐): sea F = A(C) el módulo libre cuya base tiene cardinalidad igual a la del
conjunto C, y sea {fi }i∈C una base de F ; definimos ψ :PF → M por ψ(fi ) := xi ,
entonces ψ es sobre. Sea φ : M → F definido porP φ(x) := i∈C fi ·φP
i (x), φ es entonces
un A-homomorfismo y se tiene que (ψφ)(x) = i∈C ψ(fi ·φi (x)) = i∈C xi ·φi (x) = x,
es decir, ψφ = iM .
Para los módulos proyectivos f.g. es posible dar una caracterización matricial.

Proposición 1.2.9. Sean A un anillo y M un A-módulo. Entonces, M es proyectivo


f.g. si, y sólo si, existe una matriz cuadrada idempotente F sobre A tal que M =
hF i := módulo generado por las columnas de F .

Demostración. ⇒): Si M = 0, entonces F = 0; sea M 6= 0, entonces existe n ≥ 1 y


un A-módulo M 0 de tal forma que An = M ⊕ M 0 ; sea f : An → An la proyección
sobre M y sea F la matriz de f en la base canónica de An . Se tiene entonces que
f 2 = f y F 2 = F ; notemos que M = Im(f ) = hF i.
⇐): Sea f : An → An el A-homomorfismo definido por F (al vector canónico ei
le asignamos la i-ésima columna de F ); de F 2 = F resulta f 2 = f , además, puesto
que M = hF i, entonces Im(f ) = M y en consecuencia M es sumando directo de
An , es decir, M es proyectivo f.g.
Un resultado clásico del álgebra homológica es el lema de Schanuel que pro-
baremos enseguida. Para la demostración necesitamos una definición y un resultado
preliminar.
1.2. MÓDULOS PROYECTIVOS 15

f g
Definición 1.2.10. Una sucesión exacta corta 0 → M1 −
→ M2 −
→ M3 → 0 es
hendida si g es un homomorfismo hendido.

f g
Proposición 1.2.11. La sucesión exacta 0 → M1 −
→ M2 −
→ M3 → 0 es hendida si,
y sólo si, f es hendido.

Demostración. ⇒): supongamos que g es un homomorfismo hendido, es decir, existe


un homomorfismo g 0 : M3 → M2 tal que gg 0 = iM3 , resulta M2 = ker(g) ⊕ Im(g 0 ) =
Im(f ) ⊕ Im(g 0 ) (véase [16]). Definimos f 0 : M2 → M1 por f 0 (m2 ) := m1 , donde
m2 = f (m1 ) + z, con m1 ∈ M1 y z ∈ Im(g 0 ). Como f es inyectivo entonces f 0
está bien definido. Es claro que f 0 es un homomorfismo y además f 0 f = iM1 .
⇐): supongamos que f es hendido, entonces existe un homomorfismo f 0 : M2 →
M1 tal que f 0 f = iM1 , se tiene además que M2 = Im(f ) ⊕ ker(f 0 ); sea m3 ∈ M3 ,
existe m2 ∈ M2 tal que g(m2 ) = m3 , m2 tiene una representación única en la forma
m2 = f (m1 ) + k2 , con m1 ∈ M1 y k2 ∈ ker(f 0 ). Definimos g 0 : M3 → M2 por
g 0 (m3 ) := k2 . Veamos que g 0 está bien definido: sea m02 ∈ M2 tal que g(m02 ) = m3 y
m02 = f (m01 ) + k20 , con m01 ∈ M1 y k20 ∈ ker(f 0 ). Se tiene que m2 − m02 ∈ ker(g) =
Im(f ), luego k2 − k20 ∈ ker(f 0 ) ∩ Im(f ) = 0, es decir, k2 = k20 . Es obvio que g 0 es un
homomorfismo, además gg 0 = iM3 .

f g
Corolario 1.2.12. Si 0 → M1 − → M2 − → M3 → 0 es una sucesión exacta hendida,
g0 f0
entonces M2 ∼ = M1 ⊕M3 y la sucesión 0 → M3 − → M2 −
→ M1 → 0 es exacta hendida,
donde g 0 , f 0 son tales que gg 0 = iM3 y f 0 f = iM1 .

Demostración. En la demostración de la proposición 1.2.11 vimos que si la sucesión


f g
exacta 0 → M1 − → M2 − → M3 → 0 es hendida, entonces M2 = Im(f ) ⊕ Im(g 0 ),
luego M2 ∼ = M1 ⊕ M3 ya que f y g 0 son inyectivos. Además, la prueba mencionada
g0 f0
muestra también que se tiene la sucesión 0 → M3 − → M2 − → M1 → 0 con gg 0 = iM3
y f 0 f = iM1 . Luego, g 0 es inyectivo y f 0 es sobreyectivo. Resta ver que Im(g 0 ) =
ker(f 0 ). Con la notación de la demostración de la proposición 1.2.11 tenemos que
f 0 g 0 (m3 ) = f 0 (k2 ) = 0, es decir, Im(g 0 ) ⊆ ker(f 0 ). Finalmente, sea k2 ∈ ker(f 0 ),
entonces k2 = 0 + k2 ∈ Im(f ) ⊕ ker(f 0 ), luego m3 := g(k2 ) es tal que g 0 (m3 ) = k2 ,
es decir, ker(f 0 ) ⊆ Im(g 0 ).

Lema 1.2.13 (Lema de Schanuel). Dadas dos sucesiones exactas

0 → K1 → P1 → M → 0,
0 → K2 → P2 → M → 0,

donde P1 , P2 son proyectivos, entonces K1 ⊕ P2 ∼


= K2 ⊕ P 1 .
16 CAPÍTULO 1. ELEMENTOS BÁSICOS DE ÁLGEBRA HOMOLÓGICA

Demostración. Puesto que P1 es proyectivo se induce el siguiente diagrama conmu-


tativo
f1 g1
0 −−−→ K1 −−−→ P1 −−−→ M −−−→ 0
  
q p i
y y yM
f2 g2
0 −−−→ K2 −−−→ P2 −−−→ M −−−→ 0
donde el homomorfismo q se define de la siguiente manera: tenemos que Im(pf1 ) ⊆
ker(g2 ) = Im(f2 ), luego dado k1 ∈ K1 , pf1 (k1 ) = f2 (k2 ), con k2 ∈ K2 ; entonces
definimos q(k1 ) := k2 . q está bien definida ya que f2 es inyectiva; es claro que q es
un homomorfismo.
θ δ
Nótese que la sucesión 0 → K1 − → K2 ⊕ P 1 −→ P2 → 0 es exacta, con

θ(k1 ) := (q(k1 ), f1 (k1 )) y δ(k2 , p1 ) := p(p1 ) − f2 (k2 ).

En efecto, θ es inyectivo ya que f1 es inyectivo. Sea p2 ∈ P2 , entonces g2 (p2 ) =


g1 (p1 ), para algún p1 ∈ P1 , luego g2 p(p1 ) = g1 (p1 ) = g2 (p2 ), de donde p(p1 ) −
p2 ∈ ker(g2 ) = Im(f2 ), de esto se tiene que p(p1 ) − p2 = f2 (k2 ), y por lo tanto,
p2 = p(p1 ) − f2 (k2 ) = δ(k2 , p1 ), es decir, δ es sobreyectivo. Im(θ) ⊆ ker(δ) ya que
δ(q(k1 ), f1 (k1 )) = p(f1 (k1 )) − f2 (q(k1 )) = f2 (q(k1 )) − f2 (q(k1 )) = 0. Por último, sea
(k2 , p1 ) ∈ ker(δ), entonces δ(k2 , p1 ) = 0 = p(p1 ) − f2 (k2 ), es decir, p(p1 ) = f2 (k2 ),
luego g2 p(p1 ) = g2 f2 (k2 ) = 0 = g1 (p1 ), pero esto dice que p1 ∈ ker(g1 ) = Im(f1 ), es
decir, existe k1 ∈ K1 tal que p1 = f1 (k1 ). Resulta, θ(k1 ) = (q(k1 ), f1 (k1 )) = (k2 , p1 )
ya que f2 (q(k1 )) = pf1 (k1 ) = p(p1 ) = f2 (k2 ) y f2 es inyectivo.
Como P2 es proyectivo, entonces la sucesión anterior es hendida, luego K2 ⊕P1 ∼ =
K1 ⊕ P2 .

Se tiene la siguiente generalización del lema de Schanuel la cual se puede probar


por inducción.

Lema 1.2.14. Dadas las sucesiones exactas de A-módulos

0 → K1 → Pn → Pn−1 → · · · → P1 → M → 0
0 → K2 → Qn → Qn−1 → · · · → Q1 → N → 0

con Pi , Qi proyectivos, 1 ≤ i ≤ n, y M ∼
= N , entonces

K1 ⊕ Qn ⊕ Pn−1 ⊕ Qn−2 ⊕ · · · ∼
= K2 ⊕ Pn ⊕ Qn−1 ⊕ Pn−2 ⊕ · · ·

Demostración. Ejercicio para el lector.


1.3. MÓDULOS REFLEXIVOS 17

1.3. Módulos reflexivos


Introducimos en esta sección los módulos reflexivos y probaremos que todo módulo
proyectivo f.g. es reflexivo.

Definición 1.3.1. Sea M un A-módulo; el dual de M se denota por M ∗ y se define


por M ∗ := HomA (M, A). Se dice que M es reflexivo si el siguiente homomofismo
de A-módulos

ΦM : M → M ∗∗ , ΦM (m)(ϕ) := ϕ(m), ϕ ∈ M ∗ , m ∈ M .

es biyectivo, es decir, M ∗∗ ∼
= M.

Nótese que efectivamente ΦM es un A-homomorfismo.

Proposición 1.3.2. AA es reflexivo.

Demostración. Según la definición anterior, se tiene el A-homomorfismo


ΦA : A → A∗∗ , ΦA (a)(ϕ) := ϕ(a), ϕ ∈ A∗ , a ∈ A.
Si ΦA (a) = 0, entonces ϕ(a) = 0 para cada ϕ ∈ A∗ , en particular, iA (a) = 0 = a.
Esto demuestra que ΦA es inyectivo.
Sea Ψ ∈ A∗∗ , entonces Ψ : A∗ → A es un A-homomorfismo de A-módulos
izquierdos; sea a := Ψ(iA ), se tiene que ΦA (a) = Ψ: en efecto, sea ϕ ∈ A∗ , entonces
ΦA (a)(ϕ) := ϕ(a) = ϕ(1)a = ϕ(1)Ψ(iA ) = Ψ(ϕ(1)iA ) = Ψ(ϕ) ya que ϕ(1)iA = ϕ.
Veamos esta última igualdad: sea x ∈ A, entonces ϕ(1)iA (x) = ϕ(1)x = ϕ(x). Lo
anterior demuestra que ΦA es sobreyectivo.
De igual manera se establece que A A es reflexivo.
f g
Proposición 1.3.3. Sea 0 → M1 − → M2 − → M3 → 0 es una sucesión exacta hendida.
Entonces, M2 es reflexivo si, y sólo si, M1 y M3 son reflexivos.

Demostración. Dividimos esta prueba en varios pasos.


Paso 1. Según el corolario 1.2.12, se tiene la sucesión exacta hendida
g0 f0
0 → M3 −
→ M2 −
→ M1 → 0,
donde g 0 , f 0 son tales que gg 0 = iM3 y f 0 f = iM1 . Resultan entonces las siguientes
sucesiones exactas hendidas:
g∗ f∗
0 → M3∗ −
→ M2∗ −→ M1∗ → 0,
f 0∗ g 0∗
0 → M1∗ −→ M2∗ −→ M3∗ → 0.
De estas dos últimas a su vez se obtienen las siguientes sucesiones exactas hendidas:
18 CAPÍTULO 1. ELEMENTOS BÁSICOS DE ÁLGEBRA HOMOLÓGICA

f ∗∗ g ∗∗
0 → M1∗∗ −−→ M2∗∗ −−→ M3∗∗ → 0,
g 0∗∗ f 0∗∗
0 → M3∗∗ −−→ M2∗∗ −−→ M1∗∗ → 0.

Paso 2. Para cada A-homomorfismo f : M → M 0 se tiene que ΦM 0 f = f ∗∗ ΦM .


En efecto, sea m ∈ M , entonces ΦM 0 f (m) = ΦM 0 (f (m)) : M 0∗ → A y por el
otro lado f ∗∗ ΦM (m) = f ∗∗ (ΦM (m)) = ΦM (m)f ∗ : M 0∗ → A; sea ϕ0 ∈ M 0∗ , en-
tonces ΦM 0 (f (m))(ϕ0 ) = ϕ0 (f (m)) y por el otro lado ΦM (m)f ∗ (ϕ0 ) = ΦM (m)(ϕ0 f ) =
(ϕ0 f )(m) = ϕ0 (f (m)).
Paso 3. ⇒): Supongamos que M2 es reflexivo; se tiene el siguiente diagrama
conmutatativo, con filas exactas y ΦM2 biyectivo:
f g
0 −−−→ M1 −−−→ M2 −−−→ M3 −−−→ 0
  
Φ Φ Φ
y M1 y M2 y M3
f ∗∗ g ∗∗
0 −−−→ M1∗∗ −−−→ M2∗∗ −−−→ M3∗∗ −−−→ 0

Entonces ΦM1 es inyectivo. De igual manera se tiene el siguiente diagrama conmu-


tatativo, con filas exactas y ΦM2 biyectivo:
g0 f0
0 −−−→ M3 −−−→ M2 −−−→ M1 −−−→ 0
  
Φ Φ Φ
y M3 y M2 y M1
g 0∗∗ f 0∗∗
0 −−−→ M3∗∗ −−−→ M2∗∗ −−−→ M1∗∗ −−−→ 0

y de esta forma ΦM3 es inyectivo. Podemos entonces aplicar la proposición 1.1.13 y


concluir que ΦM1 y ΦM3 son isomorfismos, es decir, M1 y M3 son reflexivos.
Paso 4. ⇐): Esto es consecuencia del lema de los cinco y de cualquiera de los
dos diagramas del paso anterior.

Proposición 1.3.4. Para cada n ≥ 1, M1 ⊕ · · · ⊕ Mn es reflexivo si, y sólo si, Mi


es reflexivo para cada 1 ≤ i ≤ n.

Demostración. Basta probar la proposición para n = 2. Se tiene la sucesión exacta


ι π
hendida 0 → M1 − → M1 ⊕ M2 − → M2 → 0, donde ι es la inclusión canónica y π es la
proyección; el resultado se obtiene entonces de la proposición anterior.

Corolario 1.3.5. Todo módulo proyectivo f.g. es reflexivo.

Demostración. Existen n ≥ 1 y K de tal forma que An ∼ = P ⊕ K; de la proposición


anterior y la proposición 1.3.2 se obtiene que An es reflexivo, y también que P es
reflexivo.
1.4. PRODUCTO TENSORIAL 19

1.4. Producto tensorial


En esta sección construiremos a partir de dos bimódulos un nuevo bimódulo de dos
argumentos que tenga propiedades de bilinealidad; esta construcción es similar a la
realizada para el Hom en [16]. El caso particular del producto tensorial de módulos
sobre anillos conmutativos, y el estudio correspondiente de funciones multilineales
y tensores, se puede consultar en [17].
Sea A un anillo y sean M, N A-módulos a derecha e izquierda, respectivamente.
Consideremos el Z-módulo libre F := Z(M ×N ) , con base X := {µx (1)}x∈M ×N ,
donde µx es la inyección canónica de la suma directa externa. Por simplicidad en la
notación identificaremos la base X de F con el conjunto M × N . Consideremos el
Z-submódulo S ≤ F , S := hS1 ∪ S2 ∪ S3 i, donde

S1 := {(m + m0 , n) − (m, n) − (m0 , n) | m, m0 ∈ M, n ∈ N },


S2 := {(m, n + n0 ) − (m, n) − (m, n0 ) | m ∈ M, n, n0 ∈ N },
S3 := {(m · a, n) − (m, a · n) | m ∈ M, n ∈ N, a ∈ A}.

Sea M ⊗A N := F/S el Z-módulo cociente correspondiente. Este objeto se conoce


como el producto tensorial de M por N . La clase (m, n) del elemento (m, n) de F
se denota por m ⊗ n.
Algunas propiedades del Z-módulo construido son las siguientes:
(1) La función t : M × N −→ M ⊗A N definida por t(m, n) := m ⊗ n es bilineal
y balanceada, es decir,
(i) t(m + m0 , n) = t(m, n) + t(m0 , n).
(ii) t(m, n + n0 ) = t(m, n) + t(m, n0 ).
(iii) t(m.a, n) = t(m, a.n).
Las propiedades anteriores de la función t se pueden escribir también de la
siguiente manera:

(m + m0 ) ⊗ n = m ⊗ n + m0 ⊗ n,
m ⊗ (n + n0 ) = m ⊗ n + m ⊗ n0 ,
m · a ⊗ n = m ⊗ a · n.

(2) 0 ⊗ n = 0, m ⊗ 0 = 0, (−m) ⊗ n = − (m ⊗ n), m ⊗ (−n) = −(m ⊗ n).

(3) Cada elemento f de M ⊗A N se puede representar en la forma m1 ⊗ n1 +


· · · + mr ⊗ nr . Esta representación no es única: en Z2 ⊗Z Z3 se tiene que
1 ⊗ 2 = 2 ⊗ 1 = 0.
El producto tensorial se caracteriza con la siguiente propiedad universal.
20 CAPÍTULO 1. ELEMENTOS BÁSICOS DE ÁLGEBRA HOMOLÓGICA

Teorema 1.4.1. Para cada Z-módulo L y cada función bilineal y balanceada h0 :


M × N −→ L existe un único Z-homomorfismo h : M ⊗A N −→ L tal que ht = h0 :
t
M ×N - M ⊗N
p
pp
p pp
p
h0 pp
p pp h
p
?+
L

El homomorfismo h viene dado por:

h(m ⊗ n) := h0 (m, n).

Demostración. Usaremos la notación que precede al enunciado del teorema. Puesto


que F es libre con base M × N , la función h0 induce un único Z-homomorfismo
h00 : F → L tal que h00 (m, n) := h0 (m, n). Definimos entonces la función h : F/S → L
por h (z) := h00 (z), con z ∈ F . En particular, si z = (m, n) es básico, entonces
h(m ⊗ n) = h0 (m, n). Probemos que h está bien definida: sean z1 , z2 ∈ F tales que
z1 = z2 , debemos demostrar que h00 (z1 ) = h00 (z2 ). Se tiene que z1 − z2 ∈ S, entonces
z1 −z2 es una combinación lineal con coeficientes enteros de elementos de S1 ∪S2 ∪S3 ;
como h00 es un Z-homomorfismo, entonces basta mostrar que para cada sumando w
se cumple que h00 (w) = 0. Entonces para w se presentan 3 posiblidades: w ∈ S1
ó w ∈ S2 ó w ∈ S3 . En el primer caso w = (m + m0 , n) − (m, n) − (m0 , n), de donde
h00 (w) = h00 (m+m0 , n)−h00 (m, n)−h00 (m0 , n) = h0 (m+m0 , n)−h0 (m, n)−h0 (m0 , n) =
0. De igual forma se establece en los otros dos casos.
De otra parte, nótese que h es un Z-homomorfismo ya que h00 lo es. Además,
ht(m, n) = h(m ⊗ n) = h0 (m, n). Resta demostrar que h es único con la condición
ht = h0 . Sea l otro Z-homomorfismo tal que lt = h0 , entonces lt(m, n) = h0 (m, n), es
decir, l(m ⊗ n) = h(m ⊗ n). Esto muestra que l = h.

Corolario 1.4.2. Si L es un Z-módulo y h0 : M × N −→ L es una función bilineal


y balanceada tal que L satisface la propiedad universal precedente, entonces L ∼
=
M ⊗A N (isomorfismo de grupos abelianos).

Demostración. Ejercicio para el lector.

La construcción realizada anteriormente puede ser ampliada a bimódulos: sean


B M A y A NC bimódulos, entonces el grupo M ⊗A N tiene estructura de B − C-
bimódulo con las siguientes operaciones:

b · (m ⊗ n) = (b · m) ⊗ n,
(m ⊗ n) · c = m ⊗ (n · c).
1.4. PRODUCTO TENSORIAL 21

La demostración de que estas operaciones dotan al producto tensorial de estructura


de bimódulo se apoya en el siguiente razonamiento: la función h0 : M × N −→
M ⊗ N definida por h0 (m, n) := (b · m) ⊗ n es bilineal y balanceada, y por tanto
induce un homomorfismo de grupos abelianos b : M ⊗ N −→ M ⊗ N definido por
b(m ⊗ n) := (b · m) ⊗ n. Para probar que M ⊗ N es un B-módulo basta observar que
(b1 + b2 ) · (m ⊗ n) = b1 · (m ⊗ n) + b2 · (m ⊗ n), b1 · (b2 · (m ⊗ n)) = (b1 b2 ) · (m ⊗ n)
y que 1 · (m ⊗ n) = m ⊗ n. La prueba para el caso del anillo C es similar y además
es claro que (b · (m ⊗ n)) · c = b · ((m ⊗ n) · c).
Con un razonamiento similar al anterior se puede construir el producto tensorial
f g
de homomorfismos de bimódulos: sean M −→ M 0 , N −→ N 0 , B − A y A − C-
f ⊗g
homomorfismos, respectivamente. Entonces, M ⊗ N −→ M 0 ⊗ N 0 definido por
(f ⊗ g) (m ⊗ n) := f (m) ⊗ g(n)
es un B − C-homomorfismo. Se tienen las siguientes propiedades:
(a) iM ⊗ iN = iM ⊗N .
(b) (f1 f2 ) ⊗ (g1 g2 ) = (f1 ⊗ g1 ) (f2 ⊗ g2 ).
(c) Si f, g son sobreyectivos, entonces f ⊗ g es sobreyectivo.
(d) Si f, g son sobreyectivos, entonces ker (f ⊗ g) = ker(f ) ⊗ N + M ⊗ ker(g).
(e) Si f y g son isomorfismos, entonces f ⊗ g es un isomorfismo y (f ⊗ g)−1 =
f −1 ⊗ g −1 .
(f) Los funtores M ⊗ − :A M od → Ab y − ⊗ N : M odA → Ab son covariantes,
aditivos y exactos a derecha (véase [19]): iM ⊗ (g1 + g2 ) = iM ⊗ g1 + iM ⊗
g2 , (f1 + f2 ) ⊗ iN = f1 ⊗ iN + f2 ⊗ iN . De manera más general, f ⊗ (g1 + g2 ) =
f ⊗ g1 + f ⊗ g2 y también (f1 + f2 ) ⊗ g = f1 ⊗ g + f2 ⊗ g.
g1 g2
Si N1 −→ N2 −→ N3 −→ 0 es exacta, entonces
i ⊗g1 i ⊗g2
M ⊗ N1 M
−→ M ⊗ N2 M
−→ M ⊗ N3 −→ 0 es exacta.

f1 f2
Si M1 −→ M2 −→ M3 −→ 0 es exacta, entonces
f1 ⊗i f2 ⊗i
M1 ⊗ N −→N M2 ⊗ N −→N M3 ⊗ N −→ 0 es exacta.
(g) M ⊗ − y − ⊗ N no son, en general, exactos a izquierda:
f j
0 −→ Z −→ Z −→ Z2 , f (k) := 2k, y j es el canónico. Entonces,
f ⊗iZ j⊗iZ
0 −→ Z ⊗ Z2 −→2 Z ⊗ Z2 −→2 Z2 ⊗ Z2 , es decir,
2 0 iZ
0 −→ Z2 −→ Z2 −→ Z2 no es exacta.
22 CAPÍTULO 1. ELEMENTOS BÁSICOS DE ÁLGEBRA HOMOLÓGICA

(h) B MA ⊗A

= B MA ,(B −A-isomorfismo). El isomorfismo viene dado por m⊗a 7→
m · a. Similarmente, A ⊗ A NC ∼= A NC (A − C-isomorfismo). El isomorfismo
se define por a ⊗ n 7→ a · n.

(i) (B MA ⊗A NC ) ⊗C LD ∼
= B MA ⊗ (A NC ⊗C LD ) (B − D-isomorfismo).

(j) Sea R un anillo conmutativo y sean M, N , R-módulos. Entonces, M ⊗ N ∼


=
N ⊗ M (R-isomorfismo).

(k) Sea {Mi }i∈C una familia de B − A-bimódulos y {Nj }j∈D una familia de A − C-
bimódulos. Entonces se tiene el B − C-isomorfismo

( M i ) ⊗ ( Nj ) ∼
L L L
= i,j (Mi ⊗ Nj ).
L L
Demostración. Paso 1. Sean M := Mi y N := Nj . Consideremos las
i∈C j∈D
inyecciones canónicas

µi : M i → M
µj : Nj → N

y su producto tensorial
N N
µi µj : M i Nj → M ⊗ N ;

entonces el siguiente diagrama es conmutativo

M ⊗N
3
 6

µi ⊗µj 
 β



 L N
M i ⊗ Nj - (Mi Nj )
µij (i,j)∈C×D

donde µij es la inyección canónica y β se define por


P N
β(z) := (µi µj ) (zij ) ,
(i,j)∈sop(z)

L N N
con z := (zij ) ∈ (Mi Mj ), zij ∈ Mi Mj y sop(z) el soporte de
(i,j)∈C×D
z, véase [15] para la noción de soporte de un elemento de una suma directa
externa. Para z = 0 se tiene que β(0) := 0.
1.4. PRODUCTO TENSORIAL 23

Paso 2. En el siguiente diagrama conmutativo t es la canónica del prodcuto


tensorial, α0 es bilineal y balancedada y α se define por la propiedad universal
del producto tensorial:
t
M ×N - M ⊗N
p
pp
pp
pp
α0
p pp α
? pp
+
L N
(Mi Nj )
(i,j)∈C×D

con α0 ((mi ) , (nN


N N N
j )) := (mi nj ) y α ((mi ) (nj )) := (mi nj ). Nótese que el
soporte de (mi nj ) es finito ya que está inculido en el producto cartesiano
de los soprtes de (mi ) y (nj ). Es fácil probar que α0 es bilineal y balanceada.
Además, α claramente es un B − C−homomorfismo.
Paso 3. Veamos que βα = iM ⊗N . Sea w ∈ N M ⊗ N , puesto que α es un
homomorfismo se puede asumir que w = (mi ) (nj ). Entonces,
N N
βα (w) = βα ((mi ) (nj )) = β ((mi nj ))
P N N
=  N 
(µi µj ) (mi nj )
(i,j)∈sop mi nj
P N
=  N µi (mi ) µj (nj )
(i,j)∈sop mi nj
P N P
= µi (mi ) µj (nj )
i∈sop(mi ) j∈sop(nj )
N
= (mi ) (nj ) = w.
L
Paso 4. αβ también es la idéntica: sea z = (zij ) ∈ (Mi ⊗Mj ), zij ∈
(i,j)∈I×J
Mi ⊗Mj . Si z = 0, entonces
P αβ(0) = 0. Sea z 6= 0, entonces z es una suma
finita de la forma z = µij (zij ); puesto que β y µij son homomorfismos
(i,j)∈sop(z)
se puede asumir que z = µij (mi ⊗nj ). Entonces,

αβ (µij (mi ⊗nj )) = α (µi (mi ) ⊗µj (nj ))


= α ((. . . , mi , . . . ) ⊗ (. . . , nj , . . . ))
= (. . . , mi ⊗nj , . . . )
= µij (mi ⊗nj ).
24 CAPÍTULO 1. ELEMENTOS BÁSICOS DE ÁLGEBRA HOMOLÓGICA

(l) Sea A → B un homomorfismo de anillos y X un conjunto no vacı́o. Entonces,


A(X) ⊗B ∼= B (X) (A − B − isomorfismo). En consecuencia, si MA es proyectivo,
entonces M ⊗A B es un B-módulo proyectivo.

(m) Se tiene el siguiente D − B-isomorfismo:

HomC (B MA ⊗A NC ,D LC ) ∼
= HomA (B MA , HomC (A NC ,D LC )) . (1.4.1)

Demostración. Definimos la aplicación

α : HomA (M, HomC (N, L)) → HomC (M ⊗A N, L) , f 7→ α(f ) := αf ,

αf : M ⊗A N → L, αf (m ⊗ n) := fm (n),
fm : N → L, fm := f (m), m ∈ M, f ∈ HomA (M, HomC (N, L)) .
En [14] se prueba en forma detallada que αf es un C-homomorfismo bien
definido y α es un D−B-homomorfismo. Sea f tal que α(f ) = 0, entonces para
cada m ∈ M y cada n ∈ N resulta fm (n) = 0, es decir, fm = 0 para cada m,
luego f = 0. Esto demuestra que α es inyectiva. Sea t ∈ HomC (M ⊗A N, L),
definimos f : M → HomC (N, L) por f (m) := fm , con fm : N → L dada por
fm (n) := t(m ⊗ n). Se puede demostrar que fm es un C-homomorfismo y que
f es un A-homomorfismo (véase [14]); además es claro que α(f ) = t, es decir,
α es sobreyectiva. Esto completa la demostración.

(n) Se tiene el siguiente C − D-isomorfismo:

HomB (B MA ⊗A NC ,B LD ) ∼
= HomA (A NC , HomB (B MA ,B LD )).

La prueba es similar a la del literal (l) (véase también [27], teorema 2.11).

(o) Sea P un A-módulo proyectivo y Q un A−B-bimódulo el cual es B-proyectivo.


Entonces, P ⊗ Q es B-proyectivo. En particular, si R es un anillo conmutativo
y P, Q son R-módulos proyectivos, entonces P ⊗ Q es R-proyectivo.

Demostración. Sea M → N → 0 una sucesión exacta de B-módulos derechos,


entonces se tiene la sucesión exacta de A-módulos derechos HomA (Q, M ) →
HomA (Q, N ) → 0, y como P es A-proyectivo resulta la sucesión exacta de
grupos abelianos HomA (P, HomB (Q, M )) → HomA (P, HomB (Q, N )) → 0.
Según (l), resulta la sucesión exacta de grupos abelianos HomB (P ⊗A Q, M ) →
HomB (P ⊗A Q, N )) → 0, es decir, P ⊗A Q es B-proyectivo.

(p) Sea L un B-módulo proyectivo f.g. Entonces se tiene el C-isomorfismo


1.4. PRODUCTO TENSORIAL 25

HomA (B MA ,C NA ) ⊗B LD ∼
= HomA (HomB (B LD ,B MA ),C NA ).

Demostración. Notemos en primer lugar que la función h0 : HomA (M, N ) ×


L → HomA (HomB (L, M ), N ) definida por

h0 (f, l) := h0(f,l) , con h0(f,l) : HomB (L, M ) → N , h0(f,l) (g) := f (g(l)),

donde f ∈ HomA (M, N ), g ∈ HomB (L, M ), cumple las siguientes condi-


ciones: h0(f,l) es un A-homomorfismo y h0 es bilineal y B-balanceada. En efecto,
h0(f,l) (g1 +g2 ) = f [(g1 +g2 )(l)] = f [g1 (l)+g2 (l)] = f (1 (l))+f (g2 (l)) = h0(f,l) (g1 )+
h0(f,l) (g2 ); h0(f,l) (g · a) = f [(g · a)(l)] = f [g(l) · a] = f [g(l)] · a = h0(f,l) (g) · a;
h0 (f1 + f2 , l) = h0(f1 +f2 ,l) , h0(f1 +f2 ,l) (g) = (f1 + f2 )(g(l)) = f1 (g(l)) + f2 (g(l)) =
h0(f1 ,l) (g) + h0(f2 ,l) (g), es decir, h0 (f1 + f2 , l) = h0 (f1 , l) + h0 (f2 , l); h0 (f, l1 + l2 ) =
h0(f,l1 +l2 ) , h0(f,l1 +l2 ) (g) = f [g(l1 +l2 )] = f (g(l1 ))+f (g(l2 )) = h0(f,l1 ) (g)+h0(f,l2 ) (g),
es decir, h0 (f, l1 + l2 ) = h0 (f, l1 ) + h0 (f, l2 ). Finalmente, h0 (f · b, l) = h0(f ·b,l) ,
h0(f ·b,l) (g) = (f · b)[g(l)] = f [b · g(l)] = f [g(b · l)] = h0(f,b·l) (g), es decir,
h0 (f · b, l) = h0 (f, b · l).
Por la propiedad del producto tensorial, h0 induce un homomorfismo de grupos
abelianos he0 : HomA (M, N ) ⊗ L → HomA (HomB (L, M ), N ) definido por
he0 (f ⊗ l) := h0(f,l) . Observemos que he0 es un C-homomorfismo. En efecto, he0 [c ·
(f ⊗ l)] = he0 [(c · f ) ⊗ l] = h0 , h0 (g) = (c · f )[g(l)] = c · f [g(l)] =
(c·f,l) (c·f,l)
c[·h0(f,l) (g)] = [c · h0(f,l) ](g) = [c · he0 (f ⊗ l)](g), es decir, he0 [c · (f ⊗ l)] = c · he0 (f ⊗ l).
Un caso particular es cuando L = B, en esta situación claramente he0 es un
isomorfismo; y en consecuencia, si L es un B-módulo libre de dimensión finita,
entonces he0 es también un isomorfismo.
Para concluir la prueba sea L proyectivo f.g. sobre B, entonces existe F libre
sobre B de dimensión finita y L0 un B-módulo tales que F = L ⊕ L0 . Se tiene
entonces un C-isomorfismo

h : HomA (M, N ) ⊗B (L ⊕ L0 ) → HomA (HomB (L ⊕ L0 , M ), N ),


e

pero

HomA (HomB (L ⊕ L0 , M ), N ) ∼
= HomA (HomB (L, M ) ⊕ HomB (L0 , M ), N ) ∼
=
−1 −1
he0 ⊕h 00
HomA (HomB (L, M ), N ) ⊕ HomA (HomB (L0 , M ), N ) −−−−−−→
f

(HomA (M, N ) ⊗B L) ⊕ (HomA (M, N ) ⊗B L0 ),

es decir, el isomorfismo e
h del caso libre F es suma directa de los homomorfismos
−1 −1
0
h yh
e e00 correspondientes a los módulos L y L0 , respectivamente; pero e
h es
26 CAPÍTULO 1. ELEMENTOS BÁSICOS DE ÁLGEBRA HOMOLÓGICA

−1 −1
un isomorfismo si, y sólo si, he0 y he00 son isomorfismos. Esto completa la
prueba.

(q) Producto tensorial de módulos libres: sean X, Y conjuntos no vacı́os, entonces


se tiene el A-isomorfismo

A(X) ⊗ A(Y ) ∼
= A(X×Y ) .

En particular, sea R un anillo conmutativo y sean M y N dos R-módulos libres


de dimensión finita n y m con bases X = {x1 , . . . , xn } y Y = {y1 , . . . , ym },
respectivamente. Entonces, X ⊗ Y := {xi ⊗ yj | 1 ≤ i ≤ n, 1 ≤ j ≤ m} es una
base de M ⊗ N , y en consecuencia, dimR (M ⊗ N ) = nm.

(r) Sea R un dominio de integridad y sean M, N, X, Y como en el literal anterior.


Para cualesquiera elementos m ∈ M y n ∈ N se tiene que

m ⊗ n = 0 ⇔ m = 0 ó n = 0.

Concluimos esta sección con un teorema clásico del álgebra conmutativa el cual
podemos probar en general para anillos locales no conmutativos.

Teorema 1.4.3 (Kaplansky). Sea A un anillo local. Entonces, cada A-módulo


proyectivo finitamente generado es libre de dimensión finita.

Demostración. Recordemos que un anillo A (no necesariamente conmutativo) es


local si, y sólo si, A/Rad(A) es un anillo de división, véase [18]). Sea M un A-
módulo proyectivo finitamente generado sobre un anillo local A. Veamos que M es
libre. Si M = 0 no hay nada que demostrar. Sea pues 0 6= M con sistema minimal de
generadores no nulos {x1 , . . . , xn }, entonces existe un módulo N tal que An = M ⊕N .
Sea P := Rad(A), entonces al tensorizar por A/P se tiene el A/P -isomorfismo
(A/P )n = (M ⊗A A/P ) ⊕ (N ⊗A A/P ).
Pero nótese que M ⊗ A/P ∼ = M/M P (isomorfismo de A/P -módulos) y además,
aplicando el lema de Nakayama, encontramos que {x1 , . . . , xn } es un sistema minimal
de generadores del A/P -módulo M/P M , pero como A/P es un anillo de división,
entonces este conjunto es linealmente independiente, es decir, es una base de M/M P .
De esto se obtiene que
(A/P )n = (M/P M ) ⊕ (N/P N ),
luego N/P N = 0 (nótese que los anillos de división son dimensionales, véase [17]),
de donde N = P N . Pero como P = Rad(A), entonces por el Lema de Nakayama se
tiene que N = 0. Esto implica entonces que An = M .
1.5. ANILLO DE FRACCIONES 27

1.5. Anillo de fracciones


En esta sección discutiremos la existencia, construcción y unicidad de los anillos de
fracciones en el caso general no conmutativo.

Definición 1.5.1. Sean A un anillo y S un subconjunto multiplicativo de A,


es decir, S es cerrado para el producto, 1 ∈ S y 0 ∈
/ S. Se dice que un anillo B
es un anillo de fracciones a derecha de A respecto de S si se cumplen las
siguientes condiciones:

(i) Existe un homomorfismo de anillos ψ : A → B tal que

(ii) ψ(S) ⊆ B ∗

(iii) ψ(a) = 0 ⇔ as = 0, para algún s ∈ S

(iv) Cada elemento de B se puede representar en la forma

ψ(a)ψ(s)−1 , con a ∈ A, s ∈ S.

A diferencia del caso conmutativo (véase [15]), la existencia de un anillo derecho


de fracciones está supeditada al cumplimiento de dos condiciones.

Teorema 1.5.2. Sean A un anillo y S sun subconjunto multiplicativo de A. En-


tonces, A posee un anillo de fracciones a derecha respecto de S si, y sólo si, S
satisface las siguientes condiciones:

(i) Si a ∈ A y s ∈ S son tales que sa = 0, entonces existe u ∈ S tal que au = 0.

(ii) Condición de Ore a derecha: dados a ∈ A y s ∈ S existen t ∈ S y b ∈ A


tales que at = sb.

Demostración. Notemos que en la condición de Ore si a ∈ S, es decir, si los dos


elementos dados a y s son de S, entonces at = sb ∈ S.
⇒) Sea B un anillo de fracciones a derecha de A respecto de S con función ψ que
cumple las condiciones (i)-(iv) de la definición 1.5.1. Si sa = 0 con s ∈ S y a ∈ A,
entonces ψ(s)ψ(a) = 0, de donde ψ(a) = 0. Existe pues u ∈ S tal que au = 0, y
hemos probado (i). De otra parte, sean a ∈ A y s ∈ S, entonces ψ(s)−1 ψ(a) ∈ B.
Existen b0 ∈ A y t0 ∈ S tales que ψ(s)−1 ψ(a) = ψ(b0 )ψ(t0 )−1 , luego ψ(sb0 ) = ψ(at0 )
y existe u ∈ S tal que (at0 − sb0 )u = 0, tomando t := t0 u y b := b0 u se obtiene (ii).
⇐) La idea ahora es construir un anillo de fracciones a derecha. Dividiremos la
demostración en varios pasos.
Paso 1. En el conjunto A × S definimos la relación ≡ de la siguiente manera:
(a, s) ≡ (b, t) si, y sólo si, existen c, d ∈ A tales que ac = bd y sc = td ∈ S.
28 CAPÍTULO 1. ELEMENTOS BÁSICOS DE ÁLGEBRA HOMOLÓGICA

Nótese que al aplicar la condición de Ore a los elementos s, t se encuentran los


elementos c, d ∈ A tales que sc = td ∈ S.
Probemos que la relación ≡ es de equivalencia: (a, s) ≡ (a, s) ya que a1 = a1 y
s1 = s1 ∈ S; la simetrı́a es consecuencia de la simetrı́a de la relación de igualdad; por
último, si (a, s) ≡ (b, t) y (b, t) ≡ (e, r), entonces existen elementos c, d, c1 , d1 ∈ A
tales que
ac = bd, sc = td ∈ S, bc1 = ed1 , tc1 = rd1 ∈ S.
Aplicamos la condición de Ore a los elementos sc y rd1 , con lo cual existen x, y ∈ A
tales que scx = rd1 y ∈ S. Resulta
acx = bdx, scx = tdx, bc1 y = ed1 y, tc1 y = rd1 y,
luego tdx = tc1 y y entonces t(dx − c1 y) = 0. Por la condición (i) existe u ∈ S tal que
(dx − c1 y)u = 0, es decir, dxu = c1 yu, luego bdxu = bc1 yu, de donde acxu = ed1 yu
y también scxu = rd1 yu ∈ S. Esto prueba que (a, s) ≡ (e, r).
Paso 2. Denotemos por as la clase de equivalencia que contiene al par (a, s) y
mediante AS −1 al conjunto de todas las clases ası́ determinadas. Sean as , bt ∈ AS −1 ,
definimos
a b ac+bd
s
+ t
:= u
,
donde u ∈ S es el elemento determinado por la condición de Ore aplicada a los
elementos s, t ∈ S: u := sc = td. Se puede demostrar que la suma anterior está bi-
en definida, es decir, no depende de u ni de los representantes elegidos para las
fracciones, la prueba completa se puede consultar en [14]. El producto se define por
ab ac
st
:= tu
,
donde c ∈ A y u ∈ S están definidos por la condición de Ore aplicada a los elementos
b y s: bu = sc. En [14] se demuestra en forma detallada que el producto anterior
está bien definido.
Paso 3. Las operaciones anteriores dotan a AS −1 de una estructura de anillo. La
prueba completa de esta afirmación se puede consultar en [14]. Notemos que 10 = 0s ,
con s ∈ S, es el cero de este anillo; 11 = ss , s ∈ S, es el uno; y que el opuesto de as
es −as
. En efecto, veamos primero que 01 = 0s y 11 = ss : 0s = 01, 1s = s1; 1s = s1,
1s = s1. Ahora, as + 01 = a1+0ss
= as ya que s1 = 1s; as 11 = a1
1s
= as ya que 1s = s1,
1a
1s
= 1as1
= as ya que a1 = 1a; as + −a
s
= a1+(−a)1
s
= 0s ya que s1 = s1.
Paso 4. AS −1 es un anillo de fracciones a derecha de A respecto de S. En efecto,
definimos
ψ a
A−→ AS −1 , a 7→ . (1.5.1)
1
ψ es un homomorfismo de anillos ya que ψ(a + b) = a+b 1
= a1 + 1b = ψ(a) + ψ(b),
ψ(ab) = ab 1
= a1 1b = ψ(a)ψ(b), ψ(1) = 11 ; sea s ∈ S, entonces ψ(s) = 1s y 1s 1s = 11 ,
es decir, ψ(S) ⊆ (AS −1 )∗ ; sea a ∈ S tal que ψ(a) = 01 , entonces a1 = 01 y existen
1.5. ANILLO DE FRACCIONES 29

c, d ∈ A tales que ac = 0d = 0 y 1c = 1d = c ∈ S; de otra parte, si a ∈ A y u ∈ S


son tales que au = 0, entonces 0 = ψ(au) = ψ(a)ψ(u), de donde ψ(a) = 0; por
último, es claro que dado as ∈ AS −1 , as = a1 1s = ψ(a)ψ(s)−1 .
Pasamos ahora a considerar la unicidad del anillo de fracciones a derecha, en
caso de que este último exista.

Proposición 1.5.3 (Propiedad universal). Sean A un anillo y S un subconjunto


multiplicativo de A tales que AS −1 existe, es decir, las dos condiciones del teorema
1.5.2 se cumplen. Sea g : A → A0 un homomorfismo de anillos tal que g(S) ⊆
A∗0 . Entonces, existe un único homomorfismo h : AS −1 → A0 tal que el siguiente
diagrama conmuta:
ψ
A -
AS −1 p
p pp
g
p pp
pp
h
?
A0
h es definido por h( as ) := g(a)g(s)−1 . Además, si g es inyectivo, entonces h es
también inyectivo.

Demostración. Veamos que h está bien definido, el resto de la prueba se puede


consultar en [14]. Sea as = bt , entonces existen c, d ∈ A de tal forma que ac = bd y
sc = td ∈ S, resulta g(a)g(c) = g(b)g(d) y g(sc) = g(td) ∈ A∗0 , de donde
g(a)g(c) = g(a)g(s)−1 g(s)g(c) = g(a)g(s)−1 g(sc) = g(a)g(s)−1 g(td);
g(b)g(d) = g(b)g(t)−1 g(t)g(d) = g(b)g(t)−1 g(td),
con lo cual g(a)g(s)−1 = g(b)g(t)−1 .

Corolario 1.5.4. Sea A un anillo que posee anillo de fracciones a derecha respecto
del sistema multiplicativo S. Si A0 es un anillo con homomorfismo g : A → A0 tal
que g(S) ⊆ A∗0 y A0 también cumple la propiedad universal, entonces A0 ∼
= AS −1 .
Demostración. Basta aplicar dos veces la propiedad universal tanto a A0 como a
AS −1 para construir homomorfismos h : AS −1 → A0 y h0 : A0 → AS −1 tales que
sus compuestas dan las aplicaciones idénticas.

Teorema 1.5.5. Si A0 es un anillo con homomorfismo g : A → A0 que satisface


las condiciones (i)-(iv) de la definición 1.5.1, entonces A0 es un anillo derecho de
fracciones de A respecto de S y A0 ∼ = AS −1 .
Demostración. Según la definición 1.5.1, A0 es un anillo derecho de fracciones de
A respecto de S; S entonces satisface las dos condiciones del teorema 1.5.2 y por
lo tanto construimos el anillo AS −1 . Según la proposición 1.5.3, existe un único
30 CAPÍTULO 1. ELEMENTOS BÁSICOS DE ÁLGEBRA HOMOLÓGICA

homomorfismo de anillos h que cumple hψ = g. Resta ver que h es un isomorfismo:


si h( as ) = 0, entonces g(a) = 0 y existe u ∈ S tal que au = 0, es decir, as = 01 , esto
muestra que h es inyectivo. Sea x ∈ A0 , entonces x se puede representar en la forma
x = g(a)g(s)−1 , a ∈ A, s ∈ S. Entonces, x = h( as ) y h es pues sobreyectivo.

Resulta conveniente preguntar bajo qué condición un anillo A se puede sumergir


en AS −1 (en caso de que este último exista). La respuesta a esta pregunta y su
prueba son como en el caso conmutativo (véase [15]).

Proposición 1.5.6. Sean A un anillo, S un subconjunto multiplicativo de A y B


un anillo de fracciones a derecha de A respecto de S. Entonces,

(i) ψ es inyectivo si, y sólo si, S no posee divisores de cero a derecha.

(ii) ψ es biyectivo si, y sólo si, S ⊆ A∗ .

Demostración. (i) Evidente a partir de la condición (iii) en la definición 1.5.1.


(ii) ⇒): Sea s ∈ S, entonces existe a ∈ A tal que ψ(a) = ψ(s)−1 , luego 1 = ψ(as)
y de esta forma 1 = as; también, puesto que 1 = ψ(a)ψ(s) y ψ(s) es invertible,
entonces ψ(s)ψ(a) = 1, con lo cual sa = 1, es decir, s ∈ A∗ .
⇐): Según la hipotesis y la condición (ii) de la definición 1.5.1, ψ es resulta
claramente inyectivo; sea b ∈ B, según la condición (iv) de la definición 1.5.1, b =
ψ(a)(s)−1 , con a ∈ A y s ∈ S, por lo tanto, b = ψ(as−1 ) y de esta manera ψ es
sobreyectivo.

Proposición 1.5.7. Sea A un anillo que posee anillo de fracciones a derecha res-
pecto del sistema multiplicativo S. Si A se puede sumergir en AS −1 , es decir, si ψ
es inyectivo, entonces AS −1 es el menor anillo que contiene a A en el cual todos los
elementos de S son invertibles.

Demostración. Consecuencia directa de la propiedad universal.

Observación 1.5.8. (i) Es claro a partir de las definiciones y propiedades vistas


anteriormente cómo definir y caracterizar un anillo de fracciones a izquierda.
Además, todas las propiedades estudiadas en la presente sección son también válidas
por el lado izquierdo (veánse por ejemplo las proposiciones 1.5.12, 1.5.13 y 1.5.14
más adelante)
Se tiene la siguiente propiedad interesante: sean A un anillo y S un subconjunto
multiplicativo de A tales que AS −1 y S −1 A existen. Entonces, AS −1 ∼ = S −1 A. En
efecto, sean ψ : A → AS −1 y ψ 0 : A → S −1 A los homomorfismos canónicos, entonces
según la proposición 1.5.3 (y la correspondiente afirmación por el lado izquierdo)
existen homomorfismos h : AS −1 → S −1 A y h0 : S −1 A → AS −1 tales que hψ = ψ 0
y h0 ψ 0 = ψ, es decir, (hh0 )ψ 0 = ψ 0 , (h0 h)ψ = ψ, por unicidad en la proposición
1.5. ANILLO DE FRACCIONES 31

mencionada (y su correspondiente afirmación a izquierda), hh0 = iS −1 A y h0 h =


iAS −1 .
(ii) Queremos ahora mostrar que la construcción del anillo AS −1 generaliza el ca-
so conmutativo: en primer lugar las condiciones (i) y (ii) del teorema 1.5.2 se cumplen
trivialmente en el caso de un anillo conmutativo R y un subconjunto multiplicativo
S. Consideremos también la relación de equivalencia que define las fracciones de
RS −1 , es decir, (a, s) ∼ (b, t) si, y sólo si, existe u ∈ S tal que atu = bsu, luego
si hacemos c := tu y d := su se obtiene que (a, s) ≡ (b, t). Recı́procamente, si
(a, s) ≡ (b, t), entonces existen c, d ∈ R tales que ac = bd y sc = td ∈ S, luego
at(dt) = at(sc) = acts = bdts = bs(dt), con dt ∈ S, es decir, (a, s) ≡ (b, t). Con
respecto a las operaciones tenemos lo siguiente: puesto que st = ts ∈ S, entonces
a
s
+ bt = at+bs
st
, lo cual corresponde al caso conmutativo. Análogamente, como bs = sb,
entonces as bt = abts
= ab
st
.
Un caso particular muy importante de localización conmutativa es cuando S :=
R − P , con P un ideal primo de R; en esta situación el anillo de fracciones se denota
por RP y es un anillo local (véase [15]).
Ejemplo 1.5.9. Anillo total de fracciones. Sean A un anillo y
S0 := {a ∈ A|a no es divisor de cero}.
Notemos que la condición (i) del teorema 1.5.2 en este caso se cumple trivialmente.
Si se satisface la condición de Ore a derecha, entonces el anillo de fracciones AS0−1
existe y se denomina el anillo total de fracciones a derecha de A (también se
le conoce como el anillo clásico de fracciones a derecha de A o anillo de
Goldie a derecha de A). Se acostumbra a denotar por Qr (A). Ası́ pues, si Qr (A)
existe, entonces A se puede sumergir en Qr (A) y este último es el menor anillo que
contiene a A en el cual todos los elementos de S0 son invertibles. En forma análoga
se define el anillo total de fracciones a izquierda de A y se denota por Ql (A).
Si tanto Qr (A) como Ql (A) existen, entonces Qr (A) ∼= Ql (A) y este anillo se denota
simplemente por Q(A), y se le conoce como el anillo total de fracciones de A.
Ejemplo 1.5.10. Sea A un dominio, es decir, un anillo sin divisores de cero,
entonces S0 = A − {0}. Si S0 satisface la condición de Ore a derecha, entonces se
dice que A es un dominio de Ore a derecha, y en tal caso, Qr (A) es un anillo de
división, denominado el anillo de división a derecha de A: sea as 6= 0 en Qr (A),
entonces a 6= 0 y por lo tanto a ∈ S0 . Se tiene entonces que as as = aa = 11 = ss = as as ,
es decir, as es invertible en Qr (A). De manera análoga se definen los dominios de
Ore a izquierda y el anillo de división a izquierda de A. Notemos que si
A = R es un dominio de integridad (DI), entonces Q(R) es un cuerpo, el cuerpo
de fracciones de R (véase también [15]).
El ejemplo 1.5.10 plantea la siguiente pregunta: ¿bajo qué condición un dominio
satisface la condición de Ore a derecha? Comencemos con el siguiente hecho trivial.
32 CAPÍTULO 1. ELEMENTOS BÁSICOS DE ÁLGEBRA HOMOLÓGICA

Proposición 1.5.11. Sea A un anillo. A es un dominio de Ore a derecha si, y sólo


si, A se puede sumergir en un anillo de división Q tal que cada elemento z de Q se
puede escribir en la forma z = as−1 , con a ∈ A y s ∈ A − {0}.

Demostración. ⇒): basta tomar Q := Qr (A).


⇐): como A se sumerge en Q entonces A es un dominio y se tiene el homomorfis-
mo inclusión de anillos ψ : A → Q, el cual claramente cumple las cuatro condiciones
de la definición 1.5.1 con especto al sistema S0 := A − {0}. Entonces Q es un anillo
de fracciones de A respecto S0 y el teorema 1.5.2 garantiza que A es un dominio de
Ore a derecha.

Proposición 1.5.12. Sea A un dominio noetheriano a derecha. Entonces, A es un


dominio de Ore a derecha, y por tanto, el anillo de división a derecha Qr (A) existe.

Demostración. Sean a ∈ A y s 6= 0, debemos encontrar t 6= 0 y b ∈ A tales


que at = sb. Si a = 0, podemos tomar t = 1 y b = 0. Sea entonces a 6= 0; si
aA ∩ sA 6= 0, entonces hemos terminado ya que entonces existen t, b ∈ A tales que
at = sb 6= 0, con lo cual necesariamente t 6= P 0 (también b 6= 0). Vamos a asumir
que aA ∩ sA = 0, lo cual implica que la suma k≥0 sk aA es directa. En efecto, sea
aa0 + saa1 + · · · + sn aan = 0, con ai ∈ A, 0 ≤ i ≤ n. Entonces aa0 ∈ sA, de donde
a0 = 0 y también aa1 + · · · + sn−1 aan = 0. Nuevamente a1 = 0, y podemos continuar
de la misma forma encontrando que ai = 0 para cada i.
Resulta entonces la cadena ascendente infinita de ideales derechos

aA ( aA ⊕ saA ( aA ⊕ saA ⊕ s2 aA ( · · · ,
lo cual es una contradicción ya que A es noetheriano a derecha.

Otra propiedad relacionada con la condición de Noether y que permite garantizar


la existencia del anillo de fracciones es la siguiente.

Proposición 1.5.13. Sean A un anillo noetheriano a derecha, S un subconjunto


multiplicativo de A que satisface la condición de Ore a derecha. Entonces, S satisface
la condición (i) del teorema 1.5.2, y en consecuencia, AS −1 existe.

Demostración. Sean s ∈ S y a ∈ A tales que sa = 0; consideremos la cadena


ascendente de ideales derechos Ik := {x ∈ A|sk x = 0}, k ≥ 1. Existe entonces n ≥ 1
tal que In = In+i , con i ≥ 0. Aplicamos la condición de Ore a derecha a los elementos
a ∈ A y sn ∈ S y encontramos elementos t ∈ S y b ∈ A tales que at = sn b, luego
sat = 0 = sn+1 b, es decir, b ∈ In+1 = In , de donde sn b = 0, es decir, at = 0.

Veamos que la condición de Noether pasa del anillo A al anillo de fracciones


(véase [1]).
1.6. MÓDULO DE FRACCIONES 33

Proposición 1.5.14. Sean A un anillo y S un subconjunto multiplicativo de A


tales que AS −1 existe. Si A es noetheriano a derecha, entonces AS −1 es noetheriano
a derecha.

Demostración. Veamos inicialmente que los ideales derechos de AS −1 son de la forma


IS −1 := { as |a ∈ I, s ∈ S}, donde I es un ideal derecho de A (para esta parte de
la prueba no es necesario que A sea noetheriano a derecha). En efecto, es claro
que si I es un ideal derecho de A, entonces IS −1 es un ideal derecho de AS −1 ;
recı́procamente, si J es un ideal derecho de AS −1 entonces el conjunto I definido
por I := {a ∈ A| a1 ∈ J} es un ideal derecho de A que satisface IS −1 = J. Sea
I = {a1 , . . . , at i, entonces es fácil mostrar que J = { a11 , · · · , a1t i.

1.6. Módulo de fracciones


Vamos ahora a estudiar los módulos de fracciones en el caso general no conmutativo
que nos ocupa. Sea A un anillo y sea S un subconjunto multiplicativo de A tal que
AS −1 existe, sea M un A-módulo a derecha, entonces el módulo de fracciones a
derecha de M respecto de S puede ser definido y construido en forma análoga a
como vimos para los anillos. Veamos las ideas generales, los detalles de las pruebas
los dejamos al lector. En M × S definimos la relación ≈ por

(m, s) ≈ (n, t) si, y sólo si, existen c, d ∈ A tales que m · c = n · d y sc = td ∈ S.

Se puede verificar que ≈ es una relación de equivalencia, la clase de (m, s) se denota


por ms y el conjunto de todas las clases por M S −1 . Este último tiene estructura de
AS −1 -módulo a derecha respecto de las siguientes operaciones:
m n m·c+n·d
s
+ t
:= u
, con u := sc = td ∈ S

m a m·c
s
· t
:= tu
, con au = sc, c ∈ A, u ∈ S.

El homomorfismo canónico ψ : A → AS −1 dota a M S −1 de estructura natural de


A-módulo derecho y la función canónica
φ
M− → M S −1
m
m 7→ .
1
es un homomorfismo de A-módulos derechos con núcleo

ker(φ) = {m ∈ M |m · u = 0, para algún u ∈ S}.


34 CAPÍTULO 1. ELEMENTOS BÁSICOS DE ÁLGEBRA HOMOLÓGICA

Nótese que ms = 0 si, y sólo si, m ∈ ker(φ): en efecto, si ms = 01 , entonces ms · 1s = 01 1s ,


es decir, m1 = 01 , luego existen c, d ∈ A tales que m · c = 0 · d y 1c = 1d ∈ S. Resulta,
m · c = 0, con c ∈ S, es decir, m ∈ ker(f ). La afirmación recı́proca se prueba en
forma análoga.
Proposición 1.6.1 (Propiedad universal). Sea A un anillo que posee anillo de
fracciones a derecha respecto del sistema multiplicativo S. Para cada AS −1 -módulo a
derecha N y cada A-homomorfismo g : M → N existe un único AS −1 -homomorfismo
h : M S −1 → N tal que el siguiente diagrama conmuta:
φ
M -
Mp S −1
p pp
p
g
p p ph
pp
?
N
h es definido por h( ms ) := g(m) · 1s . Además, si g es inyectivo, entonces h es también
inyectivo.
Demostración. La demostración es un sencillo ejercicio que se deja al lector.
Corolario 1.6.2. Sea A un anillo que posee anillo de fracciones a derecha res-
pecto del sistema multiplicativo S. Si N es un AS −1 -módulo a derecha con A-
homomorfismo g : M → N que también cumple la propiedad universal, entonces
N∼ = M S −1 .
Demostración. La demostración es como la del corolario 1.5.4.
A partir de la proposición 1.6.1 y de la propiedad universal del producto tensorial
se puede demostrar el AS −1 -isomorfismo de módulos derechos
M S −1 ∼= M ⊗A AS −1 , (1.6.1)
donde la estructura natural de A-módulo a izquierda de AS −1 está dada por ψ.
−1
Observemos que cada elemento x ∈ M ⊗A ASP se puede representar en la forma
k
m ⊗ s , con m ∈ M y s ∈ S. En efecto, si x = i=1 mi ⊗ asii , entonces
1

k
X 1
x= m i · ai ⊗
i=1
si
s1 1 s1 1 s1 1 s1
x· = m 1 · a1 ⊗ + m 2 · a2 ⊗ + · · · + m k · ak ⊗
1 s1 1 s2 1 sk 1
0 0
1 a a
= m1 · a1 ⊗ + m2 · a2 ⊗ 02 + · · · + mk · ak ⊗ 0k
1 s2 sk
1 1 1
= m1 · a1 ⊗ + m2 · a002 ⊗ 0 + · · · + mk · a00k ⊗ 0 ,
1 s2 sk
1.6. MÓDULO DE FRACCIONES 35

si continuamos de esta manera encontramos s ∈ S y m ∈ M tales que x · 1s = m ⊗ 11 ,


es decir, x = m ⊗ 1s . El isomorfismo (1.6.1) identifica precisamente ms con m ⊗ 1s .
Otras propiedades relacionadas con los módulos de fracciones son las siguientes.

Proposición 1.6.3. Sea A un anillo que posee anillo de fracciones a derecha res-
pecto del sistema multiplicativo S. Entonces,

(i) Si la siguiente sucesión de A-módulos derechos es exacta

f1 f2
0 −→ M1 −→ M2 −→ M3 −→ 0,

entonces la sucesión de AS −1 -módulos derechos


f1 ⊗i f2 ⊗i
0 −→ M1 S −1 −→ M2 S −1 −→ M3 S −1 −→ 0,

es también exacta, con i := iAS −1 .

(ii) Sea M un A-módulo a derecha y N un submódulo de M . Entonces se tiene el


AS −1 -isomorfismo (M/N )S −1 ∼
= M S −1 /N S −1 .

(iii) Si M es un AS −1 -módulo a derecha, entonces M ∼


= M S −1 .

(iv) Sea M un A-módulo a derecha. Entonces, M es un AS −1 -módulo a derecha con


un pruducto ∗ tal que la estructura de A-módulo inducida por el homomorfismo
canónico A → AS −1 coincide con la estructura inicial de A-módulo de A si,
y sólo si, M es sin torsión respecto de S, es decir, ker(φ) = 0 y M es
S-divisible, es decir, para cada s ∈ S se tiene que M = M · s.

(v) Sea M un A-módulo derecho. M es de torsión respecto de S si, y sólo si,


M S −1 = 0.

(vi) Sean M y N AS −1 -módulos a derecha y sea f : M → N un A-homomorfismo,


entonces f es un AS −1 -homomorfismo.

Demostración. (i) Teniendo en cuenta el isomorfismo (1.6.1) y que el producto tenso-


f
rial preserva exactitud a derecha, basta probar que si M − → N es un monomorfismo,
−1 f ⊗i −1
entonces M ⊗ AS −−→ N ⊗ N S es también un monomorfismo, con i = iAS −1 .
Cada elemento x de M ⊗ AS −1 es de la forma x = m ⊗ 1s , con m ∈ M y s ∈ S;
supongamos que (f ⊗ i)(x) = 0, es decir, f (m) ⊗ 1s = 0, entonces f (m)
s
= 0 en N S −1 ,
luego existe t ∈ S tal que f (m) · t = 0, para algún t ∈ S, es decir, f (m · t) = 0, de
donde m · t = 0. Esto quiere decir que ms = 0 y ası́ m ⊗ 1s = 0.
(ii) es consecuencia directa de (i).
36 CAPÍTULO 1. ELEMENTOS BÁSICOS DE ÁLGEBRA HOMOLÓGICA

(iii) Si M es un AS −1 -módulo con producto ∗, entonces M adquiere estructura


de A-módulo mediante el homomorfismo canónico A → AS −1 dada por m ◦ a :=
m ∗ a1 . Con esta estructura construimos el AS −1 -módulo M S −1 . Tenemos entonces
el siguiente diagrama conmutativo
φ
MA -
Mp S −1
p
p pp
p
g
p pp h
pp
?
MAS −1

donde φ es el A-homomorfismo canónico y g se define por g(m) := m, es decir, g es


la función idéntica. Nótese que g es un A-homomorfismo : g es claramente adidtivo,
g(m ◦ a) = g(m ∗ a1 ) = m ∗ a1 = g(m) ∗ a. La propiedad universal de M S −1 garantiza
la existencia del AS −1 -homomorfismo h dado por h( ms ) := g(m) ∗ 1s = m ∗ 1s . Veamos
que h es biyectivo: si h( ms ) = 0, entonces m ∗ 1s = 0, luego (m ∗ 1s ) ∗ 1s = 0, luego
0 = m ∗ ( 1s 1s ) = m ∗ 11 = m, de donde ms = 01 ; además, m = m ∗ 11 = h( m1 ).
(iv) ⇒): sea · el producto de la estructura dada de A-módulo que tiene M ;
sea ∗ el producto que dota a M de una estructura de AS −1 -módulo, entonces por
hipótesis, el producto ◦ inducido por esta última estructura conincide con ·, es decir,
m · a = m ◦ a := m ∗ a1 . Sean m ∈ M y s ∈ S tales que m · s = 0, entonces m ∗ 1s = 0,
luego (m ∗ 1s ) ∗ 1s = 0, de donde m = 0. Además, dados m ∈ M y u ∈ S, se tiene que
m = (m ∗ u1 ) ∗ u1 = (m ∗ u1 ) · u, es decir, M ⊆ M · u, luego M = M · u.
⇐): sean m ∈ M y as ∈ AS −1 , debemos definir un producto m ∗ as . Puesto que
M es S-divisible, existe m0 ∈ M tal que m · a = m0 · s, luego definimos m ∗ as := m0 .
Seam m0 , m00 ∈ M tales que m · a = m00 · s, por tanto (m0 − m00 ) · s = 0, de donde
m0 = m00 ya que M sin torsión respecto de S. Es fácil probar que ∗ está bien definido
y dota a M de estructura de AS −1 -módulo a derecha.
(v) Evidente.
(vi) Sea f (z ∗ as ) := y, entonces f (z ∗ as ) ∗ 1s = y ∗ 1s , luego f (z ◦ a) = y ∗ 1s , es
decir, f (z) ◦ a = y ∗ 1s , con lo cual f (z) ∗ as = y.

Corolario 1.6.4. Sea A un anillo que posee anillo de fracciones a derecha respec-
to del sistema multiplicativo S. Entonces, S −1 : M odA → M odAS −1 es un funtor
covariante, aditivo y exacto.

Demostración. Los detalles de la prueba los dejamos al lector.

Observación 1.6.5. El módulo de fracciones a izquierda de M respecto de


S se define y construye en forma completamente análoga a como acabamos de ver,
y se denota por S −1 M .
1.6. MÓDULO DE FRACCIONES 37

Observación 1.6.6. En forma similar a como anotamos en la observación 1.5.8, los


módulos de fracciones sobre anillos conmutativos siempre existen y su estructura se
puede simplificar: en efecto, la relación ≈ en este caso toma la forma
(m, s) ≈ (n, t) si, y sólo si, existe u ∈ S tale que m · tu = n · su, para algún u ∈ S.
Las operaciones se definen por
m n m·t+n·s m a m·a
s
+ t
:= st
, s · t
:= st
.
En el caso particular en que S := R − P , con P un primo de R, el módulo localizado
se denota por MP .
Incluimos a continuación otras propiedades adicionales de los módulos de frac-
ciones pero sobre anillos conmutativos.
Proposición 1.6.7. Sean A un anillo, R un anillo conmutativo contenido en A
y S un subconjunto multiplicativo de R tal que AS −1 existe. Entonces se tiene el
isomorfismo de anillos y de A-módulos izquierdos
A ⊗R RS −1 ∼
= AS −1 .
Demostración. A ⊗R RS −1 como anillo corresponde al producto tensorial de las
R-álgebras A y RS −1 (véase [17]). La función g : A → A ⊗R RS −1 definida por
g(a) := a ⊗ 1 es un homomorfismo de anillos y satisface g(S) ⊆ (A ⊗R RS −1 )∗ .
La proposición 1.5.3 induce un homomorfismo de anillos h : AS −1 → A ⊗R RS −1
dado por h( as ) := g(a)g(s)−1 = (a ⊗ 1)(1 ⊗ 1s ) = a ⊗ 1s . Por otro lado, la función
f 0 : A × RS −1 → AS −1 definida por f 0 (a, rs ) := ar s
es bilineal y R-balanceada, luego
existe un homomorfismo de grupos abelianos f : A ⊗R RS −1 → AS −1 definido por
f (a ⊗ rs ) := ars
. Nótese que hf = iA⊗RS −1 y f h = iAS −1 , luego h es un isomomorfismo
de anillos.
Veamos que además h es un homomorfismo de A-módulos izquierdos: h(a0 · as ) =
0 0
h( a1 as ) = h( asa ) = a0 a ⊗ 1s = a0 · (a ⊗ 1s ) = a0 · h( as ).
Proposición 1.6.8. Sean R un anillo conmutativo, S un sistema multiplicativo de
R y M, N módulos sobre R. Entonces,
(M ⊗R N ) S −1 ∼
= M S −1 ⊗RS −1 N S −1 .
Demostración. En el siguiente diagrama conmutativo
t - M S −1 ⊗ −1 N S −1
M S −1 × N S −1 RS
p p ppp
p
ppp
h0 pp ppp
p p ppp h
? p

(M ⊗R N )S −1
38 CAPÍTULO 1. ELEMENTOS BÁSICOS DE ÁLGEBRA HOMOLÓGICA

t( ms , nr ) := ms ⊗ nr , h0 ( ms , nr ) := m⊗n
sr
, h( ms ⊗ nr ) := m⊗n
sr
, t y h0 son bilineales y RS −1 -
balanceadas, luego por la propiedad universal del producto tensorial, h queda bien
definida y es un RS −1 -homomorfismo.
De otra parte, se tiene el siguiente diagrama conmutativo
t0 - M ⊗R N
M ×N
pp
pp
p pp
pp
g 00
p p p g0
pp
pp
? +
M S −1 ⊗RS −1 N S −1

con t0 (m, n) := m ⊗ n, g 00 (m, n) := m1 ⊗ n1 , g 0 (m ⊗ n) := m1 ⊗ n1 , t0 y g 00 bilineales y R-


balanceadas, luego g 0 (m ⊗ n) := m1 ⊗ n1 está bien definida y es un R-homomorfismo.
Según la proposición 1.6.1, existe un RS −1 -homomorfismo g : (M ⊗R N )S −1 →
M S −1 ⊗RS −1 N S −1 definido por g( zs ) := g 0 (z) · 1s , con z ∈ M ⊗R N . Se puede probar
fácilmente que hg = i(M ⊗N )S −1 , gh = iM S −1 ⊗N S −1 .

Sea q una propiedad de un anilo R (o de un R-módulo M ). Se dice que q es


una propiedad local-global para R (para M ) si se cumple la siguiente condición:
R (respectivamente M ) tiene la propiedad q si, y sólo si, para todo primo P , RP
(respectivamente MP ) tiene la propiedad q.

Proposición 1.6.9. Sean R un anillo conmutativo y M un R-módulo. Entonces las


siguientes condiciones son equivalentes:

(i) M = 0.

(ii) MP = 0 para cada ideal primo P .

(iii) MP = 0 para cada ideal maximal de P .

Demostración. (i)⇒(i)⇒(iii) son evidentes.


(iii)⇒(i): supongamos que M 6= 0, entonces existe 0 6= m ∈ M y Ann(x) 6= R,
por lo tanto, existe un ideal maximal P de R tal que Ann(x) ⊆ P . Pero como
MP = 0, entonces m1 = 01 , con lo cual existe s ∈
/ P tal que m · s = 0, pero esto es
contradictorio ya que s ∈ Ann(x).

Proposición 1.6.10. Sean R un anillo conmutativo y f : M → N un homomorfis-


mo de R-módulos. Entonces las siguientes condiciones son equivalentes:

(i) f es inyectivo (sobreyectivo, biyectivo).

(ii) fP es inyectivo (sobreyectivo, biyectivo) para cada primo P .


1.6. MÓDULO DE FRACCIONES 39

(iii) fP es inyectivo (sobreyectivo, biyectivo) para cada ideal maximal P .

Demostración. La prueba de estas afirmaciones es un sencillo ejercicio que resulta


de aplicar la proposición anterior.

Proposición 1.6.11. Sea M un R-módulo proyectivo finitamente generado. En-


tonces, para cada ideal primo P de R, MP es un RP -módulo libre de dimensión
finita.

Demostración. Según la propiedad (k) de la sección 1.4, MP es un RP -módulo


proyectivo f.g., luego por el teorema de Kaplansky (teorema 1.4.3), MP es RP -libre
de dimensión finita.

Sean R un anillo conmutativo y M un R-módulo proyectivo finitamente generado,


se dice que M es de rango constante n si dimRP (MP ) = n, para cada ideal primo
P de R. Se escribe entonces rankP (M ) = n.
Vimos en la demostración del teorema de Kaplansky que M/P M es un espacio
vectorial sobre el cuerpo R/P , donde P es un maximal de R. Si M es proyectivo de
rango constante n, entonces se tiene la siguiente relación entre n y la dimensión del
espacio M/P M .

Proposición 1.6.12. Sean R un anillo conmutativo, P un ideal maximal de R y


M un R-módulo. Entonces,

(i) MP /P RP MP es un R/P -espacio vectorial.

(ii) M/P M ∼
= MP /P RP MP (R/P -isomorfismo).

(iii) Si M es proyectivo de rango constante n, entonces dimR/P (M/P M ) = n.

Demostración. (i) Recordemos que la estructura de R/P -espacio para M/P M viene
dada por m · r := m · r. De manera similar, MP /P RP MP es un espacio vectorial
sobre el cuerpo RP /P RP con el producto ms · ar := m·a
sr
. Sin embargo, se tiene el

siguiente isomorfismo de cuerpos R/P = RP /P RP dado por a 7→ a1 . Esto convierte
a MP /P RP MP en un R/P -espacio con el producto dado por ms · a := ms · a1 := m·a
s
.
(ii) Se tiene el R/P -isomomorfismo
M/P M ∼
= MP /P RP MP , m 7→ m
1
.
(iii) Puesto que M es proyectivo de rango constante n respecto de R, entonces
MP es libre de dimensión n respecto de RP ; nótese que si {z1 , . . . , zn } es una base
de MP respecto de RP , entonces {z1 , . . . , zn } es una base de MP /P RP MP respec-
to de RP /P RP , luego n = dimRP /P RP (MP /P RP MP ) = dimR/P (MP /P RP MP ) =
dimR/P (M/P M ).
40 CAPÍTULO 1. ELEMENTOS BÁSICOS DE ÁLGEBRA HOMOLÓGICA

Proposición 1.6.13. Sean R un anillo conmutativo y M y N R-módulos proyec-


tivos de rango constante m y n, respectivamente. Entonces,
L
(i) rankP (M N ) = rankP (M ) + rankP (N ).

(ii) rankP (M ⊗ N ) = rankP (M ) rankP (N ).

Demostración. Estas afirmaciones son evidentes si se consideran las localizaciones


de una suma directa y el producto tensorial.

1.7. Módulos inyectivos


Esta sección es en cierto sentido dual de la sección 1.2. Recordemos que un A-módulo
N es inyectivo si HomA (−, N ) preserva la exactitud a derecha.

Teorema 1.7.1. Sea N un A-módulo. Entonces las siguientes condiciones son equi-
valentes:

(i) N es inyectivo.

(ii) Para cada homomorfismo inyectivo f y cada homomorfismo g, existe un ho-


momorfismo h tal que el siguiente diagrama conmuta:

N
p
6Ip p
g p hp
pp
p
M f
- L

f
(iii) Cada homomorfismo inyectivo N −→ M es hendido.

Demostración. (i)⇒(ii): se tiene la sucesión exacta


f∗
HomA (L, N ) −→ HomA (M, N ) → 0,
es decir, f ∗ es sobreyectivo y (ii) está probado.
(ii)⇒(i): (ii) significa que f ∗ es sobreyectivo, con lo cual N es inyectivo.
(ii)⇒(iii): se tiene el diagrama conmutativo

N
p
6Ip p
iN p hp
pp
p
N f
- M
1.7. MÓDULOS INYECTIVOS 41

luego hf = iN , es decir, f es hendido.


(iii)⇒(ii): consideremos los homomorfismos

N
6
g

M f
- L

con f inyectivo; sea J := (N ⊕ L)/K, donde K := {(g(m), −f (m))|m ∈ M }.


Definimos la función u : N → J, u(n) := (n, 0); notemos que u es un homomorfismo
inyectivo, luego por hipótesis existe un homomorfismo v : J → N tal que vu = iN .
Sea entonces h := vw, con w : L → J definido por w(l) := (0, l). Resulta, hf = g. En
efecto, hf (m) = vwf (m) = v[(0, f (m))] = v[(g(m), 0)] = vu(g(m)) = iN (g(m)) =
g(m).

Proposición 1.7.2. Cada sumando directo de un módulo inyectivo es inyectivo.

Demostración. Sea N un módulo inyectivo y sea N 0 un sumando directo de N ,


N = N 0 ⊕ N 00 ; consideremos los homomorfismos

N 0 ⊕ N 00
p
6Kp p
π µ pp
pp
? pp
N0 p ph
pp
6 p pp
g pp
pp
M f
- L

donde f es inyectivo, π es la proyección canónica y µ es la inyección canónica;


tenemos que hf = µg, luego (πh)f = πµg = g. Esto demuestra que N 0 es inyectivo.

Q
Proposición 1.7.3. Sea {Ni }i∈C una familia de A-módulos. Entonces, i∈C Ni es
inyectivo si, y sólo si, cada Ni es inyectivo.
42 CAPÍTULO 1. ELEMENTOS BÁSICOS DE ÁLGEBRA HOMOLÓGICA

Q
Demostración. ⇒): Puesto Ni es inyectivo se tiene el diagrama conmutativo

Y
Ni
p
6Kp p
µi pp
pp
pp
Ni p pt
pp
6 p pp
g pp
pp
M f
- L

con f inyectivo. Considerando la proyección canónica πi obtenemos un homomorfis-


mo πi t tal que πi tf = g: en efecto, tf = µi g, de donde πi tf = πi µi g = g.
⇐): Consideremos los homomorfismos

Ni
p
6Kp p
πi pp
pp
p p
Ni p p pti
Y
p
p pp
6 Ip p t p p
g p p pp
p p pp
M f
- L

donde f es inyectivo y πi es la proyección canónica; tenemos que ti f = πi g y Q


πi t = ti ,
luego πi (tf ) = πi g para cada i, de donde tf = g. Esto demuestra que Ni es
inyectivo.

Teorema 1.7.4 (Teorema de Baer). Un A-módulo N es inyectivo si, y sólo si,


para cada ideal derecho I de A, cada A-homomorfismo g : I → N se puede extender
hasta A.

Demostración. ⇒): sea I un ideal derecho de A; consideremos el diagrama

N
6
g

I ι
- A
1.7. MÓDULOS INYECTIVOS 43

por el teorema 1.7.1, existe h : A → N tal que el siguiente diagrama es conmutativo

N
p
6Ip p h
g pp
pp
I ι
- A

⇐): supongamos ahora que tenemos el siguiente diagrama

N
6
g

M f
- L

donde f es inyectivo; la idea es definir un homomorfismo h : L → N tal que hf = g.


Sea S la colección de parejas de la forma (Lα , gα ), donde Im(f ) ≤ Lα ≤ L y
gα : Lα → N es un homomorfismo tal que gα f = g. S no es vacı́o ya que f (M ) ∼ = M,
−1 −1
es decir, existe f : f (M ) → M tal que f (M ) ≤ L y (gf )f = g; notemos
también que (S, ≥) es parcialmente ordenado, donde (Lα , gα ) ≥ (Lβ , gβ ) si, y sólo si,
Lα ≥ Lβ y gαS|Lβ = gβ . Consideremos en S un subconjunto S 0 totalmente ordenado
y sea L0 := Lα ∈S 0 Lα ; notemos que Im(f ) ≤ L0 ≤ L, además, sea h0 : L0 → N ,
h0 (l0 ) := gα (l0 ) si l0 ∈ Lα . Notemos que l0 es un homomorfismo y satisface h0 f = g,
(L0 , h0 ) ≥ (Lα , gα ) para cada (Lα , gα ) ∈ S 0 ; por el Lema de Zorn existe una pareja
maximal (L0 , g0 ) ∈ S de tal forma que Im(f ) ≤ L0 ≤ L y g0 : L0 → N es un
homomorfismo que satisface g0 f = g.
Supongamos que L0 6= L, veamos que esto conduce a una contradicción. Sea
x ∈ L, x ∈ / L0 , definimos I := {a ∈ A|x · a ∈ L0 }, I es un ideal derecho de A y la
función ϕ : I → N definida por ϕ(a) := g0 (x · a) es un homomorfismo; por hipótesis
ϕ se puede extender a un homomorfimso ϕ0 : A → N ; sea ϕ0 (1) := n0 ∈ N . Notemos
que si a ∈ I, entonces ϕ(a) = g0 (x · a) = ϕ0 (a) = ϕ0 (1) · a = n0 · a, es decir,
g0 (x · a) = n0 · a, para cada a ∈ I.
Sea L00 := L0 + {xi; notemos que Im(f ) ≤ L00 ≤ L, además, la aplicación h00 : L00 →
N definida por h00 (l0 +x·a) := g0 (l0 )+n0 ·a, l0 ∈ L0 , a ∈ A, es un homomorfismo bien
definido: en efecto, si l0 +x·a = l00 +x·a0 , entonces l0 −l00 = x·a0 −x·a = x·(a0 −a) ∈ L0
y podemos aplicar g0 de tal forma que g0 (l0 ) − g0 (l00 ) = n0 · (a0 − a) ya que a0 − a ∈ I,
es decir, g0 (l0 ) + n0 · a = g0 (l00 ) + n0 · a0 . El homomorfismo h00 satisface h00 f = g ya que
si z ∈ M , entonces f (z) ∈ L0 , de donde h00 (f (z)) = h00 (f (z) + 0) = g0 (f (z)) = g(z).
Hemos entonces encontrado (L00 , h00 ) ∈ S tal que L0 L00 y h00 |L0 = g0 , luego
(L0 , g0 ) (L00 , h00 ), pero esto contradice la escogencia de (L0 , g0 ).
44 CAPÍTULO 1. ELEMENTOS BÁSICOS DE ÁLGEBRA HOMOLÓGICA

Corolario 1.7.5. Sean A un anillo y N un A-módulo. Las siguientes condiciones


son equivalentes:

(i) N es inyectivo.

(ii) Cada sucesión exacta 0 → N → M → L → 0 es hendida.

(iii) Cada sucesión exacta 0 → N → M → L → 0, con L cı́clico, es hendida.

Demostración. . (i)⇒(ii): Evidente.


(ii)⇒(iii): Evidente.
(iii)⇒(i): Sea I un ideal derecho de A y sea g : I → N un A-homomorfismo.
Entonces el siguiente diagrama es conmutativo, con filas exactas:
ι j
0 −−−→ I −−−→ A −−−→ A/I −−−→ 0
  
g p i
y y y A/I
q h
0 −−−→ N −−−→ S −−−→ A/I −−−→ 0

donde S es la suma fibrada de g y ι, es decir, S := (N ⊕ A)/K, con K :=


{(g(x), −ι(x))|x ∈ I}, q(n) := (n, 0), p(a) := (0, a), h((n, a)) := a. Por la hipótesis,
existe t : S → N tal que tq = iN . Se obtiene entonces el homomorfismo u := tp
de A en N el cual satisface uι = tpι = tqg = g. Según el teorema de Baer, N es
inyectivo.

Corolario 1.7.6. Si D es un dominio de Ore a derecha, entonces Qr (D) es un


D-módulo derecho inyectivo.

Demostración. Es claro que ψ : D → Qr (D) dota a Qr (D) de estructura de D-


módulo a derecha. Según el teorema de Baer, basta demostrar que para cada ideal
derecho I de D, cada D-homomorfismo f : I → Qr (D) se puede extender a D.
Sean a, b dos elementos no nulos de I, por la condición de Ore a derecha existen
t, u ∈ D−{0} tales que at = bu, luego f (a)·t = f (b)·u, con lo cual f (a) 1t = f (b) u1 , de
donde f (a) = f (b) u1 1t = f (b) ut . Resulta, f (a) a1 = f (b) ut a1 = f (b) at
u u
= f (b) bu = f (b) 1b .
Ası́, sea c := f (a) a1 = f (b) 1b ∈ Qr (D) este elemento común asociado a I. Definimos
fe : D → Qr (D) por fe(d) := c d1 , es claro que fe es un D-homomorfismo y para cada
a ∈ I no nulo se tiene que fe(a) = c a1 = f (a) a1 a1 = f (a); es claro que fe(0) = f (0).
Ası́ pues, fe extiende f .

Otra consecuencia interesante del teorema de Baer es el siguiente resultado.

Corolario 1.7.7. Sea A un anillo noetheriano a derecha. Entonces la suma directa


de A-módulos inyectivos es un módulo inyectivo.
1.7. MÓDULOS INYECTIVOS 45

Demostración. Sean {Ni }i∈C una familia de A-módulos inyectivos Le I := {a1 , . . . an i


un ideal derecho de A; consideremos un homomorfismo f : I → i∈C Ni ; cada f (ai )
tiene soporte finito, luego f (a1 ), . . . , f (an ) involucran solamente un subconjunto
finito de C, digamos {i1 , . . . , im }, entonces Im(f ) ⊆ N1 ⊕ · · · ⊕ Nm , pero como
esta última suma es un módulo inyectivo, entonces se tiene el siguiente diagrama
conmutativo:
M
Ni
i∈C

6
ι0

N1 ⊕ · · · ⊕ Nm
p
6Ip p
pp g
f0 pp
pp
pp
I ι
- A
con f 0 (x) := f (x), x ∈ I. Puesto que gι = f 0 , entonces (ι0 g)ι = ι0 f 0 = f .

Proposición 1.7.8. Sea A un anillo. Entonces las siguientes condiciones son equi-
valentes:

(i) A es semisimple.

(ii) Cada ideal derecho de A es inyectivo.

(iii) Cada izquierdo de A es inyectivo.


ι
Demostración. (i)⇒(ii): Sea I un ideal derecho de A y sea I −
→ M un homomorfismo
inyectivo de A-módulos derechos. Como A es semisimple, entonces M es semisimple
(véase [18]), luego ι(I) es sumando directo de M , es decir, ι es hendido. Esto dice
que I es inyectivo.
(ii)⇒(i): Nuevamente sea I un ideal derecho de A, entoncs el homomorfismo
ι
inclusión I −
→ A es hendido, lo cual implica que I es sumando directo de A. Esto
establece que A es semisimple.
La prueba de (i)⇔(iii) es similar.
Una propiedad homológica muy importante relacionada con los módulos inyec-
tivos dice que todo A-módulo se puede sumergir en un módulo inyectivo. Para la
demostración necesitamos algunos preliminares.

Definición 1.7.9. Sea M un A-módulo; M es divisible si para cada elemento


a ∈ A que no sea divisor de cero se cumple que M · a = M .
46 CAPÍTULO 1. ELEMENTOS BÁSICOS DE ÁLGEBRA HOMOLÓGICA

Algunas propiedades elementales de los módulos divisibles se presentan a con-


tinuación.

Proposición 1.7.10. Sea A un anillo. Entonces

(i) Si M es un A-módulo divisible y N es un submódulo de M , entonces M/N es


divisible.

(ii) Cada sumando directo de un módulo divisible es divisible.

(iii) El producto y la suma directa externa de módulos divisibles es divisible.

Demostración. Todas las afirmaciones se prueban fácilmente a partir de la definición


de módulo divisible.

Proposición 1.7.11. Todo módulo inyectivo es divisible. Para dominios de ideales


principales derechos se cumple la afirmación recı́proca.

Demostración. Sean M un A-módulo inyectivo, m ∈ M y a ∈ A que no es divisor


de cero; la función f : {ai → M , f (ar) := m · r es un homomorfismo bien definido.
En efecto, si ar = as, entonces a(r − s) = 0, luego r = s con lo cual m · r = m · s.
Por el toerema de Baer, f se puede extender a un homomorfismo f 0 : A → M ; en
particular, f 0 (a) = f (a) = f (a1) = m · 1 = m = f 0 (1a) = f 0 (1) · a = m0 · a, es decir,
M es divisible por a.
Sea D un un dominio de ideales principales derechos y sea I un ideal derecho
de D, consideremos un homomorfismo f : I → M ; sea I = {ai. Si a = 0, entonces
I = 0 y f = 0, luego en este caso f se extiende a todo D. Sea pues a 6= 0;
f (a) ∈ M , entonces existe m0 ∈ M tal que f (a) = m0 · a; definimos f 0 : D → M ,
f 0 (r) := m0 · r. Claramente f 0 es un homomorfismo; si b ∈ I entonces b = ac, luego
f 0 (b) = m0 · b = m0 · ac = (m0 · a) · c = f (a) · c = f (ac) = f (b), es decir, f 0 extiende
a f.

Ejemplo 1.7.12. No todo módulo libre es inyectivo. En efecto, ZZ es libre pero no


es inyectivo ya que no es divisible.

Corolario 1.7.13. Sean D un dominio de Ore a derecha y Qr (D) su anillo de


división, entonces

(i) Qr (D) es un D-módulo derecho divisible.

(ii) Cada Qr (D)-espacio vectorial derecho es un D-módulo divisible.

Demostración. (i) Consecuencia directa de las proposiciones 1.7.6 y 1.7.11.


(ii) Resulta de (i) y de la proposción 1.7.10 ya que un espacio vectorial sobre
Qr (D) es suma directa externa de copias de Qr (D).
1.7. MÓDULOS INYECTIVOS 47

Proposición 1.7.14. Cada grupo abeliano se puede sumergir en un grupo abeliano


inyectivo (= divisible).

Demostración. Sea G un grupo abeliano, entonces G es de la forma G = F/S, donde


F := Z(G) es un grupo abeliano libre y S es un subgrupo de F ; sabemos que Z(G) =
⊕g∈G Zg , con Zg = Z para cada g. Consideremos la inclusión canónica Zg ,→ Q,
luego ⊕g∈G Zg ,→ Q(G) y S es un subgrupo de Q(G) , con lo cual G ,→ Q(G) /S, pero
Q(G) /S es Z-divisible, y por lo tanto, Q(G) /S es inyectivo.

Proposición 1.7.15. Sean A un anillo y G un grupo abeliano inyectivo (=divisible).


Entonces HomZ (A, G) es un A-módulo derecho inyectivo.

Demostración. HomZ (A AZ ,Z GZ ) es un Z − A-bimódulo con producto (h · x)(a) :=


f
h(xa), donde h ∈ HomZ (A AZ ,Z GZ ), x, a ∈ A. Sea M − → L un A-homomorfismo
inyectivo de módulos derechos, resulta el Z-homomorfismo de módulos derechos
f∗
HomA (Z LA , HomZ (A AZ ,Z GZ )) −→ HomA (Z MA , HomZ (A AZ ,Z GZ )); se tiene el dia-
grama
f∗
HomA (Z LA , HomZ (A AZ ,Z GZ )) −−−→ HomA (Z MA , HomZ (A AZ ,Z GZ ))
 
θ θ
y1 y2
HomZ (Z LA ⊗A A,Z GZ ) HomZ (Z MA ⊗A A,Z GZ )
 
α1 α2
y y
f0
HomZ (Z LA ,Z GZ ) −−−→ HomZ (Z MA ,Z GZ )

donde las flechas verticales son isomorfismos (véase (1.4.1)) y f 0 es sobreyectiva ya


que G es inyectivo (proposición 1.7.11). Si el diagrama resulta conmutativo, entonces
f ∗ es sobreyectivo y la prueba habrı́a terminado. Veamos cómo están definidos los
homomorfismos del diagrama anterior:
f ∗ (h) = hf , con h : L → HomZ (A, G), h(l) := hl : A → G;
θ1 (h) = θ1h , con θ1h : L ⊗ A → G, θ1h (l ⊗ 1) := hl (1);
θ2 (g) = θ2g , con θ2g : M ⊗ A → G, θ2g (m ⊗ 1) := gm (1), donde
g : M → HomZ (A, G), g(m) := gm : A → G;
α1 (t) = α1t : L → G, con α1t (l) := t(l ⊗ 1), donde t : L ⊗ A → G;
α2 (s) = α2s : M → G, con α2s (m) := s(m ⊗ 1), donde s : M ⊗ A → G;
f 0 (u) := uf , con u : L → G.
Con las definiciones anteriores tenemos que
h h
(f 0 α1 θ1 )(h) = f 0 α1 (θ1h ) = f 0 [(α1 )θ1 ] = (α1 )θ1 f ,
luego para m ∈ M resulta
48 CAPÍTULO 1. ELEMENTOS BÁSICOS DE ÁLGEBRA HOMOLÓGICA

h
(α1 )θ1 f (m) = θ1h (f (m) ⊗ 1) = hf (m) (1);
por otro lado
hf
(α2 θ2 f ∗ )(h) = α2 θ2 (hf ) = α2 [θ2hf ] = (α2 )θ2 ,
luego para m ∈ M se tiene que
hf
(α2 )θ2 (m) = θ2hf (m ⊗ 1) = (hf )m (1), pero (hf )m = (hf )(m) = h(f (m)) = hf (m) ,
luego (hf )m (1) = hf (m) (1).

Teorema 1.7.16. Todo módulo se puede sumergir en un módulo inyectivo.

Demostración. Sea M un A-módulo a derecha; consideremos a M como Z-módulo a


derecha, entonces existe un grupo abeliano inyectivo G tal que M se puede sumergir
l
en G, M − → G, y por la proposición 1.7.11, G es Z-divisible. Según la proposición
1.7.15, HomZ (A AZ ,Z GZ ) es un A-módulo derecho inyectivo; se tiene la aplicación
f
M−
→ HomZ (A AZ ,Z GZ ), m 7→ fm , con fm : A → G, fm (a) := l(m · a).
fm es obviamente un homomorfismo de grupos abelianos; veamos que f es un A-
homomorfismo de módulos derechos: claramente f (m + m0 ) = f (m) + f (m0 ), además
para x ∈ A se tiene que f (m · x) = fm·x y para a ∈ A resulta fm·x (a) = l((m · x) ·
a) = l(m · (xa)), pero [(fm ) · x](a) = fm (xa) = l(m · (xa)), y esto demuestra que
f (m · x) = f (m) · x.
Por último, probemos que f es inyectivo: si f (m) = 0, entonces fm = 0, luego
fm (1) = 0 = l(m), por lo tanto, m = 0.

Mostramos a continuación otra caracterización de los módulos proyectivos a


través de módulos inyectivos, y de manera dual, de los módulos inyectivos a través
de módulos proyectivos.

Teorema 1.7.17. Sean P y Q módulos. Entonces,

(i) P es proyectivo si, y sólo si, para cada homomorfismo sobreyectivo f , con
Q inyectivo, y cada homomorfismo g, existe un homomorfismo h tal que el
siguiente diagrama conmuta:

P
pp
h p p
p g
pp
?
Q - Q0
f
1.7. MÓDULOS INYECTIVOS 49

(ii) Q es inyectivo si, y sólo si, para cada homomorfismo inyectivo f , con P proyec-
tivo, y cada homomorfismo g, existe un homomorfismo h tal que el siguiente
diagrama conmuta:
Q
p
6 Ip p h
g pp
pp
0 - P
P f

Demostración. Veamos la prueba de (i), la segunda parte queda como ejercicio al


lector.
⇒): esta parte es siempre cierta sin la condición de inyectividad sobre Q.
⇐): consideremos los homomorfismos

?
M - M 00
t

con t sobreyectivo; M se puede sumergir en un módulo inyectivo Q, s : M → Q, se


tiene entonces el siguiente diagrama conmutativo con filas exactas y Q00 := coker(sι),
donde M 0 := ker(t), ι es la inclusión, j es el homomorfismo canónico y q se define
por q(m00 ) := js(m), con m00 := t(m):
P

p
y
ι t
0 −−−→ M 0 −−−→ M −−−→ M 00 −−−→ 0
  
i 0 s q
yM y y
sι j
0 −−−→ M 0 −−−→ Q −−−→ Q00 −−−→ 0
Por hipótesis existe un homomorfismo h : P → Q tal que jh = qp. Veamos que
Im(h) ⊆ Im(s): en efecto, sea x ∈ P , entonces jh(x) = qp(x) = j(s(m)), con
p(x) = t(m), luego h(x)−s(m) ∈ ker(j) = Im(sι), de donde h(x)−s(m) = (sι)(m0 ),
es decir, h(x) = s(m − m0 ). Se tiene entonces el siguiente diagrama conmutativo, lo
cual prueba que P es proyectivo:

p
P
p pp
s h p
−1
p p
p pp
?
M t
- M 00
50 CAPÍTULO 1. ELEMENTOS BÁSICOS DE ÁLGEBRA HOMOLÓGICA

En efecto, notemos en primer lugar que q es inyectivo (si q(m00 ) = 0 = js(m),


con t(m) = m00 , entonces s(m) ∈ ker(j), luego s(m) = sι(m0 ) = s(m0 ), de donde
m = m0 , pero como m0 ∈ M 0 = ker(t), entonces t(m) = t(m0 ) = 0 = m00 ). Sea
ahora x ∈ P , entonces h(x) = s(m), con m ∈ M , de donde m = s−1 h(x) y resulta
t(m) = t(s−1 h(x)) := m00 . Se tiene pues que q(m00 ) = js(m), es decir, q[t(s−1 h(x))] =
js[s−1 h(x)] = jh(x) = qp(x), pero como q es inyectivo, entonces t(s−1 h(x)) =
p(x).

Concluimos con la versión inyectiva de la propiedad (o) de la sección 1.4.

Observación 1.7.18. Sea L un B − D-bimódulo el cual es finitamente presentado


sobre B y sea N un C − A-bimódulo el cual es inyectivo sobre A. Entonces se tiene
el siguiente C − D-isomorfismo:

HomA (B MA ,C NA ) ⊗B LD ∼
= HomA (HomB (B LD ,B MA ),C NA ).

La demostración la dejamos al lector. Véase [27], lema 3.60.

1.8. Módulos planos


Hemos visto que − ⊗A N y N ⊗A − son funtores exactos a derecha pero no siempre
son exactos a izquierda. Se tiene entonces la siguiente nueva clase de módulos.

Definición 1.8.1. Un A-módulo N es plano si N ⊗A − preserva exactitud a izquier-


da, es decir, para cada sucesión exacta de A-módulos izquierdos 0 → M1 → M2 →
M3 → 0, la sucesión de grupos abelianos 0 → N ⊗A M1 → N ⊗A M2 → N ⊗A M3 → 0
es también exacta.

De manera similar se define la noción de A-módulo izquierdo plano.

Proposición 1.8.2. Sea A un anillo que posee anillo de fracciones a derecha respec-
to del sistema multiplicativo S. Entonces, AS −1 es un A-módulo izquierdo plano. En
particular, si D es un dominio de Ore a derecha, entonces Qr (D) es un D-módulo
izquierdo plano.

Demostración. Esto es una consecuencia directa de la proposición 1.6.3.

En forma análoga, si S −1 A existe, entonces S −1 A es un A-módulo derecho plano.

Teorema 1.8.3. Sea N un A-módulo. Entonces las siguientes condiciones son equi-
valentes:

(i) N es plano.
1.8. MÓDULOS PLANOS 51

(ii) Si f : M1 −→ M2 es inyectivo, entonces iN ⊗ f : N ⊗A M1 → N ⊗A M2 es


inyectivo.

(iii) Si f : M1 −→ M2 es inyectivo y M1 , M2 son finitamente generados, entonces


iN ⊗ f es inyectivo.

Demostración. (i)⇒(ii): sea f : M1 → M2 un homomorfismo inyectivo, y considere-


f j
mos la sucesión exacta 0 → M1 −
→ M2 −
→ M2 /f (M1 ) → 0. Por hipótesis, la sucesión
i ⊗f
N N i ⊗j
0 → N ⊗ M1 −− −→ N ⊗ M2 −− −→ N ⊗ (M2 /f (M1 )) → 0 es exacta, por lo tanto
iN ⊗ f es un homomorfismo inyectivo.
(ii)⇒(i): teniendo en cuenta la hipótesis y que N ⊗− es siempre exacto a derecha,
entonces esta implicación es evidente.
(ii)⇒(iii): esta implicación es obvia.
f
(iii)⇒(ii): sea M1 →
− M2 un homomorfismo inyectivo y sea u := n1 ⊗m1 +· · ·+nt ⊗
mt ∈ ker(iN ⊗ f ), con mi ∈ M1 , ni ∈ N, 1 ≤ i ≤ t. Consideremos los submódulos
f0
M10 := hm1 , . . . , mt } ≤ M1 y M20 := hf (m1 ), . . . , f (mt )} ≤ M2 ; sea M10 −
→ M20
0 0 0 0 0 0
definido por f (m ) := f (m ), con m ∈ M1 . Es claro que f es también inyectivo,
i ⊗f 0
entonces por la hipótesis se tiene que N ⊗ M10 −N−−→ N ⊗ M20 es inyectivo, por lo
tanto, (iN ⊗ f 0 ) (u) = (iN ⊗ f 0 ) (n1 ⊗ m1 + · · · + nt ⊗ mt ) = n1 ⊗ f 0 (m1 ) + · · · + nt ⊗
f 0 (mt ) = n1 ⊗ f (m1 ) + · · · + nt ⊗ f (mt ) = (iN ⊗ f ) (n1 ⊗ m1 + · · · + nt ⊗ mt ) = 0,
luego u = n1 ⊗ m1 + · · · + nt ⊗ mt = 0.
α
Proposición 1.8.4. Sea A −
→ B un homomorfismo de anillos y N un A-módulo
plano. Entonces N ⊗A B es un B-módulo plano.
f
Demostración. Si M1 − → M2 es un homomorfismo inyectivo de módulos a izquierda
sobre B, entonces se tiene el siguiente diagrama conmutativo, donde las flechas
verticales son isomorfismos y iN ⊗ f es por hipótesis inyectivo:

A
i(N ⊗ B) ⊗f
(N ⊗A B) ⊗B M1 −−−− −−→ (N ⊗A B) ⊗B M2
↓ ↓
N ⊗A (B ⊗B M1 ) N ⊗A (B ⊗B M2 )
↓ ↓
i ⊗f
N
N ⊗A M1 −− −→ N ⊗A M2
Se tiene entonces que i(N ⊗A B) ⊗f es inyectivo, y por lo tanto, N ⊗A B es B-plano.

Proposición 1.8.5. Sea {Ni }i∈C una familia de A-módulos. Entonces,


L
i∈C Ni es plano ⇐⇒ ∀i ∈ C , Ni es plano.
52 CAPÍTULO 1. ELEMENTOS BÁSICOS DE ÁLGEBRA HOMOLÓGICA

f
L ⇒): sea M1 −
Demostración. →LM2 un homomorfismo inyectivo, entonces el homo-
morfismo ( i∈C Ni )⊗M1 → ( i∈C Ni )⊗M2 es inyectivo. Entonces el homomorfismo
L
L i∈C (iNi ⊗f ) L
i∈C (N i ⊗ M 1 −−−−−−−
) − → i∈C (Ni ⊗ M2 ) es inyectivo, y en consecuencia cada ho-
iN ⊗f
momorfismo Ni ⊗ M1 −−i−→ Ni ⊗ M2 es inyectivo.
f
⇐): sea M1 − → M2 un homomorfismo inyectivo, entonces para cada i ∈ C,
iN ⊗f
el homomorfismoLNi ⊗ M1 −−i−→ Ni ⊗ M2 es inyectivo, luego el homomorfismo
L i∈C (iNi ⊗f ) L
i∈C (N i ⊗LM 1 ) −
− −−−−−−→ L i∈C (Ni ⊗ M2 ) es inyectivo, y por lo tanto, el homo-
morfismo ( i∈C Ni ) ⊗ M1 → ( i∈C Ni ) ⊗ M2 es inyectivo.

Corolario 1.8.6. Todo módulo libre es plano.

Demostración. Según el teorema 1.8.3 AA es plano, luego, de acuerdo con la propo-


sición anterior, el módulo libre A(X) es plano.

Corolario 1.8.7. Todo módulo proyectivo es plano.

Demostración. Consecuencia directa de la proposición 1.8.5.

Concluimos esta introducción a la teorı́a de módulos planos mostrando otras


propiedades para el caso particular sobre anillos conmutativos. La primera dice que
la condición “ser plano”es local-global (véase la parte final de la sección 1.5)

Proposición 1.8.8. Sean R un anillo conmutativo y N un R-módulo. Entonces las


siguientes condiciones son equivalentes:

(i) N es plano.

(ii) NP es RP -plano para cada ideal primo P .

(iii) Np es RP -plano para cada ideal maximal P .

Demostración. (i)⇒(ii): consecuencia directa de la proposición 1.8.4.


(ii)⇒(iii): evidente.
f
(iii)⇒(i): sea M1 −→ M2 un R-homomomorfismo inyectivo, entonces por la
fp
proposición 1.8.2, (M1 )p −→ (M2 )p es un RP -homomorfismo inyectivo, y por la
fp ⊗iN
hipótesis, (M1 )p ⊗ Np −→P (M2 )p ⊗ Np es un RP -homomorfismo inyectivo. Pero
de acuerdo con la proposición 1.6.8, se tiene el siguiente diagrama conmutativo:
fP ⊗iN
P
(M1 )P ⊗ NP −−−−−→ (M2 )P ⊗ NP
h1 ↓ ↓ h2
g
(M1 ⊗ N )P −
→ (M2 ⊗ N )P
1.8. MÓDULOS PLANOS 53

donde h1 , h2 son isomorfismos definidos por h1 ( ms11 ⊗ nr11 ) := ms11⊗n


r1
1
, h2 ( ms22 ⊗ nr22 ) :=
m2 ⊗n2
s2 r2
, y g por construcción es definido como g := h2 (fP ⊗iNP )h1 −1 . Nótese entonces
que g es un homomorfismo inyectivo y coincide con (f ⊗ iN )P para cada maximal
P . Según la proposición 1.6.10, f ⊗ iN es inyectivo, luego N es plano.

La versión plana de la propiedad (o) de la sección 1.4 es el siguiente teorema.

Proposición 1.8.9. Sean R un anillo conmutativo, L un R-módulo plano, M un R-


módulo de presentación finita y N un R-módulo. Entonces se tiene el R-isomorfismo

HomR (M, N ) ⊗R L ∼
= HomR (M, N ⊗R L).

Demostración. El siguiente diagrama es conmutativo


φ
HomR (M, N ) × L - HomR (M, N ) ⊗R L
ppp
p p ppp
ppp
ppp
ppp β
α
p p
? ppp

HomR (M, N ⊗R L)

donde φ(h, l) := h ⊗ l, α(h, l) := αh,l , αh,l (m) := h(m) ⊗ l es bilineal y R-balanceada


y β definida por β(h ⊗ l) := αh,l es un R-homomorfismo. Notemos que si M = Rr
es libre de dimensión finita, entonces β es un R-isomorfismo. En efecto,
HomR (Rr , N ) ⊗R L ∼
= HomR (R, N )r ⊗R L ∼
= N r ⊗R L ∼
= (N ⊗R L)r ,
donde este isomorfismo compuesto viene definido por
γ(h ⊗ l) := (h(e1 ) ⊗ l, . . . , h(er ) ⊗ l),
con h ∈ HomR (Rr , N ) y {ei }ri=1 la base canónica de Rr ; también,
HomR (Rr , N ⊗R L) ∼
= HomR (R, N ⊗R L)r ∼
= (N ⊗R L)r ,
donde este isomorfismo compuesto viene definido por
δ(g) := (g(e1 ), . . . , g(er )).
con g ∈ HomR (Rr , N ⊗R L). Pero nótese que δ −1 γ = β: en efecto, δβ(h ⊗ l) =
δ(αh,l ) = (αh,l (e1 ), . . . , αh,l (er )) = (h(e1 ) ⊗ l, . . . , h(er ) ⊗ l) = γ(h ⊗ l).
p q
Sea Rr − → Rs − → M → 0 una presentación finita de M , se tiene entonces la
q∗ p∗
sucesión exacta 0 → HomR (M, N ) − → HomR (Rs , N ) − → HomR (Rr , N ), pero como
L es plano resulta la sucesión exacta
L q ∗ ⊗i L p∗ ⊗i
0 → HomR (M, N ) ⊗R L −−−→ HomR (Rs , N ) ⊗R L −−−→ HomR (Rr , N ) ⊗R L
54 CAPÍTULO 1. ELEMENTOS BÁSICOS DE ÁLGEBRA HOMOLÓGICA

y se tiene el siguiente diagrama conmutativo con filas exactas:


q ∗ ⊗iL p∗ ⊗iL
0 −−−−→ HomR (M, N ) ⊗R L −−−−→ HomR (Rs , N ) ⊗R L −−−−→ HomR (Rr , N ) ⊗R L
  
β0 y β 00 y

βy
 

q0 p0
0 −−−−→ HomR (M, N ⊗R L) −−−−→ HomR (Rs , N ⊗ L) −−−−→ HomR (Rr , N ⊗R L)

β 0 (q ∗ ⊗ iL )(h ⊗ l) = β 0 (hq ⊗ l) = αhq,l , αhq,l (ei ) = hq(ei ) ⊗ l;


q 0 β(h ⊗ l) = q 0 (αh,l ) = αh,l q, αh,l q(ei ) = h(q(ei )) ⊗ l;
β 00 (p∗ ⊗ iL )(u ⊗ l) = β 00 (up ⊗ l) = αup,l , αup,l (ei ) = up(ei ) ⊗ l;
p0 β 0 (u ⊗ l) = p0 (αu,l ) = αu,l p, αu,l p(ei ) = u(p(ei )) ⊗ l.
Puesto que β 0 y β 00 son isomorfismos, entonces, por el lema de los cinco, β es
también un isomorfismo.

Proposición 1.8.10. Sean R un anillo conmutativo, A una R-álgebra y M un R-


módulo de presentación finita. Entonces, para cada A-módulo derecho N se tiene el
R-isomorfismo

HomR (MR , NR ) ∼
= HomA (M ⊗R A, NA )

Demostración. Puesto que R es conmutativo, M tiene también estructura de R-


módulo a izquierda, con lo cual M ⊗R A es un R − A-bimódulo, y en consecuencia,
HomA (M ⊗R A, NA ) es un R-módulo a derecha. De manera similar, N es un R-
módulo derecho ya que se tiene el homomorfismo natural de anillos R → A, r 7→ r ·1.
Definimos β : HomR (M, N ) → HomA (M ⊗R A, N ) por β(h) := βh , con βh :
M ⊗R A → N inducida por la función bilineal y R-balanceada β 0 : M × A → N ,
β 0 (m, a) := h(m)·a, con m ∈ M , a ∈ A y h ∈ HomR (M, N ). Entonces, βh (m⊗a) :=
h(m)·a. Notemos que βh es efectivamente un A-homomorfismo de módulos derechos.
A su vez, β es un R-homomorfismo de módulos derechos.
De manera similar a como vimos en la prueba de la proposición anterior, si
M = Rr es R-libre de dimensión finita, entonces β es un R-isomorfismo: en efecto,
HomR (Rr , N ) ∼
= HomR (R, N )r ∼
= N r,
donde este isomorfismo compuesto viene definido por
γ(h) := (h(e1 ), . . . , h(er )),
con h ∈ HomR (Rr , N ) y {ei }ri=1 la base canónica de Rr ; se tiene también el isomor-
fismo
HomA (Rr ⊗R A, N ) ∼
= HomA ((R ⊗R A)r , N ) ∼
= HomA (Ar , N ) ∼
= HomA (A, N )r ∼
= N r,
donde este isomorfismo compuesto viene definido por
δ(g) := (g(e1 ⊗ 1), . . . , g(er ⊗ 1)),
1.9. ANILLOS HEREDITARIOS 55

con g ∈ HomR (Rr ⊗R A, N ) y {ei }ri=1 la base canónica de Rr . Observemos que


β = δ −1 γ, es decir, δβ = γ: δβ(h ⊗ l) = δ(βh ) = (βh (e1 ⊗ 1), . . . , βh (er ⊗ 1)) =
(h(e1 ), . . . , h(er )) = γ(h).
Sea Rr → Rs → M → 0 una presentación finita de M , al igual que en la
demostración de la proposición anterior, se tiene el siguiente diagrama conmutativo
con filas exactas:
0 −−−−→ HomR (M, N ) −−−−→ HomR (Rs , N ) −−−−→ HomR (Rr , N )
  
  
y y y
0 −−−−→ HomR (M ⊗R A, N ) −−−−→ HomA (As , N ) −−−−→ HomA (Ar , N )

Puesto que las dos flechas verticales de la derecha son isomorfismos, entonces la
primera es también un isomorfismo.
La siguiente propiedad es dual a la proposición 1.6.8.
Proposición 1.8.11. Sean R un anillo conmutativo, S un sistema multiplicativo de
R, M un R-módulo de presentación finita y N un R-módulo arbitrario. Entonces,
se tiene el RS −1 -isomorfismo
HomR (M, N )S −1 ∼
= HomRS −1 (M S −1 , N S −1 ).
Demostración. HomR (M, N )S −1 ∼ = HomR (M, N ) ⊗R RS −1 , pero como RS −1 es R-
plano, por la proposición 1.8.9 se obtiene el R-isomorfismo HomR (M, N )⊗R RS −1 ∼ =
−1 −1
HomR (M, N S ); podemos ahora aplicar la proposición 1.8.10 con A := RS y
obtenemos el R-isomorfismo HomR (M, N )S −1 ∼ = HomR (M S −1 , N S −1 ), pero según
la proposición 1.6.3 (vi), este es un RS −1 -isomorfismo.

1.9. Anillos hereditarios


Una tarea clásica en álgebra homológica es investigar clases de anillos para los cuales
los módulos proyectivos coinciden con los libres (véase el teorema 1.4.3). Probaremos
que para dominios de ideales principales (DIP s), los módulos proyectivos y los
libres coinciden. Esta tarea puede ser abordada de una manera bastante general a
través de los llamados anillos hereditatrios que estudiaremos en la presente sección.
Iniciamos con el caso conmutativo y estudiaremos al final los anillos hereditarios no
conmutativos.
Proposición 1.9.1. Sea R un dominio de integridad (DI). Entonces, cada R-módu-
lo proyectivo es sin torsión.
Demostración. En efecto, sea M un módulo proyectivo, entonces M es sumando
directo de un R-módulo libre, es decir, M es un submódulo de un módulo libre F .
Sea X una base de F y supongamos que m ∈ M es un elemento de torsión, es decir,
56 CAPÍTULO 1. ELEMENTOS BÁSICOS DE ÁLGEBRA HOMOLÓGICA

existe 0 6= r ∈ R tal que m · r = 0. Existen elementos x1 , . . . , xt ∈ X y r1 , . . . , rt ∈ R


tales que m = x1 · r1 + · · · + xt · rt , luego r1 r = · · · = rt r = 0, de donde ri = 0 para
cada 1 ≤ i ≤ t, es decir, m = 0.
Corolario 1.9.2. Sean R un DIP y M un R-módulo proyectivo finitamente gene-
rado. Entonces M es libre.
Demostración. En efecto, por la proposición 1.9.1 sabemos que M es un módulo sin
torsión, pero como R es un DIP , M resulta libre (véase [16]).
Corolario 1.9.3. Sean K un cuerpo y M un módulo proyectivo finitamente gene-
rado sobre K[x]. Entonces M es libre.
Demostración. Puesto que K[x] es un DIP , basta aplicar el corolario 1.9.2.
Veremos más adelante que la finitud es una condición superflua en los dos coro-
larios anteriores (véase el corolario 1.9.18).
Definición 1.9.4. Sea R un anillo conmutativo, R es hereditario (AH) si cada
ideal de R es proyectivo. Si R es además un dominio de integridad, entonces se dice
que R es un dominio de Dedekind (DD).
Ejemplo 1.9.5. (i) Todo DIP es un DD.
√ K1 , K2 cuerpos. Entonces, K1 ×√K2 es un AH pero no es un DD.
(ii) Sean
(iii) Z[ −5] no es un DIP . En efecto, Z[ −5] no es un DF U ya que 9 presenta
por lo√menos dos√factorizaciones
 √ en producto de irreducibles: 9 = 3 · 3 =
distintas
2 + −5 2 − −5 . Esto hace que Z[ −5] no sea un DIP . En [25] se demuestra

que Z[ −5] es un DD; la prueba completa es algo extensa y utiliza resultados que
se apartan demasiado del tema y los propósitos del presente cuaderno.
Teorema 1.9.6. Sea R un AH y sea M un R-módulo libre finitamente generado.
Entonces, cada submódulo N de M es isomorfo a una suma directa finita de ideales
de R, y en consecuencia N es proyectivo.
Demostración. Si M = 0, entonces N = 0 y la condición se cumple trivialmente.
Sea M 6= 0 y sea X = {x1 , . . . , xn } una base de M . Vamos a probar el teorema por
inducción sobre n.
n = 1: sea N ≤ M = hx1 i ∼ = R, entonces N es isomorfo a un submódulo de R,
es decir, N es isomorfo a un ideal de R.
Supongamos que cada submódulo de un módulo libre de dimensión n − 1 es
una suma directa finita de ideales de R, y sea N ≤ M , donde X = {x1 , . . . , xn }
es una base de M . Cada elemento v ∈ N se representa en forma única en la forma
v = x1 · r1 + · · · + xn · rn , definimos entonces
φ:N →R
v 7→ rn .
1.9. ANILLOS HEREDITARIOS 57

Nótese que φ es un homomorfismo e I := Im (φ) es un ideal de R; se tiene entonces


el homomorfismo sobreyectivo φ : N → I; por hipótesis I es proyectivo y entonces
N = ker(φ)⊕N 0 , con N 0 ∼ = I. Pero ker (φ) = N ∩(hx1 i ⊕ · · · ⊕ hxn−1 i) es submódulo
del módulo libre hx1 i ⊕ · · · ⊕ hxn−1 i. Por inducción, ker (φ) es isomorfo a una suma
directa finita de ideales de R, ker (φ) ∼ = I1 ⊕ · · · ⊕ It . En total, N ∼
= I1 ⊕ · · · ⊕ It ⊕ I,
y el teorema está probado.

Corolario 1.9.7. Sea R un AH y sea M un R-módulo proyectivo finitamente gene-


rado. Entonces,

(i) M es suma directa finita de ideales de R.

(ii) Si N es un submódulo de M , entonces N es proyectivo.

Demostración. (i) Sabemos que M es sumando directo de un R-módulo libre F de


dimensión finita, por tanto, M es submódulo de F . Por el teorema anterior, M es
suma directa finita de ideales de R.
(ii) N es un submódulo de F , nuevamente por el teorema anterior, N es proyec-
tivo.

Probaremos a continuación que los dominios de Dedekind son noetherianos.

Definición 1.9.8. Sea R un dominio de integridad y K su cuerpo de fracciones. Un


R-submódulo no nulo I de K es un ideal fraccionario de R si existe 0 6= r ∈ R
tal que Ir ⊆ R. Además, se define I −1 := {α ∈ K | Iα ⊆ R}.

Proposición 1.9.9. Sea R un dominio de integridad y K su cuerpo de fracciones.

(i) Si I es un ideal fraccionario de R, entonces II −1 ⊆ R y además I −1 es un


ideal fraccionario de R.

(ii) Sea I ≤ R un ideal no nulo de R, entonces I es un ideal fraccionario de R.

Demostración. (i) I −1 6= ∅ ya que existe r en R − {0} (y por tanto en K) tal que


Ir ⊆ R, luego r ∈ I −1 . Es claro que I −1 es un R-submódulo no nulo de K. Sea ar ∈ I
no nulo, entonces para cada α ∈ I −1 se tiene que ar α ∈ R, es decir, ar I −1 ⊆ R, luego
aI −1 = I −1 a ⊆ R, donde a ∈ R − {0}.
(ii) I es un R-submódulo no nulo de K; además, I1 ⊆ R.

Definición 1.9.10. Un ideal fraccionario I de R es invertible si II −1 = R.

Proposición 1.9.11. Sean R y K como antes.

(i) Sea F(R) la colección de ideales fraccionarios de R. Entonces, F(R) es un


monoide conmutativo.
58 CAPÍTULO 1. ELEMENTOS BÁSICOS DE ÁLGEBRA HOMOLÓGICA

(ii) Sea I(R) la colección de ideales invertibles de R. Entonces I(R) es un grupo


abeliano.

(iii) Si I es un ideal fraccionario invertible, entonces I es finitamente generado.

Demostración. (i) Se define un producto en F(R) por


n
P
IJ := { αi βi | αi ∈ I, βi ∈ J, n ≥ 1}.
i=1

Esta es una operación binaria interna en F(R) ya que IJ es un R-submódulo no


nulo de K, y además IJrs ⊆ R, con r, s no nulos de R tales que Ir ⊆ R, Js ⊆ R.
Es claro que esta operación es asociativa y que IJ = JI; además el neutro de este
producto es R.
(ii) Sea I ∈ I(R), entonces II −1 = R = I −1 I, es decir, cada elemento de I(R)
tiene inverso. n
(iii) Puesto que II −1 = R, entonces 1 = αi βi , con αi ∈ I, βi ∈ I −1 , luego si
P
i=1
n
P
α es un elemento cualquiera de I se tiene que α = αi (αβi ), pero cada αβi ∈ R,
i=1
entonces I = hα1 , . . . , αn i.

Teorema 1.9.12. Sea R un DI e I un ideal fraccionario. I es invertible si, y sólo


si, I es proyectivo.
n
P
Demostración. ⇒): sea I un ideal fraccionario invertible, entonces 1 = αi βi , con
i=1
αi ∈ I, βi ∈ I −1 . Definimos ϕi : I → R por ϕi (α) := αβi , con α ∈ I. Nótese que
Pn n
P
ϕi es un R-homomorfismo; además α = αi (αβi ) = αi ϕi (α). Las colecciones
i=1 i=1
{α1 , . . . , αn } y {ϕ1 , . . . , ϕn } satisfacen las condiciones del teorema 1.2.8, por lo tanto,
I es proyectivo.
⇐): sea I un ideal proyectivo, entonces I posee una base proyectiva {αk }k∈K con
R-homomorfismos {ϕk : I → R}k∈K ; sea α 6= 0 en I, entonces α = α1 ϕ1 (α) + · · · +
αt ϕt (α), con ϕi (α) 6= 0. Se puede también escribir
α = α1 ϕ1α(α) α + · · · + αt ϕtα(α) α = α(α1 ϕ1α(α) + · · · + αt ϕtα(α) )
n
ϕk (α) P
y definimos βk := α
∈ K − {0}, luego 1 = αk βk . Nótese que hβ1 , . . . , βn i es un
k=1
ideal fraccionario de R y además R ⊆ Ihβ1 , . . . , βn i. Veamos que Ihβ1 , . . . , βn i ⊆ R.
Sea γ ∈ I, veamos que para cada generador βi = ϕiα(α) se tiene que γβi ∈ R: si γ = 0,
entonces la propiedad se tiene trivialmente; sea γ 6= 0; sean α = as y γ = bt , entonces
αt
b ϕi ( t ) bϕi ( αt ) ϕi ( bα ) ϕi (γα) ϕi (γα)s ϕi (γαs) aϕi (γ)
γβi = t α
= α
= α
t
= α
= a
= a
= a
= ϕi (γ) ∈ R.
1.9. ANILLOS HEREDITARIOS 59

Corolario 1.9.13. Sea R un DD. Entonces R es Noetheriano.

Demostración. Sea I un ideal de R, si I = 0 entonces I es finitamente generado.


Sea I 6= 0, entonces I es proyectivo, y por tanto, invertible. Pero como vimos en la
proposición 1.9.11, un ideal invertible es finitamente generado.

El teorema 1.9.6 es válido removiendo la hipótesis de finitud, es decir, si R es


AH, entonces cada submódulo de un R-módulo libre es isomorfo a una suma directa
de ideales de R. De igual manera, en el corolario 1.9.7, si M es proyectivo, entonces
M es suma directa de ideales de R, y si N es un submódulo de M entonces N es
también proyectivo. Estas propiedades son ciertas aún en el caso no conmutativo,
tal como veremos enseguida.

Definición 1.9.14. Un anillo A es hereditario a derecha si cada ideal derecho


es proyectivo.

Observación 1.9.15. Compare la definición anterior con la proposición 1.7.8.

Teorema 1.9.16 (Kaplansky). Sean A un anillo hereditario a derecha y M un


A-módulo derecho libre. Entonces, cada submódulo N de M es isomorfo a una suma
directa de ideales derechos de A, y en consecuencia, N es proyectivo.

Demostración. (i) Si M = 0 el resultado se tiene trivialmente.


P Sea M 6= 0 y sea
X := {xα }α∈K una base de M ; sabemos que M = α∈K ⊕ {xα i. Por el principio
de buena ordenación podemos suponer que K es un conjunto bien ordenado, es
decir, cada subconjunto de K tiene primer elemento, y además que K es totalmente
ordenado. Para cada α ∈ K definimos
P
Mα := ⊕ {xβ i,
β<α
P
Mα+1 := ⊕ {xβ i = Mα ⊕ {xα i,
β≤α

M0 := 0,

donde 0 el primer elemento de K. Sea N ≤ M ; cada elemento z ∈ N ∩ Mα+1 admite


una representación única en la forma z = u + xα · a, donde u ∈ Mα . Definimos
φα : N ∩ Mα+1 → A por φα (z) := a; nótese que φα es un A-homomorfismo y su
imagen es un ideal derecho de A, redefinimos entonces φα : N ∩ Mα+1 → Im (φα ) y
este homomorfismo es sobreyectivo; puesto que Im (φα ) es proyectivo entonces N ∩
Mα+1 = ker (φα ) ⊕ Nα , donde
P Nα ∼= Im (φα ). Pero ker (φα ) = N ∩ Mα . P
Probemos que N = α∈K ⊕ Nα . En primer lugar P veamos que N = α∈K Nα :
puesto que para cada α ∈ K, Nα ≤ N , entonces α∈K Nα ≤ N ; supongamos que
60 CAPÍTULO 1. ELEMENTOS BÁSICOS DE ÁLGEBRA HOMOLÓGICA

no se alcanza la igualdad. Para cada z ∈ N ≤ M = ∪α∈K Mα+1 , existe α tal que


z ∈ Mα+1 ; sea αz elP menor α tal que z ∈ Mαz +1 ; suponemos entonces que existen
z ∈ N tales que z ∈ / α∈K Nα , escojamos uno de tales z con la condición de que αz
sea mı́nimo. Entonces, z ∈ N ∩ Mαz +1 , por lo tanto, z = uP+ v, donde u ∈ N ∩ Mαz
y v ∈PNαz . Se sigue entonces que u = z − v ∈ N , pero u ∈
/ α∈K Nα (de lo contrario
z ∈ α∈K Nα ), pero como P u ∈ Mαz , entonces αu < αz , lo cual es contradictorio. Se
tiene entonces que N = α∈K Nα .
Para terminar, veamos que la suma es directa: supongamos que 0 = vα1 +· · ·+vαn ,
con vαi ∈ Nαi y podemos asumir que α1 < · · · < αn . Entonces, vα1 + · · · + vαn−1 =
−vαn ∈ Nαn ∩ (N ∩ Mαn ) = 0. De manera recurrente encontramos que cα1 = · · · =
0 = c αn .
Ası́ pues, N es isomorfo a una suma directa de ideales de R. Como cada ideal es
proyectivo, entonces N es proyectivo.

Corolario 1.9.17. Sea A un AH y sea M un A-módulo proyectivo. Entonces,

(i) M es suma directa de ideales derechos de A.

(ii) Si N es un submódulo de M , entonces N es proyectivo.

Demostración. (i) Como M es proyectivo, entonces M es submódulo de un módulo


libre, el teorema 1.9.16 garantiza que M es suma directa de ideales derechos de R.
(ii) Como en (i), N es un submódulo de un módulo libre, y por el teorema 1.9.16,
N es proyectivo.

Corolario 1.9.18. Sea R un DIP .

(i) Sea M un R-módulo libre. Entonces, cada submódulo N de M es libre, y


dim(N ) ≤ dim(M ).

(ii) Si M proyectivo entonces M es libre.

Demostración. (i) Puesto que cada DIP es un AH (realmente un DD) entonces


dado N ≤ M , N es isomorfo a una suma directa de ideales de R, pero estos son
principales, y por lo tanto libres. En total, N es P
libre. Usando la notación de la
demostración del teorema 1.9.16 se tiene que N = α∈K ⊕ Nα ; teniendo en cuenta
que algunos Nα pueden ser nulos, entonces dim (N ) ≤ card(K) = dim(M ).
(ii) Sea M proyectivo, entonces M es un submódulo de un módulo libre, luego
por (i), M es libre.

Ejemplo 1.9.19. Según la proposición 1.8.3, QZ es plano, pero no es proyectivo,


ya que de lo contrario saldrı́a libre por el corolario anterior (véase [16]).
1.9. ANILLOS HEREDITARIOS 61

Ejemplo 1.9.20. Veamos que el producto infinito de módulos proyectivos no siem-


pre es proyectivo (véase la proposición 1.2.4). Sea ZN el producto cartesiano de N
copias de ZZ , notemos que ZN no es proyectivo. En efecto, si fuera proyectivo, en-
tonces, según el corolario anterior, serı́a libre, pero en Fuchs, L., Infinite Abelian
Groups, Volume I, Academic Press 1970, Teorema 19.2, se demuestra que ZN no es
libre. Veamos otra demostración de este hecho. Si es libre, existe un conjunto no vacı́o
X no enumerable tal que ZN ∼ = Z(X) . Entonces, HomZ (ZN , Z) ∼ = HomZ (Z(X) , Z) ∼
=
Z (producto de X copias de ZZ ). Si logramos demostrar que HomZ (ZN , Z) ∼
X
= Z(N) ,
obtenemos una contradicción: ZX no enumerable, Z(N) enumerable. Probemos en-
tonces el isomorfismo HomZ (ZN , Z) ∼ = Z(N) . Sea X := {µn (1)}n∈N la base canónica
de Z(N) , donde µn : Z → Z(N) es la inyección canónica de la suma directa externa.
Consideremos la aplicación
α
X−
→ HomZ (ZN , Z), α(µn (1)) := αn , αn : ZN → Z, αn [(fi )] := fn .
Nótese que αn es efectivamente un Z-homomorfismo. α induce un Z-homomorfismo,
también denotado por α,
α
Z(N) −
→ HomZ (ZN , Z).
Vamos a demostrar que α es biyectivo. Sea z := k1 µn1 (1) + · · · + km µnm (1) ∈ ker(α),
entonces k1 αn1 + · · · + km αnm = 0, y por lo tanto, para cada (fi ) ∈ ZN se tiene que
k1 fn1 + · · · + km fnm = 0. Tomemos entonces (fi ) tal que fn1 = 1 y fj = 0 para
j 6= n1 , resulta k1 = 0. De igual manera se prueba que k2 = · · · = km = 0, es decir,
z = 0. Para concluir el ejemplo veamos que α es sobreyectivo. Sea h ∈ HomZ (ZN , Z);
necesitamos un elemento z ∈ Z(N) tal que α(z) = h. Con la notación anterior, se debe
tener que k1 αn1 +· · ·+km αnm = h, donde m, kj y nj deben ser calculados, 1 ≤ j ≤ m.
Para cada (fi ) ∈ ZN , se debe cumplir que (k1 αn1 + · · · + km αnm )[(fi )] = h[(fi )], es
(1) (1) (1)
decir, k1 fn1 +· · ·+km fnm = h[(fi )]. Como antes, sea (fi ) ∈ ZN con fn1 = 1 y fj =
(1)
0 para j 6= n1 . Entonces k1 = h[(fi )]; ası́ pues, dado h, si conocemos n1 , entonces
k1 está unı́vocamente determinado. Lo mismo podemos decir para k2 , . . . , km . Solo
resta calcular m y n1 , . . . , nm a partir de h.
Concluimos esta sección con otra caracterización de los anillos hereditarios no
conmutativos, y como corolario, otra caracterización de los dominios de Dedekind.
Teorema 1.9.21 (Cartan-Eilenberg). Sea A un anillo. Entonces las siguientes
condiciones son equivalentes:
(i) A es hereditario a derecha.

(ii) Cada submódulo de un módulo proyectivo es proyectivo.

(iii) Cada cociente de un módulo inyectivo es inyectivo.


62 CAPÍTULO 1. ELEMENTOS BÁSICOS DE ÁLGEBRA HOMOLÓGICA

Demostración. (i)⇒(ii): esto es el contenido del corolario 1.9.17 (ii).


(ii)⇒(i): AA es libre, y por lo tanto, proyectivo; sea I un ideal derecho de A,
entonces I es proyectivo.
(iii)⇒(ii): sean P un módulo proyectivo y P 0 un submódulo de P ; debemos
completar el siguiente diagrama con un homomorfismo h0 : P 0 → Q, donde f es
sobreyectivo y se puede asumir que Q es inyectivo (teorema 1.7.17 (i)):

P0
g
?
Q - Q0
f

Por hipótesis Q0 es inyectivo, entonces se tiene el siguiente diagrama conmutativo

Q0
p
6 Ip p t
g pp
pp
P0 ι
- P

y como P es proyectivo resulta el siguiente diagrama conmutativo:

P
pp
h p p
p t
pp
?
Q - Q0
f

resulta tι = g, f h = t, luego f (hι) = g, el homomorfismo buscado es pues h0 := hι.


(ii)⇒(iii): para esta parte se procede en forma dual a como acabamos de ver,
pero utilizando el teorema 1.7.17 (ii).
En relación con la proposición 1.7.11 se tiene la siguiente caracterización de los
dominios de Dedekind.

Corolario 1.9.22. Sea R un DI. R es un DD si, y sólo si, cada R-módulo divisible
en inyectivo.

Demostración. ⇐): sea N un R-módulo inyectivo, entonces N es divisible (proposi-


ción 1.7.11), luego cada módulo cociente de N es divisible, y por la hipótesis, inyec-
tivo. Según el teorema 1.9.21, R es un DD.
⇒): sean N un R-módulo divisible, I un ideal P no nulo de R y f : I → N
un R-homomorfismo; como I es invertible, sea 1 = ni=1 αi βi , con αi ∈ I y βi ∈
1.10. EJERCICIOS 63

I −1 ⊆P K. Para cada P i existe xi ∈ N tal que fP α ∈ I, entonces


(αi ) = xi αi ; sea P
n
α = ( i=1 αi βi )α = i=1 αi (βi α) y f (α) = f ( i=1 αi (βi α)) = ni=1 f (αi )βi α =
n n
Pn Pn
x i α i βi α = zα, con z := i=1 xi αi βi ∈ N . Definimos la función f : R → N
e
i=1
por fe(r) := z · r, notemos que fe es un R-homomorfismo que extiende f , y por el
teorema de Baer N es inyectivo.

1.10. Ejercicios
f h g
1. Sea M1 − → M2 − → M3 −
→ M4 exacta. Demuestre que f es sobreyectiva si, y sólo
si, g es inyectiva.

2. Considere el siguiente diagrama de A-módulos y A-homomorfismos


M

f
y
g h
0 −−−→ N −−−→ L −−−→ Q
con fila exacta y tal que hf = 0. Demuestre que existe un único A-homomorfismo
t : M → N tal que gt = f .

3. Considere el siguiente diagrama conmutativo de A-módulos y A-homomorfismos


0 0 0
  
  
y y y
f0 f0
0 −−−→ M10 −−−
1
→ M20 −−−
2
→ M30 −−−→ 0
  
p q r
y y y
f1 f2
0 −−−→ M1 −−−→ M2 −−−→ M3 −−−→ 0
  
s  u
y yt y
f 00 f 00
0 −−−→ M100 −−−
1
→ M200 −−−
2
→ M300 −−−→ 0
  
  
y y y
0 0 0
con columnas exactas. Demuestre que:

(i) Si las dos últimas filas son exactas, entonces la primera fila es exacta.
(ii) Si las dos primeras filas son exactas, entonces la última fila es exacta.
64 CAPÍTULO 1. ELEMENTOS BÁSICOS DE ÁLGEBRA HOMOLÓGICA

4. Demuestre la parte (ii) del teorema 1.1.2.

5. Demuestre el lema 1.2.14. Sugerencia: Tenga en cuenta lo siguiente: (a) El


lema de Schanuel es válido no solo para iM sino para cualquier isomorfismo
M∼ = N . (b) Considere las sucesiones exactas
f1
0 → ker(f1 ) → P1 −
→M →0
g1
0 → ker(g1 ) → Q1 −
→ N → 0.
(c) Construya sucesiones exactas en la forma

0 → K1 → Pn → Pn−1 → · · · → P3 → P2 ⊕ Q1 → ker(f1 ) ⊕ Q1 → 0
0 → K2 → Qn → Qn−1 → · · · → Q3 → Q2 ⊕ P1 → ker(g1 ) ⊕ P1 → 0.

6. Demuestre el corolario 1.4.2.

7. Demuestre las propiedades (a) - (f); (i), (j,) (l), (q) y (r) de la sección 1.3.
Q Q
8. Ilustre con un ejemplo que M ⊗A ( j Nj )  j (M ⊗A Nj ). Por ejemplo,
considere A := Z, M := Q, Nj := Zpj , con p irreducible, j ≥ 1.

9. Demuestre que Q ⊗Z Q ∼
= Q.
10. Sea R → B un homomorfismo de anillos conmutativos y sean M, N dos R-
módulos. Demuestre que

(M ⊗R N ) ⊗R B ∼
= (M ⊗R B) ⊗B (N ⊗R B).

11. Sea M un A − A-bimódulo; se dice que M es invertible si existe un A − A-


bimódulo de tal forma que M ⊗A N ∼ = A y N ⊗A M ∼ = A (isomorfismo de
A − A-bimódulos). Demuestre que:

(i) EndA (MA ) ∼=A


(ii) EndA (A M ) ∼
= Aop .
12. Sea R un anillo conmutativo y sea A una R-álgebra. Note que Ae := A ⊗R A
es una R-álgebra (véase [17]) y se denomina el álgebra envolvente de A.
Demuestre que:

(i) Un grupo abeliano M tiene estructura de A − A-bimódulo si, y sólo si,


M es un Ae -módulo izquierdo si, y sólo si, M es un Ae -módulo derecho.
(ii) Demuestre que si A es un anillo simple, entonces A es un Ae -módulo
simple (Sugerencia: utilice (i) y considere los ideales biláteros de A).
1.10. EJERCICIOS 65

13. Sean A un anillo, P un A-módulo proyectivo e I un ideal bilátero propio de


A. Demuestre que P/P I es un A/I-módulo proyectivo.

14. Sean A un anillo y M un A-módulo; un recubrimiento proyectivo, tam-


bién denominado, envolvente proyectiva de M , es un módulo P con un
homomorfismo sobreyectivo f : P → M tal que ker(f ) pequeño en P (un
submódulo N de un módulo L es pequeño si para cada submódulo K de L se
cumple N + K = L si, y sólo si, K = L). Demuestre que dos recubrimientos
proyectivos de M son isomorfos.

15. Sea P un A-módulo izquierdo finitamente generado. Demuestre que P es


proyectivo si, y sólo si, iP ∈ Im(v), donde v : HomA (P, A)⊗A P → HomA (P, P )
es definido por f ⊗ x 7→ fe y fe(y) := f (y)x, con f ∈ HomA (P, A), x, y ∈ P
(Sugerencia: use el teorema de la base proyectiva).

16. Demuestre la proposición 1.5.6.

17. Demuestre la proposición 1.6.1.

18. Demuestre el corolario 1.6.2.

19. Demuestre el isomorfismo (1.6.1).

20. Demuestre la proposición 1.6.10.

21. Sean A un anillo primo y S un subconjunto multiplicativo de elementos re-


gulares de A (es decir, elementos que no son divisores de cero) de tal forma
que AS −1 existe. Demuestre que AS −1 es también un anillo primo.

22. Sea R un dominio de integridad (anillo conmutativo sin divisores de cero) y


sean I, J ideales de R tales que IP = JP para cada ideal maximal P de R.
Demuestre que I = J (Sugerencia: dado x ∈ J considere (I : x) y el elemento
x
1
∈ JP ).

23. Sean R un anillo conmutativo y S1 ⊆ S2 subconjuntos multiplicativos de R.


Demuestre que para cada R-módulo M se tiene el isomorfismo (M S1−1 )S2−1 ∼
=
M S2−1 ∼
= (M S2−1 )S1−1 .

24. Sea A un anillo y sea S un subconjunto multiplicativo de A tal que AS −1


existe. Sea M un A-módulo; un submódulo L de M se dice S-saturado si
satisface la siguiente condición:

Si m ∈ M y s ∈ S de tal forma que m · s ∈ L, entonces m ∈ L.


66 CAPÍTULO 1. ELEMENTOS BÁSICOS DE ÁLGEBRA HOMOLÓGICA

Demuestre que existe una correspondencia biyectiva entre los submódulos S-


saturados de M y los submódulos de M S −1 dada por L 7→ LS −1 .
25. Sea R un anillo conmutativo y sea M un R-módulo tal que existen enteros
m, n ≥ 1 con M ⊕ Rn ∼
= Rm . Demuestre que
(a) m ≥ n.
(b) Si p, q ≥ 1 son enteros tales que M ⊕ Rp ∼
= Rq , entonces m − n = q − p.
26. Demuestre la proposición 1.7.10.
27. Demuestre la parte (ii) del toerema 1.7.17.
28. Demuestre la afirmación de la observación 1.7.18.
29. Sean M, N módulos inyectivos. Demuestre que M/N es inyectivo.
30. Sean R un anillo conmutativo, N un R-módulo inyectivo y A una R-álgebra.
Demuestre que HomR (A, N ) es un A-módulo inyectivo.
31. (Lema de Schanuel para inyectivos) Dadas dos sucesiones exactas

0 → M → E1 → L1 → 0,
0 → N → E2 → L2 → 0,

donde E1 , E2 son inyectivos y M ∼


= N , demuestre que L1 ⊕ E2 ∼
= L2 ⊕ E1 .
32. Dadas las sucesiones exactas de A-módulos

0 → M → D0 → D1 → · · · → Dn−1 → Dn → L1 → 0
0 → N → E0 → E1 → · · · → En−1 → En → L2 → 0

con Di , Ei inyectivos, 0 ≤ i ≤ n, y M ∼
= N , demuestre que

L1 ⊕ En ⊕ Dn−1 ⊕ En−2 ⊕ · · · ∼
= L2 ⊕ Dn ⊕ En−1 ⊕ Dn−2 ⊕ · · ·

33. Sea P un A-módulo plano y Q un A−B-bimódulo el cual es B-plano. Entonces,


P ⊗ Q es B-plano. En particular, si R es un anillo conmutativo y P, Q son
R-módulos planos, entonces P ⊗ Q es R-plano.
34. Demuestre la conmutatividad del diagrama de la proposición 1.8.9.
35. Demuestre la conmutatividad del diagrama de la proposición 1.8.10.
36. Concluir la prueba del ejemplo 1.9.20.
Capı́tulo 2

Ext

Este capı́tulo está dedicado al estudio de una técnica que extiende el Hom (véase
[16]) y resulta muy útil para el análisis de propiedades homológicas de anillos y
módulos. Como siempre, vamos a suponer que A es un anillo no necesariamente
conmutativo y, salvo que se advierta lo contrario, los módulos serán considerados a
derecha.

2.1. Definición
Sean M un A-módulo y F0 el A-módulo libre canónico con una base {em }m∈M
de cardinalidad igual a la de M , entonces existe un homomorfismo sobreyectivo
f0 : F0 −→ M , definido por f0 (em ) := m, m ∈ M . Si consideramos el núcleo de f0 ,
entonces podemos construir el módulo libre F1 y el correspondiente homomorfismo
sobreyectivo f10 : F1 −→ ker(f0 ), de tal forma que obtenemos la sucesión exacta
f1 f0
F1 −
→ F0 −
→ M → 0,
donde f1 := lf10 y l es la inclusión de ker(f0 ) en F0 . Podemos repetir esta construcción
para F1 y continuar de esta forma para obtener la sucesión exacta
fi+1 fi f1 f0
· · · → Fi+1 −−→ Fi −
→ Fi−1 → · · · → F1 −
→ F0 −
→ M → 0. (2.1.1)
Hemos demostrado que todo A-módulo M tiene al menos una sucesión exacta como
en (2.1.1) con módulos libres Fi , i ≥ 0. Esto también muestra que M tiene al menos
una sucesión exacta con módulos proyectivos Pi en la siguiente forma
fi+1 fi f1 f0
· · · → Pi+1 −−→ Pi −
→ Pi−1 → · · · → P1 −
→ P0 −
→ M → 0. (2.1.2)
Sea ahora N un A-módulo cualquiera, entonces se obtiene el complejo
0 f∗ 1 f∗
0 → HomA (M, N ) −→ HomA (P0 , N ) −→ HomA (P1 , N ) → · · · (2.1.3)

67
68 CAPÍTULO 2. EXT

con f0∗ inyectivo. En efecto, como HomA ( , N ) es exacto a izquierda, entonces f0∗

inyectivo y además fi+1 fi∗ = (fi fi+1 )∗ = 0∗ = 0. Esta última sucesión induce la
siguiente definición.
Definición 2.1.1. Sean M y N dos A-módulos, se definen los grupos abelianos
Ext0A (M, N ) := HomA (M, N ),

(2.1.4)
ExtiA (M, N ) := ker(fi+1 )/Im(fi∗ ), i ≥ 1.
Notemos que la definición (2.1.4) parece depender de la resolución proyectiva
elegida. Sin embargo, se tiene la siguiente propiedad.
Teorema 2.1.2. Para cada i ≥ 0, el grupo ExtiA (M, N ) es independiente de la
resolución proyectiva (2.1.2).
Demostración. Dividimos esta prueba en varios pasos.
α
Paso 1. En primer lugar notemos que si M − → M 0 es un A-homomorfismo,
entonces construimos para M y para M 0 resoluciones proyectivas
fi+1 fi f1 f0
(P ) : · · · −−−−−−→ Pi+1 −−−−−−→ Pi −−−−−−→ · · · −−−−−−→ P1 −−−−−−→ P0 −−−−−−→ M −−−−−−→ 0

αy
 (2.1.5)

gi+1 gi g1 g0
(Q) : · · · −−−−−−→ Qi+1 −−−−−−→ Qi −−−−−−→ · · · −−−−−−→ Q1 −−−−−−→ Q0 −−−−−−→ M 0 −−−−−−→ 0

La sobreyectividad de g0 garantiza la existencia de un homomorfismo α0 : P0 →


Q0 tal que g0 α0 = αf0 . Ahora consideremos el homomorfismo compuesto g0 α0 f1 ,
entonces g0 α0 f1 = αf0 f1 = 0, por lo tanto, Im(α0 f1 ) ⊆ ker(g0 ) = Im(g1 ). La
proyectividad de P1 garantiza la existencia de un homomorfismo α1 : P1 → Q1
tal que α0 f1 = g1 α1 . Continuando de esta manera resulta el siguiente diagrama
conmutativo con filas exactas:
fi+1 fi f1 f0
−−−→ Pi+1 −−−→ Pi −−−→ · · · −−−→ P1 −−−→ P0 −−−→ M −−−→ 0
    
αi+1 y αi y α1 y α0 y
    
αy

gi+1 gi g1 g0
−−−→ Qi+1 −−−→ Qi −−−→ · · · −−−→ Q1 −−−→ Q0 −−−→ M 0 −−−→ 0
Aplicamos HomA ( , N ) y resulta el siguiente diagrama conmutativo

g0 ∗
g1 ∗
g2
0 −−−−−−→ HomA (M 0 , N ) −−−−−−→ HomA (Q0 , N ) −−−−−−→ HomA (Q1 , N ) −−−−−−→ HomA (Q2 , N ) −−−−−−→ · · ·
   
α∗ y α∗ α∗ α∗
   
0y 1y 2y

f0 ∗
f1 ∗
f2
0 −−−−−−→ HomA (M, N ) −−−−−−→ HomA (P0 , N ) −−−−−−→ HomA (P1 , N ) −−−−−−→ HomA (P2 , N ) −−−−−−→ · · ·

donde las filas son complejos. Entonces se inducen los homomorfismos de grupos
α∗
ExtiA (M 0 , N )(Q) = ker(gi+1

)/Im(gi∗ ) −→
i ∗
ker(fi+1 )/Im(fi∗ ) = ExtiA (M, N )(P )
definidos por
2.1. DEFINICIÓN 69

αi∗ (h) := αi∗ (h) = hαi , i ≥ 1,


∗ ∗
donde h ∈ ker(gi+1 ) ⊆ HomA (Qi , N ). Notemos que hαi ∈ ker(fi+1 ); además, αi∗
está bien definida y es un homomorfismo de grupos abelianos. Se observa entonces
que ExtiA (M 0 , N ) parece depender de la resolución (Q), ExtiA (M, N ) de la resolu-
ción (P) y αi∗ de αi . Debemos mostrar que estos objetos son independientes de las
resoluciones y de los homomorfismos αi .
Paso 2. Fijemos la resolución (P) para M y la resolución (Q) para M 0 ; probemos
que el homomorfismo entre ExtiA (M 0 , N )(Q) y ExtiA (M, N )(P ) no depende de αi .
Para esto vamos a demostrar primero que si α = 0, entonces αi∗ = 0 para cada
i ≥ 1. En efecto, puesto que g0 α0 = 0, entonces Im(α0 ) ⊆ ker(g0 ) = Im(g1 ), y por
la proyectividad de P0 existe un homomorfismo d0 : P0 → Q1 tal que g1 d0 = α0 .
Tenemos α1 − d0 f1 ∈ HomA (P1 , Q1 ), entonces g1 (α1 − d0 f1 ) = g1 α1 − g1 d0 f1 =
α0 f1 − α0 f1 = 0, por lo tanto, Im(α1 − d0 f1 ) ⊆ ker(g1 ) = Im(g2 ). Existe entonces
un homomorfismo d1 : P1 → Q2 tal que g2 d1 = α1 − d0 f1 . Se tiene entonces que
α1 = g2 d1 + d0 f1 , y podemos continuar de esta misma manera para definir un
homomorfismo di : Pi → Qi+1 tal que
αi = gi+1 di + di−1 fi , i ≥ 1.
De esta relación resulta
αi∗ = d∗i gi+1

+ fi∗ d∗i−1

y para h ∈ ker(gi+1 ) se tiene que
αi∗ (h) = (d∗i gi+1

)(h) + (fi∗ d∗i−1 )(h)
= d∗i (gi+1

(h)) + fi∗ (d∗i−1 (h))
= 0 + fi∗ (d∗i−1 (h))
= fi∗ (d∗i−1 (h)),
esto indica que αi∗ (h) ∈ Im(fi∗ ), y de esta manera αi∗ = 0.
Pasamos ahora a considerar el homomorfismo α : M → M 0 . Supongamos que
tenemos dos colecciones de homomorfismos αi , αi0 tal que el siguiente diagrama con-
muta
fi+1 fi f1 f0
−−−→ Pi+1 −−−→ Pi −−−→ · · · −−−→ P1 −−−→ P0 −−−→ M −−−→ 0
    
αi+1 yα0i+1 αi yα0i α1 yα01 α0 yα00
    
αy

gi+1 gi g1 g0
−−−→ Qi+1 −−−→ Qi −−−→ · · · −−−→ Q1 −−−→ Q0 −−−→ M 0 −−−→ 0
es decir, para cada i ≥ 0, αi fi+1 = gi+1 αi+1 y αi0 fi+1 = gi+1 αi+1
0
, además, αf0 = g0 α0
0
y αf0 = g0 α0 . De estas relaciones se desprende que
g0 (α0 − α00 ) = 0 = 0f0
(αi − αi0 )fi+1 = gi+1 (αi+1 − αi+1
0
).
70 CAPÍTULO 2. EXT

Estamos entonces en el caso ya probado de un homomorfismo nulo entre M y M 0 , con


lo cual podemos concluir que (αi − αi0 )∗ = 0, es decir, αi∗ = (αi0 )∗ . Hemos completado
la prueba de lo anunciado en el paso 2.
Paso 3. Vamos ahora a demostrar que ExtiA (M, N ) no depende de la resolución
(P) usada para definirlo, es decir, probaremos que ExtiA (M, N )(P ) ∼ = ExtiA (M, N )(Q) .
Para esto consideremos entonces dos resoluciones (P) y (Q) de M y los siguientes
diagramas conmutativos:

fi+1 fi f1 f0
−−−→ Pi+1 −−−→ Pi −−−→ · · · −−−→ P1 −−−→ P0 −−−→ M −−−→ 0
    
αi+1 y αi y α1 y α0 y
    
iM y

gi+1 gi g1 g0
−−−→ Qi+1 −−−→ Qi −−−→ · · · −−−→ Q1 −−−→ Q0 −−−→ M −−−→ 0
    

βi+1 y

βi y

β1 y

β0 y

iM y

fi+1 fi f1 f0
−−−→ Pi+1 −−−→ Pi −−−→ · · · −−−→ P1 −−−→ P0 −−−→ M −−−→ 0
    
αi+1 y αi y α1 y α0 y
    
iM y

gi+1 gi g1 g0
−−−→ Qi+1 −−−→ Qi −−−→ · · · −−−→ Q1 −−−→ Q0 −−−→ M −−−→ 0

Tenemos entonces
α∗ β∗
ExtiA (M, N )(Q) −→
i
ExtiA (M, N )(P ) −→
i
ExtiA (M, N )(Q)

(αi βi )∗
ExtiA (M, N )(Q) −−−−→ ExtiA (M, N )(Q)

y además,

i∗
ExtiA (M, N )(Q) −
→i
ExtiA (M, N )(Q) ,

donde ii : Qi → Qi es la idéntica de Qi . De acuerdo con el paso 2, i∗i = (αi βi )∗ ,


pero (αi βi )∗ = βi∗ αi∗ e i∗i = iExtiA (M,N )(Q) . De igual manera, (βi αi )∗ = αi∗ βi∗ y
i∗i = iExtiA (N,L)(P ) , esto muestra que ExtiA (M, N )(P ) ∼
= ExtiA (M, N )(Q) , y el teorema
está probado.

Según el teorema anterior, para definir y calcular el grupo ExtiA (M, N ) podemos
utilizar resoluciones libres como en (2.1.1) en lugar de resoluciones proyectivas. Es
decir, una resolución libre para M basta para calcular ExtiM (M, N ). Presentamos a
continuación una interpretación de los grupos Ext desde el punto de vista funtorial
(véase [19]).
2.1. DEFINICIÓN 71

Corolario 2.1.3. Sea N un A-módulo fijo. Entonces, para cada i ≥ 0


Exti ( ,N )
M odA −−−A−−−→ Ab
M 7−→ ExtiA (M, N )
α α∗
→ M 0 7−→ ExtiA (M 0 , N ) −→
M− i
ExtiA (M, N )

es un funtor contravariante, donde M odA denota la categorı́a de los A-módulos a


derecha y Ab la categorı́a de los grupos abelianos.
Demostración. En la prueba del teorema 2.1.2 vimos que ExtiA (M, N ) es indepen-
diente de la resolución (P) usada para definirlo, de igual manera, ExtiA (M 0 , N ) es
independiente de la resolución (Q). Además, vimos que fijando estas resoluciones, el
homomorfismo αi∗ : ExtiA (M 0 , N ) → ExtiA (M, N ) es independiente de αi . Ası́ pues
tenemos el homomorfismo
α∗
ExtiA (M 0 , N ) = ker(gi+1

)/Im(gi∗ ) −→
i ∗
ker(fi+1 )/Im(fi∗ ) = ExtiA (M, N )

definido por
αi∗ (h) = hαi ,

donde h ∈ ker(gi+1 ). Notemos que si α = iM , entonces podemos tomar la misma
resolución proyectiva para M de tal forma que para cada i, αi = iPi , y entonces
α β
→ M0 →
αi∗ = iExtiA (M,N ) . De igual forma, si M − − M 00 es una composición de ho-
momorfismos, entonces veamos que para cada i ≥ 0, (βα)∗i = αi∗ βi∗ : se tiene el
diagrama
fi+1 fi f1 f0
−−−→ Pi+1 −−−→ Pi −−−→ · · · −−−→ P1 −−−→ P0 −−−→ M −−−→ 0
    
αi+1 y αi y α1 y α0 y
    
αy

gi+1 gi g1 g0
−−−→ Qi+1 −−−→ Qi −−−→ · · · −−−→ Q1 −−−→ Q0 −−−→ M 0 −−−→ 0
    

βi+1 y

βi y

β1 y

β0 y

βy

hi+1 h h h
−−−→ Li+1 −−−→ Li −−−i→ · · · −−−→ L1 −−−
1 0
→ L0 −−−→ M 00 −−−→ 0
El homomorfismo βα induce morfismos θi := (βα)i : Pi → Li que satisfacen θi fi+1 =
hi+1 θi+1 , pero para este mismo homomorfismo βα la colección de morfismos γi :=
βi αi también satisface esta condición. En efecto, γi fi+1 = βi αi fi+1 = βi gi+1 αi+1 =
hi+1 βi+1 αi+1 = hi+1 γi+1 . Por lo tanto, tal y como vimos en la demostración del
teorema 2.1.2, los homomorfismos inducidos por estas dos colecciones son iguales,
es decir, θi∗ = γi∗ , luego (βα)∗i = γi∗ = αi∗ βi∗ = αi∗ βi∗ . Todo lo anterior muestra que
ExtiA ( , N ) es un funtor contravariante.
72 CAPÍTULO 2. EXT

Observación 2.1.4. (i) Si M es un A-módulo y N es un B − A bimódulo, entonces


ExtiA (M, N ) es un B-módulo a izquierda, para cada i ≥ 0. Ası́ pues, para cada
i ≥ 0, ExtiA ( , N ) : M odA →B M od es un funtor contravariante.
En particular, si R es un anillo conmutativo y M, N son R-módulos, entonces
ExtiR (M, N ) es un R-módulo y ExtiR ( , N ) : M odR → M odR es un funtor con-
travariante, para cada i ≥ 0.
(ii) La teorı́a covariante de los funtores extiA (M, ) se construye de manera si-
milar, en este caso la construcción se realiza por medio de resoluciones inyectivas de
un A-módulo N y se puede demostrar que para cada i ≥ 0 y cualesquiera A-módu-
los M y N se tiene ExtiA (M, N ) ∼ = extiA (M, N ) (isomorfismo de grupos abelianos,
véase [33]). Veamos solo la parte correspondiente al paso 1 de la demostración del
teorema 2.1.2. Sea N un A-módulo, entonces existe un A-módulo inyectivo L0 y
f0
un A-homomofismo inyectivo N − → L0 (teorema 1.7.16); consideremos el módulo
cociente L0 /Im(f0 ), entonces existe un módulo inyectivo L1 y un homomorfimso
l
inyectivo L0 /Im(f0 ) − → L1 ; si j : L0 → L0 /Im(f0 ) es el homomorfismo canónico
f0 f1
entonces resulta la sucesión exacta 0 → N −
→ L0 −
→ L1 , donde f1 := lj. Podemos
continuar de la misma manera y obtener una resolución inyectiva para N :
f0 f1 fi fi+1
0→N −
→ L0 − → Li −−→ Li+1 → · · ·
→ L1 → · · · → Li−1 −

Sea M un A-módulo, aplicamos HomA (M, ) y obtenemos el complejo


(f0 )∗ (f1 )∗
0 → HomA (M, N ) −−−→ HomA (M, L0 ) −−−→ HomA (M, L1 ) → · · ·

el cual permite definir los grupos abelianos


ext0A (M, N ) := HomA (M, N ),
extiA (M, N ) := ker((fi+1 )∗ )/Im((fi )∗ ), i ≥ 1.
α
→ N 0 es un A-homomorfismo, entonces construimos para N y para N 0 resolu-
Si N −
ciones inyectivas
f0 f1 fi fi+1
(L) : 0 −−−−−−→ N −−−−−−→ L0 −−−−−−→ · · · −−−−−−→ Li−1 −−−−−−→ Li −−−−−−→ Li+1 −−−−−−→ · · ·

αy

g0 g1 gi gi+1
(K) : 0 −−−−−−→ N 0 −−−−−−→ K0 −−−−−−→ · · · −−−−−−→ Ki−1 −−−−−−→ Ki −−−−−−→ Ki+1 −−−−−−→ · · ·

Por la inyectividad de K0 resulta el diagrama conmutativo

K0
p
6Ip p α0
g0 α pp
pp
N - L0
f0
2.2. PROPIEDADES BÁSICAS 73

De igual manera, como K1 es inyectivo se tiene el diagrama conmutativo

K1
p
6 Ip p
pp
α0 p p α1
pp
pp
pp
L0 / ker(f1 ) - L 1
f1

donde f1 (x0 ) := f1 (x0 ); α0 (x0 ) := g1 α0 (x0 ), con x0 ∈ L0 . Notemos que α0 , f1


están bien definidas y son A-homomorfismos, con f1 inyectivo. Obserevemos que
α1 f1 = g1 α0 . Podemos repetir esta construcción y obtenemos el siguiente diagrama
conmutativo
f0 f1 fi fi+1
(L) : 0 −−−−−−→ N −−−−−−→ L0 −−−−−−→ · · · −−−−−−→ Li−1 −−−−−−→ Li −−−−−−→ Li+1 −−−−−−→ · · ·
    
αy
 α0 
y αi−1 
y αi 
y αi+1 
y
g0 g1 gi gi+1
(K) : 0 −−−−−−→ N 0 −−−−−−→ K0 −−−−−−→ · · · −−−−−−→ Ki−1 −−−−−−→ Ki −−−−−−→ Ki+1 −−−−−−→ · · ·

aplicamos HomA (M, ) y obtenemos el diagrama conmutativo de complejos


(f0 )∗ (f1 )∗ (f2 )∗
(L) : 0 −−−−−−→ HomA (M, N ) −−−−−−→ HomA (M, L0 ) −−−−−−→ HomA (M, L1 ) −−−−−−→ HomA (M, L2 ) −−−−−−→ · · ·
   
α∗ 
y (α0 )∗ y
 (α1 )∗ y
 (α2 )∗ y

(g0 )∗ (g1 )∗ (g2 )∗


(K) : 0 −−−−−−→ HomA (M, N 0 ) −−−−−−→ HomA (M, K0 ) −−−−−−→ HomA (M, K1 ) −−−−−−→ HomA (M, K2 ) −−−−−−→ · · ·

Se induce entonces la aplicación de grupos abelianos


(αi )∗
extiA (M, N )(L) = ker(fi+1 )∗ /Im(fi )∗ −−−→ ker((gi+1 )∗ )/Im((gi )∗ ) = extiA (M, N 0 )(K)
definda por
(αi )∗ (h) := (αi )∗ (h) = αi h, i ≥ 1.
El resto de la construcción de los grupos extiA (M, N ) se realiza en forma análoga a
como vimos en la demostración del teorema 2.1.2.

2.2. Propiedades básicas


Pasamos ahora a demostrar uno de los teoremas centrales y de frecuente aplicación
de la técnica Ext.
Teorema 2.2.1 (Sucesión exacta larga). La sucesión exacta de A-módulos
ι π
0→K−
→N −
→M →0 (2.2.1)
induce las siguientes sucesiones exactas de grupos abelianos, donde L es un A-módulo
fijo cualquiera:
74 CAPÍTULO 2. EXT

(i)
π∗ ι∗ θ
0 →Ext0A (M, L) −→ Ext0A (N, L) −
→ Ext0A (K, L) −
→1

π ιe θ
Ext1A (M, L) −→
1
Ext1A (N, L) −
→1
Ext1A (K, L) −
→2
f

π ιe θ
Ext2A (M, L) −→
2
Ext2A (N, L) −
→2
Ext2A (K, L) −
→3
···
f

πe ιe θi+1
ExtiA (M, L) −
→i
ExtiA (N, L) −
→i
ExtiA (K, L) −−→
πg
i+1 ιg
i+1
Exti+1 i+1
A (M, L) −−→ ExtA (N, L) −−→ Exti+1
A (K, L) → · · ·

(ii)
ι π θ
0 →Ext0A (L, K) −
→∗
Ext0A (L, N ) −→

Ext0A (L, M ) −
→1

ι π θ
Ext1A (L, K) −
→1
Ext1A (L, N ) −→
1
Ext1A (L, M ) −
→2

ι π θ
Ext2A (L, K) −
→2
Ext2A (L, N ) −→
2
Ext2A (L, M ) −
→3
···
ι π θi+1
ExtiA (L, K) −
→i
ExtiA (L, N ) −
→i
ExtiA (L, M ) −−→
ιi+1 πi+1
Exti+1
A (L, K) −−→ Exti+1 i+1
A (L, N ) −−→ ExtA (L, M ) → · · ·

Demostración. Solo presentaremos la prueba de (ii), la demostración de (i) queda


como ejercicio para el lector.
Puesto que HomA (L, ) es exacto a izquierda, debemos demostrar que Im(π∗ ) =
ker(θ1 ), Im(θ1 ) = ker(ι1 ) y que para cada i ≥ 1, Im(ιi ) = ker(πi ), Im(πi ) =
ker(θi+1 ), Im(θi+1 ) = ker(ιi+1 )
Paso 0. Para realizar todas las pruebas anteriores, consideremos una resolución
proyectiva de L:
fi+1 fi f1 f0
· · · → Pi+1 −−→ Pi −
→ Pi−1 → · · · → P1 −
→ P0 −
→ L → 0, (2.2.2)
aplicamos HomA ( , K), HomA ( , N ) y HomA ( , M ) y obtenemos el siguiente
diagrama conmutativo de complejos:
K
f0 K
f1 K
f2
0 −−−−−−→ HomA (L, K) −−−−−−→ HomA (P0 , K) −−−−−−→ HomA (P1 , K) −−−−−−→ HomA (P2 , K) −−−−−−→ · · ·
   
ι∗ 
y ι0 
y ι1 
y ι2 
y
N
f0 N
f1 N
f2
0 −−−−−−→ HomA (L, N ) −−−−−−→ HomA (P0 , N ) −−−−−−→ HomA (P1 , N ) −−−−−−→ HomA (P2 , N ) −−−−−−→ · · ·
   
π∗ 
y π0 
y π1 
y π2 
y
M
f0 M
f1 M
f2
0 −−−−−−→ HomA (L, M ) −−−−−−→ HomA (P0 , M ) −−−−−−→ HomA (P1 , M ) −−−−−−→ HomA (P2 , M ) −−−−−−→ · · ·

Veamos la construcción de θ1 :
θ
HomA (L, M ) = Ext0A (L, M ) −
→1
Ext1A (L, K) = ker(f2K )/Im(f1K ) (2.2.3)
h 7→ θ1 (h) := h0 (2.2.4)
2.2. PROPIEDADES BÁSICAS 75

donde h0 se define por el siguiente diagrama tal y como vimos en la demostración


del teorema 2.1.2 al usar que P0 y P1 son proyectivos:

f2 f1 f0
P2 −−−→ P1 −−−→ P0 −−−→ L −−−→ 0
  
 0
yh
 00
yh

yh (2.2.5)
ι π
0 −−−→ K −−−→ N −−−→ M −−−→ 0

Nótese que efectivamente h0 ∈ ker(f2K ). Para concluir este paso inicial veamos que
θ1 está bien definida, es decir, no depende de los homomorfismos h00 , h0 que hacen el
diagrama anterior conmmutativo: sean h00 , g 00 : P0 → N tales que πh00 = hf0 = πg 00 ,
y de la misma manera, sean h0 , g 0 : P1 → K tales que ιh0 = h00 f1 , ιg 0 = g 00 f1 .
Entonces, ι(h0 − g 0 ) = (h00 − g 00 )f1 , la idea es demostrar que h0 = g 0 , es decir, que
h0 − g 0 ∈ Im(f1K ). Debemos entonces construir t ∈ HomA (P0 , K) tal que h0 − g 0 =
f1K (t) = tf1 . Como πh00 = πg 00 , entonces Im(h00 − g 00 ) ⊆ ker(π) = Im(ι), se tiene
entonces el siguiente diagrama conmutativo

P0
p
p pp
t pp
p h00 −g 00
p
pp
?
K - Im(ι)
ι

Resulta ιtf1 = (h00 − g 00 )f1 = ι(h0 − g 0 ), luego tf1 = h0 − g 0 . Es claro que θ1 es un


homomorfismo de grupos abelianos (de módulos, en caso de ser esta la situación
general que se esté considerando).
Paso 1. Im(π∗ ) = ker(θ1 ): tenemos
∗π 1 θ
HomA (L, N ) −→ HomA (L, M ) −
→ Ext1A (L, K) = ker(f2K )/Im(f1K )

sea h ∈ Im(π∗ ), entonces existe p ∈ HomA (L, N ) tal que h = πp, definimos h00 :=
pf0 , luego hf0 = πpf0 = πh00 ; además, h00 f1 = pf0 f1 = 0 = ι0. Se tiene entonces que
el diagrama (2.2.5) es conmutativo con h0 := 0, de donde θ1 (h) = h0 = 0 = 0, es
decir, Im(π∗ ) ⊆ ker(θ1 ).
Sea ahora h ∈ ker(θ1 ), entonces h ∈ HomA (L, M ) y θ1 (h) = h0 = 0, de donde h0 ∈
Im(f1K ) ⊆ ker(f2K ) ⊆ HomA (P1 , K), existe t ∈ HomA (P0 , K) tal que h0 = f1K (y), es
decir, h0 = tf1 ; además, se tienen las condiciones del diagrama (2.2.5). Necesitamos
definir un homomorfismo p : L → N tal que πp = h, con lo cual tendrı́amos
h = π∗ (p) ∈ Im(π∗ ); sea x ∈ L, existe u ∈ P0 tal que f0 (u) = x, definimos
p(x) := (h00 − ιt)(u). Se prueba fácilmente que p está bien definida; finalmente,
πp(x) := π(h00 − ιt)(u) = πh00 (u) − πιt(u) = hf0 (u) = h(x).
76 CAPÍTULO 2. EXT

Paso 2. Im(θ1 ) = ker(ι1 ): de manera más general definamos primero los homo-
morfismos ιi :
K ι
ker(fi+1 )/Im(fiK ) = ExtiA (L, K) −
→i
ExtiA (L, N ) = ker(fi+1
N
)/Im(fiN )
(2.2.6)
h0 7→ ιi (h0 ) := ιi (h0 ) = ιh0

con h0 ∈ ker(fi+1K
) ⊆ HomA (Pi , K). Notemos que fi+1N
(ιh0 ) = ιh0 fi+1 = ιfi+1
K
(h0 ) = 0,
es decir, ιh0 ∈ ker(fi+1
N
). Se prueba fácilmente que ιi está bien definida.
0
Sean h, h definidas por medio del diagrama (2.2.5) tales que h0 = θ1 (h), entonces
ι1 (h0 ) = ιh0 pero ιh0 = h00 f1 , es decir, ιh0 ∈ Im(f1N ), luego, ι1 (h0 ) = 0. Hemos
probado que Im(θ1 ) ⊆ ker(ι1 ).
Veamos ahora que ker(ι1 ) ⊆ Im(θ1 ): sea h0 ∈ ker(ι1 ), con h0 ∈ ker(f2K ), en-
tonces ι1 (h0 ) = 0 y f2K (h0 ) = 0, luego ιh0 ∈ Im(f1N ) y h0 f2 = 0, existe pues
h00 ∈ HomA (P0 , N ) tal que ιh0 = h00 f1 . Estamos pues en la situación del diagra-
ma (2.2.5), pero falta precisamente definir h : L → M ; sea x ∈ L, entonces existe
u ∈ P0 tal que f0 (u) = x, definimos h(x) := πh00 (u); es fácil probar que h es un ho-
momorfismo bien definido, y además por el diagrama (2.2.5) se tiene que θ1 (h) = h0 ,
es decir, h0 ∈ Im(θ1 ).
Paso 3. Para cada i ≥ 1, Im(ιi ) = ker(πi ): los homomorfismos πi se definen en
forma análoga a como hicimos en (2.2.6):

N π
ker(fi+1 )/Im(fiN ) = ExtiA (L, N ) −
→i
ExtiA (L, M ) = ker(fi+1
M
)/Im(fiM )
(2.2.7)
g 7→ πi (g) := πi (g) = πg
N M
con g ∈ ker(fi+1 ) ⊆ HomA (Pi , N ). Notemos que fi+1 (πg) = πgfi+1 = 0, es decir,
M
πg ∈ ker(fi+1 ). Se prueba fácilmente que πi está bien definida. Veamos que Im(ιi ) ⊆
ker(πi ): πi (ιi (h0 )) = πιh0 = 0.
N
ker(πi ) ⊆ Im(ιi ): sea g ∈ ker(πi ), con g ∈ ker(fi+1 ) ⊆ HomA (Pi , M ); resulta
πi (g) = 0 = πg y también fi+1 (g) = gfi+1 = 0. Tenemos πg ∈ Im(fiM ), luego existe
N

h ∈ HomA (Pi−1 , M ) tal que πg = hfi . Buscamos un homomorfismo h0 ∈ ker(fi+1 K


)⊆
0 0 N
HomA (Pi , K) tal que ιi (h ) = g, es decir, tal que ιh − g ∈ Im(fi ) ⊆ HomA (Pi , N ).
Como Pi−1 es proyectivo existe t ∈ HomA (Pi−1 , N ) tal que πt = h:

Pi−1
p
t pp
pp
p h
pp
?
N π
- M

Notemos que Im(g−tfi ) ⊆ Im(ι): sea (g−tfi )(x) ∈ Im(g−tfi ), luego π(g−tfi )(x) =
πg(x) − πtfi (x) = hfi (x) − hfi (x) = 0, es decir, π(g − tfi )(x) ∈ ker(π) = Im(ι). Se
2.2. PROPIEDADES BÁSICAS 77

tiene entonces el diagrama conmutativo ya que Pi es proyectivo:

Pi
pp
h0 p p
p pp g−tfi
?
K ι
- N

Resulta pues que ιh0 − g = −tfi = fiN (−t) ∈ Im(fiN ).


Paso 4. Para cada i ≥ 1, Im(πi ) = ker(θi+1 ): igual que en los pasos precedentes,
lo primero es definir el homomofismo θi+1 :
θi+1
M
ker(fi+1 )/Im(fiM ) = ExtiA (L, M ) −−→ Exti+1 K K
A (L, K) = ker(fi+2 )/Im(fi+1 )
(2.2.8)
h 7→ θi+1 (h) := h0 ,

donde h0 se define de la siguiente manera: h ∈ ker(fi+1


M
) ⊆ HomA (Pi , M ), debemos
0 K
definir un homomorfismo h ∈ ker(fi+2 ) ⊆ HomA (Pi+1 , K); como Pi es proyectivo
existe h00 que hace el siguiente diagrama conmutativo

Pi
p
h00 p p
pp
p h
pp
?
N π
- M

tenemos además que hfi+1 = 0 luego Im(h00 fi+1 ) ⊆ ker(π) = Im(ι); como Pi+1 es
proyectivo resulta el siguiente diagrama conmutativo

Pi+1
p
p pp
h pp
0
p h00 fi+1
p
pp
?
K - Im(ι)
ι

Notemos que h0 ∈ ker(fi+2 K K


): fi+2 (h0 ) = h0 fi+2 , sea z ∈ Pi+2 , entonces ιh0 fi+2 (z) =
h00 fi+1 fi+2 (z) = 0, pero ι es inyectiva, luego h0 fi+2 (z) = 0, es decir, h0 fi+2 = 0.
Ası́ pues, h0 está definida con las siguientes condiciones: ιh0 = h00 fi+1 y πh00 = h.
Para demostrar que θi+1 está bien definida podemos repetir exactamente la mis-
ma prueba que hicimos en el paso 0, a continuación del diagrama (2.2.5), cuan-
do demostramos que θ1 está bien definida, basta cambiar 1 por i + 1, es decir,
tomando i = 0. Veamos que Im(πi ) ⊆ ker(θi+1 ): sea g como en (2.2.7), tenemos
θi+1 (πi (g)) = θi+1 (πg); puesto que πg = πg y ι0 = gfi+1 , entonces h00 := g y
78 CAPÍTULO 2. EXT

h0 := 0 cumplen las dos condiciones para definir θi+1 , luego θi+1 (πg) = 0, es decir,
θi+1 πi = 0.
M
ker(θi+1 ) ⊆ Im(πi ): sea h ∈ ker(θi+1 ), con h ∈ ker(fi+1 ) ⊆ HomA (Pi , M ),
0 0 K
entonces θi+1 (h) = 0 = h , donde h ∈ ker(fi+2 ) ⊆ HomA (Pi+1 , K), por lo tanto,
h ∈ Im(fi+1 K
) y existe t ∈ HomA (Pi , K) tal que h0 = fi+1 K
(t); resulta h0 = tfi+1 ,
además, por la defincición de θi+1 , existe h ∈ HomA (Pi , N ) tal que πh00 = h y
00

ιh0 = h00 fi+1 . Debemos construir un homomorfismo g ∈ ker(fi+1 K


) ⊆ HomA (Pi , N )
tal que πi (g) = h, luego g debe cumplir las condiciones πg = h y gfi+1 = 0, es
decir, las condiciones πg − h = fiM (u) = ufi con algún u ∈ HomA (Pi−1 , M ) y
gfi+1 = 0. Definimos g := h00 − ιt y se tiene que πg − h = π(h00 − ιt) − h =
πh00 − πιt − h = h − 0 − h = 0 = 0h, luego podemos tomar u := 0; además,
gfi+1 = (h00 − ιt)fi+1 = h00 fi+1 − ιtfi+1 = ιh0 − ιh0 = 0.
Paso 5. Para cada i ≥ 1, Im(θi+1 ) = ker(ιi+1 ): sean h, h0 y h00 con las condi-
ciones del paso 4; entonces, ιi+1 (θi+1 (h)) = ιh0 , pero notemos que ιh0 = fi+1 N
(h00 ) ∈
N
Im(fi+1 ), es decir, ιh0 = 0. Esto demuestra que Im(θi+1 ) ⊆ ker(ιi+1 ).
Para concluir la demostración del teorema veamos que ker(ιi+1 ) ⊆ Im(θi+1 ):
sea h0 ∈ ker(ιi+1 ), con h0 ∈ ker(fi+2 K
) ⊆ HomA (Pi+1 , K), entonces ιi+1 (h0 ) = 0, es
decir, ιh ∈ Im(fi+1 ) y h fi+2 = 0, luego ιh0 = h00 fi+1 , con h00 ∈ HomA (Pi , N ).
0 N 0

Necesitamos un homomorfismo h ∈ ker(fi+1 M


) ⊆ HomA (Pi , M ) tal que πh00 = h; con
esto se cumplirı́an las condiciones del paso 4 y tendrı́amos que θi+1 (h) = h0 , es decir,
h0 ∈ Im(θi+1 ). Definimos h := πh00 , se cumple pues en forma trivial la condición
requerida y además fi+1 M
(πh00 ) = πh00 fi+1 = πιh0 = 0.

Teorema 2.2.2. Sea P un A-módulo. Entonces las siguientes condiciones son equi-
valentes:

(i) P es proyectivo.

(ii) Para cada módulo N y cada i ≥ 1 se tiene que ExtiA (P, N ) = 0.

(iii) Para cada módulo N , Ext1A (P, N ) = 0.

Demostración. (i)⇒(ii): si P es proyectivo, entonces se tiene la resolución proyectiva


0P i
··· → 0 −
→ P −→ P → 0,
con lo cual para cada A-módulo N resulta el complejo
i∗p 0∗
0 → HomA (P, N ) −
→ HomA (P, N ) −
→ HomA (0, N ) → · · · ,
y de esta forma ExtiA (P, N ) = 0, para cada i ≥ 1.
(ii)⇒(iii): evidente.
(iii)⇒(i): consideremos una sucesión exacta
ι π
0→K−
→L−
→ M → 0,
2.2. PROPIEDADES BÁSICAS 79

aplicando el teorema 2.2.1 (ii), se tiene la siguiente sucesión exacta


∗ ι ∗ π 1 θ
0 → HomA (P, K) −
→ HomA (P, L) −→ HomA (P, M ) −
→ Ext1A (P, K) → · · · ,
por lo tanto, la siguiente sucesión es exacta
∗ ι ∗ π
0 → HomA (P, K) −
→ HomA (P, L) −→ HomA (P, M ) → 0
Esto quiere decir que P es proyectivo.
Teorema 2.2.3. Sea N un A-módulo. Entonces las siguientes condiciones son equi-
valentes:
(i) N es inyectivo.
(ii) Para cada módulo M y cada i ≥ 1 se tiene que ExtiA (M, N ) = 0.
(iii) Para cada módulo M , Ext1A (M, N ) = 0.
(iv) Para cada módulo cı́clico M , Ext1A (M, N ) = 0.
Demostración. (i)⇒(ii): para la prueba podrı́amos usar resoluciones inyectivas de
N y proceder como en la demostración del teorema anterior. Sin embargo, hare-
mos la prueba mediante resoluciones proyectivas de M . Existen P proyectivo y un
epimorfismo π : P → M ; sea K := ker(π), entonces tenemos la sucesión exacta
ι π
0→K − →P − → M → 0. Aplicamos el teorema 2.2.1 (i) y obtenemos la sucesión
exacta
π∗ ι∗ 1 θ π
0 → HomA (M, N ) −→ HomA (P, N ) − → HomA (K, N ) − → Ext1A (M, N ) −→
1
f

ιe1 θ2
Ext1A (P, N ) −
→ Ext1A (K, N ) −
→ ···
pero como N es inyectivo, entonces ι∗ es sobreyectivo y como P es proyectivo en-
tonces Ext1A (P, N ) = 0. Se tiene entonces la siguiente porción exacta
π∗ ι∗ 1 θ
0 → HomA (M, N ) −→ HomA (P, N ) − → HomA (K, N ) − → Ext1A (M, N ) → 0,
luego Ext1A (M, N ) ∼
= HomA (K, N )/ ker(θ1 ) = HomA (K, N )/HomA (K, N ) = 0. La
prueba se completa mediante inducción sobre i: supongamos que ExtjA (M, N ) = 0
para cada 1 ≤ j ≤ i y cada A-módulo M, entonces
θi+1 =0 πg
i+1
0 = ExtiA (K, N ) −−−−→ Exti+1 i+1
A (M, N ) −−→ ExtA (P, N ) = 0

es decir, Exti+1
A (M, N ) = 0.
(ii)⇒(iii): evidente.
(iii)⇒(iv): evidente.
(iv)⇒(i): probaremos que N es inyectivo mediante el teorema de Baer (véase el
teorema 1.7.4). Sea I un ideal derecho de A y sea g : I → N un A-homomorfismo
ι j
y consideremos la sucesión exacta 0 → I − → A − → A/I, con j el homomorfismo
canónico. Se tiene entonces la sucesión exacta larga
80 CAPÍTULO 2. EXT

j∗ 1 ι∗ θ π
0 → HomA (A/I, N ) −
→ HomA (A, N ) −
→ HomA (I, N ) −
→ Ext1A (A/I, N ) −→
1 f

Ext1A (A, N ) → · · · ,
pero como A/I es cı́clico, entonces entonces Ext1A (A/I, N ) = 0 y se obtiene la
porción exacta
j∗ ι∗
0 → HomA (A/I, N ) −
→ HomA (A, N ) −
→ HomA (I, N ) → 0,
luego ι∗ es sobreyectivo y por lo tanto existe g 0 : A → N tal que ι∗ (g 0 ) = g, es decir,
g 0 ι = g y ası́ g se puede extender a todo A.
Teorema 2.2.4. Para cada i ≥ 0 se tiene que
Ns ) ∼
M Y Y
ExtiA ( Mr , = ExtiA (Mr , Ns ).
Demostración. Dividimos
Q la demostración en dos pasos.
i ∼ Q i
Paso 1. ExtA (M, s Ns ) = s ExtA (M, Ns ), para cada i ≥ 0. Para i = 0 esta
propiedad corresponde al HomA (véase [16]). Consideremos entonces que i = 1.
Cada módulo Ns se puede sumergir en un módulo inyectivo Es ; sea Ls := Es /Ns ,
resultan entonces las sucesiones exactas
s s ι π
0 → Ns −
→ Es −→ Ls → 0
Q Q
s ιs πs Y
Y Y
0→ Ns −−−→ Es −−s−→ Ls → 0,
s s s
Q
puesto que Es y s Es son inyectivos resultan las sucesiones exactas
(s)
(ιs )∗ (πs )∗ θ1 1
0 → HomA (M, Ns ) −−−−→ HomA (M, Es ) −−−−→ HomA (M, Ls ) −−−→ ExtA (M, Ns ) → 0
( s πs )∗
Q Q
Y ( s ιs )∗ Y Y θ 1
Y
0 → HomA (M, Ns ) −
−−−−−−
→ HomA (M, Es ) −−−−−−−→ HomA (M, Ls ) −
→ ExtA (M, Ns ) → 0
s s s s
Q (s)
θ1
Q Q
Y (ιs )∗ Y (πs )∗ Y Y 1
0→ HomA (M, Ns ) −−−−−→ HomA (M, Es ) −
−−−−−
→ HomA (M, Ls ) −−−−−→ ExtA (M, Ns ) → 0,
s s s s

se tiene entonces el diagrama conmutativo con filas exactas, f, g los isomorfismos


del Hom y h definido por la sobreyectividad de θ
Q
( πs )∗ θ
HomA (M, s Es ) −−−−→ HomA (M, s Ls ) −−−→ Ext1A (M, s Ns ) −−−→ 0
Q Q Q
  
f g 
y y yh
Q Q (s)
(πs )∗ 1 θ
Ext1A (M, Ns ) −−−→ 0
Q Q Q
s HomA (M, Es ) −−−−→ s HomA (M, Ls ) −−−→ s
El lema de los cinco garantiza Qque h es un isomorfismo. Mediante inducción sobre i
y teniendo en cuenta que Es y Es son inyectivos, resulta el diagrama conmutativo
δ
0 −−−→ Exti−1 Ls ) −−−→ ExtiA (M, Ns ) −−−→ 0
Q Q
A (M,
 
β γ
y y
α
0 −−−→ s Exti−1 i
Q Q
A (M, Ls ) −−−→ s ExtA (M, Ns ) −−−→ 0,
2.2. PROPIEDADES BÁSICAS 81

con γ := αβδ −1 . L
Paso 2. ExtiA ( r Mr , N ) ∼
= r ExtiA (Mr , N ), para cada i ≥ 0. Para i = 0
Q
esta es la propiedad correspondiente de HomA . Sea i = 1; para cada Mr existe Pr
proyectivo y una sucesión exacta en la forma
0 → Kr → P r → M r → 0
y a partir de ésta resultan las siguientes suceciones exactas:
L L L
0 → r Kr → r Pr → r Mr → 0,

1
0 → HomA (Mr , N ) → HomA (Pr , N ) → HomA (Kr , N ) → ExtA (Mr , N ) → 0
1
M M M M
0 → HomA ( Mr , N ) → HomA ( Pr , N ) → HomA ( Kr , N ) → ExtA ( Mr , N ) → 0
r r r r
1
Y Y Y Y
0→ HomA (Mr , N ) → HomA (Pr , N ) → HomA (Kr , N ) → ExtA (Mr , N ) → 0,
r r r r

como en el paso anterior, se tiene entonces el diagrama conmutativo con filas exactas,
f 0 , g 0 los isomorfismos del Hom y h0 definido por sobreyectividad:
HomA ( r Pr , N ) −−−→ HomA ( r Kr , N ) −−−→ Ext1A ( r Mr , N ) −−−→ 0
L L L
  
f 0 g0  0
y y yh
1
Q Q Q
r HomA (Pr , N ) −−−→ r HomA (Kr , N ) −−−→ r ExtA (Mr , N ) −−−→ 0

El lema de los cinco garantiza que h0 es un isomorfismo. El resto de la demostración


se hace como en el paso anterior mediante inducción.
Concluimos esta sección con el siguiente teorema relacionado con la proposición
1.8.11.
Teorema 2.2.5. Sean R un anillo conmutativo noetheriano y S un sistema multi-
plicativo de R. Entonces, para cada i ≥ 0 y cualesquiera R-módulos M, N , con M
f.g., se tiene el RS −1 -isomorfismo
ExtiR (M, N )S −1 ∼
= ExtiRS −1 (M S −1 , N S −1 ).
Demostración. La demostración se puede realizar por inducción sobre i. Para i = 0
el resultado corresponde a la proposición 1.8.11 ya que al ser M f.g. sobre el anillo
noetheriano R, entonces M es de presentación finita.
Veamos ahora la prueba para i = 1. Usando nuevamente que M es f.g. y que R
es noetheriano se tiene la sucesión exacta de R-módulos
0→K→F →M →0
con F libre de bases finitas y K f.g. Puesto que el funtor S −1 es exacto, entonces
resulta la sucesión exacta de RS −1 -módulos
0 → KS −1 → F S −1 → M S −1 → 0;
82 CAPÍTULO 2. EXT

nótese que F S −1 es RS −1 -libre. Estas dos sucesiones exactas inducen las siguientes
sucesiones exactas largas
1 θ
0 → HomR (M, N ) → HomR (F, N ) → HomR (K, N ) −
→ Ext1R (M, N ) → · · · ,
0 → HomRS −1 (M S −1 , N S −1 ) → HomRS −1 (F S −1 , N S −1 ) →
HomRS −1 (KS −1 , N S −1 ) → Ext1RS −1 (M S −1 , N S −1 ) → · · · .
Nuevamente, como S −1 es exacto, de la primera se obtiene la siguiente sucesión
exacta de RS −1 -módulos
θ S −1
0 → HomR (M, N )S −1 → HomR (F, N )S −1 → HomR (K, N )S −1 −−
1
−→
Ext1R (M, N )S −1 → · · · .
Teniendo en cuenta que F y F S −1 son libres, extraemos de las dos sucesiones ante-
riores las siguientes porciones exactas, y donde el diagrama resulta conmutativo:
θ1 S −1
HomR (F, N )S −1 −−−−−−→ HomR (K, N )S −1 −−−−−−→ Ext1
R (M, N )S
−1
−−−−−−→ 0
  
g
yf
 
y yh

HomRS −1 (F S −1 , N S −1 ) −−−−−−→ HomRS −1 (KS −1 , N S −1 ) −−−−−−→ Ext1


R (M, N )S
−1
−−−−−−→ 0

f, g son los isomorfismos de la proposición 1.8.11 (nótese que F, K son de pre-


sentación finita) y h se define mediante la sobreyectividad de θ1 S −1 . Puesto que f, g
son isomorfismos, entonces h es también un isomorfismo.
Para completar la demostración, de las sucesiones exactas largas se tiene el dia-
grama

0 −−−→ ExtiR (K, N )S −1 −−−→ Exti+1


R (M, N )S
−1
−−−→ 0
 
 
y y
0 −−−→ ExtiRS −1 (KS −1 , N S −1 ) −−−→ Exti+1
RS −1 (M S
−1
, N S −1 ) −−−→ 0

donde el isomorfismo vertical de la izquierda es por el paso de inducción y el isomor-


fismo vertical de la derecha es por construcción invirtiendo isomorfismo horizontal
superior y componiendo con los otros dos isomorfismos.

2.3. Ejemplos
Presentaremos a continuación algunos casos particulares encaminados a calcular
ExtiZ (M, N ), donde M es un grupo abeliano f.g. y N es un grupo abeliano arbitrario.
Ejemplo 2.3.1. Si F es un A-módulo libre, entonces ExtiA (F, N ) = 0 para cada
i ≥ 1 y cada A-módulo N . En particular,
ExtiA (A, N ) = 0, para cada i ≥ 1 y cada A-módulo N .
2.3. EJEMPLOS 83

Ejemplo 2.3.2. Sea A un anillo hereditario a derecha, entonces ExtiA (M, N ) = 0


para cada i ≥ 2 y cualesquiera módulos M, N . En efecto, para M existe un A-
f1 f0
módulo proyectivo P y se tiene la resolución proyectiva 0 → ker(π) − → P − →
M → 0 (véase el teorema 1.9.21); aplicamos HomA ( , N ) y obtenemos 0 →
f∗ f∗
HomA (M, N ) −→0
HomA (P, N ) −→ 1
HomA (ker(π), N ) → 0 → · · · , luego fi∗ = 0
para i ≥ 2 y entonces ExtiA (M, N ) = 0 para i ≥ 2. Notemos que Ext1A (M, N ) =
HomA (ker(π), N )/Im(f1∗ ), pero no podemos conlucir nada especial en el caso i = 1.
En particular,

ExtiZ (M, N ) = 0, para i ≥ 2 y cualesquiera grupos abelianos M, N .

por ejemplo,

ExtiZ (Zm , Zn ) = 0 para cada i ≥ 2 y cualesquiera m, n ≥ 0,


ExtiZ (R, R) = ExtiZ (R, C) = ExtiZ (C, C) = 0, para i ≥ 2.

De otra parte, puesto que Q es Z-inyectivo (corolario 1.7.6), entonces ExtiZ (M, Q) =
0 para i ≥ 1 y cada grupo abeliano M , y de manera particular,

ExtiZ (Q, Q) = ExtiZ (R, Q) = ExtiZ (C, Q) = 0, para i ≥ 1.

Ejemplo 2.3.3. El ejemplo anterior deja planteado el siguiente problema: calcular


Ext1Z (M, N ), con M, N grupos abelianos arbitrarios. Veamos entonces una propiedad
que ayudará a resolver parcialmente el problema. Para A un anillo arbitrario, N un
A-módulo cualquiera e i ≥ 1, calculemos ExtiA (A/{xi, N ).
Si x = 0, entonces ExtiA (A/{xi, N ) = ExtiA (A, N ) = 0; para x = 1 se tiene que
ExtiA (A/{xi, N ) = ExtiA (0, N ) = 0. Sea x 6= 0, 1 tal que x no es divisor de cero;
f1 f0
consideremos la resolución libre 0 → A −
→ A −
→ A/{xi → 0, f0 es el homomor-
0 f∗
fismo canónico y f1 (a) := x · a, se obtiene el complejo 0 → HomA (A/{xi, N ) −→
1 f∗
HomA (A, N ) −→ HomA (A, N ) → 0 → · · · , con lo cual ExtiA (A/{xi, N ) = 0 para
i ≥ 2. Veamos el caso i = 1:

Ext1A (A/{xi, N ) = HomA (A, N )/Im(f1∗ ) ∼


= N/(N · x)
ya que HomA (A, N ) ∼ = N e Im(f1∗ ) ∼ = N · x. Veamos este último isomorfismo:

sea g ∈ Im(f1 ), entonces existe h ∈ HomA (A, N ) tal que g = hf1 , definimos
α : Im(f1∗ ) → N por α(g) := hf1 (1). La función α está bien definida ya que si
g = hf1 = h0 f1 , entonces hf1 (1) = h0 f1 (1); α es claramente aditiva; sea a ∈ A,
entonces g · a = (hf1 ) · a, luego [(hf1 ) · a](1) = (hf1 (1)) · a, es decir, α(g · a) = α(g) · a.
Además, α es inyectiva ya que si α(g) = hf1 (1) = 0, entonces g(a) = (hf1 )(a) =
(hf1 )(1 · a) = (hf1 )(1) · a = 0 para cada a ∈ A, es decir, g = 0. Por último, notemos
84 CAPÍTULO 2. EXT

que Im(α) = N · x: en efecto, α(g) := hf1 (1) = h(x) = h(1 · x) = h(1) · x ∈ N · x;


recı́procamente, sea n · x ∈ N · x, entonces h ∈ HomA (A, N ) definida por h(1) := n
es tal que hf1 (1) = n · x, luego g := hf1 es tal que α(g) = n · x.
Del resultado probado obtenemos en particular que para cada grupo abeliano N ,

Ext1Z (Zm , N ) ∼
= N/(N · m), m ≥ 2.

Ejemplo 2.3.4. Con los resultados de los ejemplos anteriores podemos ya calcular
Ext1Z (M, N ), con M un grupo abeliano f.g. y N un grupo abeliano arbitrario: en
efecto, M es de la forma M = Zr ⊕ Zm1 ⊕ · · · ⊕ Zmt , con r ≥ 0 (el rango de M )
y mj ≥ 2, 1 ≤ j ≤ t (sus factores invariantes, véase [16]). Con el teorema 2.2.4
podemos concluir que

Ext1Z (Zr ⊕ Zm1 ⊕ · · · ⊕ Zmt , N ) ∼


= N/(N · m1 ) ⊕ N/(N · mt ), mj ≥ 2, 1 ≤ j ≤ t.

En particular, puesto que Zn /Zn · m ∼


= Zd , con d := m.c.d.(m, n), resulta
(
0, m = 0, 1; n ≥ 0
Ext1Z (Zm , Zn ) =
Zd , m ≥ 2; n ≥ 0; d := m.c.d.(m, n).

Este ejemplo también permite ilustrar que ExtiA (M, N )  ExtiA (N, M ). Por ejem-
plo, Ext1Z (Z, Zn ) = 0 pero Ext1Z (Zn , Z) ∼
= Zn .

Otros casos particulares interesantes son los siguientes.

Ejemplo 2.3.5. Sea A un anillo hereditario a derecha y sea P un A-módulo. P es


proyectivo si, y sólo si, Ext1A (P, F ) = 0 para cada A-módulo libre F . En efecto, si P
es proyectivo, entonces por el teorema 2.2.2, Ext1A (P, F ) = 0 para cada A-módulo
libre F . Para demostrar la afirmación recı́proca, consideremos el diagrama

P
g

?
M f
- N

con f sobreyectivo; sea F libre y π : F → M sobreyectivo; sea K := ker(f π). Se



tiene entonces la sucesión exacta 0 → K → F −→ N → 0, y de aquı́, la sucesión
exacta larga

(f π)∗
0 → HomA (P, K) → HomA (P, F ) −−−→ HomA (P, N ) → Ext1A (P, K) → · · ·
2.4. EJERCICIOS 85

como F es proyectivo y A es hereditario a derecha, entonces K es proyectivo (teore-


ma 1.9.21) y existen K 0 y F1 libre tales que K ⊕ K 0 = F1 , luego 0 = Ext1A (P, F1 ) =
Ext1A (P, K) ⊕ Ext1A (P, K 0 ), de donde Ext1A (P, K) = 0. Se obtiene entonces la suce-
(f π)∗
sión exacta 0 → HomA (P, K) → HomA (P, F ) −−−→ HomA (P, N ) → 0; en conse-
cuencia, existe h0 ∈ HomA (P, F ) tal que f πh0 = g, es decir, el siguiente diagrama
es conmutativo, con h := πh0
P
h
g

?
M f
- N

Esto demuestra que P es proyectivo.


Ejemplo 2.3.6. Sea R un DD y sea Q(R) su cuerpo de fracciones. Sea K :=
Q(R)/R. Entonces, un R-módulo N es inyectivo si, y sólo si, Ext1R (K, N ) = 0. En
efecto, si N es inyectivo, entonces por el teorema 2.2.3, Ext1R (K, N ) = 0. Recı́pro-
camente, consideremos la sucesión exacta 0 → R → Q(R) → K → 0, entonces de la
suceción exacta larga resulta la sucesión exacta
0 → HomR (K, N ) → HomR (Q(R), N ) → HomR (R, N ) → Ext1R (K, N ) = 0,
por lo tanto, N ∼ = HomR (Q(R), N )/HomR (K, N ) es una imagen homomorfa de
HomR (Q(R), N ), el cual es R-divisible por ser un Q(R)-espacio vectorial (véase la
proposición 1.7.10 y el corolario 1.7.13), luego N es divisible. El corolario 1.9.22
garantiza que N es inyectivo.

2.4. Ejercicios
1. Calcule:
(i) Ext1A (Mn (A), N ), donde A es un anillo, Mn (A) es el anillo de matrices
cuadradas de tamaño n × n, n ≥ 1, y N es un A-módulo derecho.
(ii) Ext1A (M, Mn (A)), donde A es un anillo tal que AA es inyectivo y M es
un A-módulo derecho.
2. Realice la demostración de la parte (i) del teorema 2.2.1.
3. Realice la demostración del teorema 2.2.5.
4. Demuestre la siguiente generalización del teorema 2.2.5. Sean R un anillo con-
mutativo noetheriano y B una R-álgebra conmutativa plana. Entonces, para
cada i ≥ 0 y cualesquiera R-módulos M y N , con M f.g., se tiene el B-
isomorfismo
86 CAPÍTULO 2. EXT

ExtiR (M, N ) ⊗R B ∼
= ExtiB (M ⊗R B, N ⊗R B).

5. Dos módulos M y N son proyectivamente equivalentes si existen módulos


proyectivos P, Q tales que M ⊕ P ∼
= N ⊕ Q. De manera análoga se definen los
módulos son inyectivamente equivalentes. Demuestre que estas relaciones
son de equivalencia.

6. Sean M, N módulos proyectivamente equivalentes. Demuestre que para cada


módulo L y n ≥ 1, ExtnA (M, L) ∼
= ExtnA (N, L).

7. Sean
fi+1 fi f1 f0
· · · → Pi+1 −−→ Pi −
→ Pi−1 → · · · → P1 −
→ P0 −
→ M → 0.
gi+1 gi g1 g0
· · · → Qi+1 −−→ Qi −
→ Qi−1 → · · · → Q1 −
→ Q0 −
→ M → 0.
dos resoluciones proyectivas del módulo M . Sean Kn := ker(fn ), Kn0 := ker(gn ),
n ≥ 0. Demuestre que Kn y Kn0 son proyectivamente equivalentes (sugerencia:
utilice el lema 1.2.14).

8. Enuncie y demuestre el ejercicio anterior para resoluciones inyectivas.

9. Muestre que las propiedades de los dos ejercicios anteriores no son siempre
ciertas para resoluciones planas (sugerencia: considere el Z-módulo Q/Z y
utilice el teorema 1.9.21).

10. Sean M, N módulos inyectivamente equivalentes. Demuestre que para cada


módulo L y n ≥ 1, ExtnA (L, M ) ∼
= ExtnA (L, N ).
ι π
11. Sea 0 −→ K − →N − → M −→ 0 una sucesión exacta hendida de A-módulos.
Sea L un A-módulo. Demuestre que para cada i ≥ 0 las siguientes sucesiones
son exactas hendidas:
ι π
0 −→ ExtiA (L, K) −→
i∗
ExtiA (L, N ) −→
i∗
ExtiA (L, M ) −→ 0,

π∗ ι∗
0 −→ ExtiA (M, L) −→
i
ExtiA (N, L) −
→i
ExtiA (K, L) −→ 0.
Capı́tulo 3

Tor

Estudiamos ahora la extensión del producto tensorial y algunas propiedades básicas


similares a las consideradas en el capı́tulo anterior para el Ext. Recordemos que A
representa un anillo no necesariamente conmutativo.

3.1. Definición y propiedades básicas


Sea N un A-módulo a izquierda y consideremos una resolución proyectiva de N en
la siguiente forma
fi+1 fi f1 f0
· · · → Pi+1 −−→ Pi −
→ Pi−1 → · · · → P1 −
→ P0 −
→ N → 0; (3.1.1)
sea ahora M un A-módulo derecho, al tensorizar por M se obtiene el complejo
iM ⊗fi+1 iM ⊗f1 iM ⊗f0
· · · → M ⊗A Pi+1 −−−−−−→ M ⊗A Pi → · · · → M ⊗A P1 −−−−→ M ⊗A P0 −−−−→ M ⊗A N → 0.

Esta sucesión induce la siguiente definición.


Definición 3.1.1. Sea N un A-módulo a izquierda y sea M un A-módulo a derecha.
Para cada i ≥ 1 se define

T oriA (M, N ) := ker(iM ⊗ fi )/Im(iM ⊗ fi+1 ), (3.1.2)

y
T or0A (M, N ) := M ⊗A N .
La definición anterior parece depender de la resolución proyectiva elegida para
N . Sin embargo, se tiene la siguiente propiedad.
Teorema 3.1.2. Para cada i ≥ 0, el grupo T oriA (M, N ) es independiente de la
resolución proyectiva (3.1.1).

87
88 CAPÍTULO 3. TOR

Demostración. La demostración es similar a la realizada para el funtor Ext (teorema


2.1.2) y la dejamos como ejercicio al lector.
Según el teorema anterior, para definir y calcular los grupos T oriA (M, N ) pode-
mos utilizar resoluciones libres en lugar de resoluciones proyectivas. Es decir, con
una resolución libre para N basta para calcular T oriA (M, N ) para cada i ≥ 0. Pre-
sentamos la interpretación funtorial del T or.
Corolario 3.1.3. Sea M un A-módulo derecho. Entonces, para cada i ≥ 0
T orA (M, )
A M od −−−i−−−−→ Ab
N 7−→ T oriA (M, N )
α i ⊗α
→ N 0 7−→ T oriA (M, N ) −−
N− M i
−−→ T oriA (M, N 0 )

es un funtor covariante, donde A M od denota la categorı́a de los A-módulos a izquier-


da y Ab denota la categorı́a de los grupos abelianos.
Demostración. La prueba completa la dejamos al lector; veamos solamente cómo se
define el homomorfismo iM ⊗ αi . Construimos para N y N 0 resoluciones proyectivas,
calculamos los homomorfismos αi como en la demostración del teorema 2.1.2, luego
tensorizamos por M y obtenemos el diagrama conmutativo de complejos
iM ⊗fi iM ⊗f1 iM ⊗f0
(P ) : · · · −−−−−−→ M ⊗ Pi −−−−−−→ · · · −−−−−−→ M ⊗ P1 −−−−−−→ M ⊗ P0 −−−−−−→ M ⊗ N −−−−−−→ 0
   
iM ⊗αi y iM ⊗α1 y iM ⊗α0 y iM ⊗αy
   

iM ⊗gi iM ⊗g1 iM ⊗g0


(Q) : · · · −−−−−−→ M ⊗ Qi −−−−−−→ · · · −−−−−−→ M ⊗ Q1 −−−−−−→ M ⊗ Q0 −−−−−−→ M ⊗ N 0 −−−−−−→ 0

Se define entonces
i ⊗α
T oriA (M, N ) = ker(iM ⊗ fi )/Im(iM ⊗ fi+1 ) −−
M i
−−→ T oriA (M, N 0 ) = ker(iM ⊗ gi )/Im(iM ⊗ gi+1 )
z 7→ (iM ⊗ αi )(z)

con z ∈ ker(iM ⊗ fi ). Es claro que (iM ⊗ αi )(z) ∈ ker(iM ⊗ gi ) y se puede probar


fácilmente que iM ⊗ αi está bien definida.
Observación 3.1.4. (i) Si M es un B − A bimódulo, entonces T oriA (M, N ) es un
B-módulo, para cada i ≥ 0. En particular, si R es un anillo conmutativo y M, N
son R-módulos, entonces T oriA (M, N ) es un R-módulo, para cada i ≥ 0.
(ii) Si Aop es el opuesto del anillo A (véase [16]), entonces para cada i ≥
0, T oriA (M, N ) ∼
= T oriA (N, M ). Si R es un anillo conmutativo, T oriR (M, N ) ∼
op
=
T oriR (N, M ).
(iii) Los grupos T or se pueden construir también por resoluciones proyectivas
del primer argumento y se puede demostrar que estos grupos resultan isomorfos a
los definidos en (3.1.2).
(iv) Para N fijo, T oriA ( , N ) es también un funtor covariante.
3.1. DEFINICIÓN Y PROPIEDADES BÁSICAS 89

Corolario 3.1.5. Si P es un A-módulo proyectivo, entonces para cada i ≥ 1,


T oriA (P, N ) = 0 = T oriA (M, P ), para cada A-módulo izquierdo N y cada A-módulo
derecho M . En particular,

T oriA (A, N ) = 0 = T oriA (M, A).

Demostración. Evidente al hacer la resolcuión proyectiva 0 → P → P → 0.

Teorema 3.1.6. (i) Sea L un A-módulo a izquierda, entonces la sucesión exacta de


A-módulos a derecha
ι π
0→K−
→N −
→M →0

induce la sucesión exacta de grupos abelianos

A ιi+1 ⊗iL A πi+1 ⊗iL


A θi+1
· · · →T ori+1 (K, L) −−−−→ T ori+1 (N, L) −−−−−→ T ori+1 (M, L) −−→
ι ⊗i π ⊗i θ
T oriA (K, L) −−
i L
−→ T oriA (N, L) −−
i L
−→ T oriA (M, L) −
→i
···
ι ⊗i π ⊗i θ
T or1A (K, L) −−
1 L
−→ T or1A (N, L) −−
1 L
−→ T or1A (M, L) −
→1

ι⊗i π⊗i
T or0A (K, L) −−→
L
T or0A (N, L) −−−→
L
T or0A (M, L) → 0.

(ii) Sea L un A-módulo a derecha, entonces la sucesión exacta de A-módulos a


izquierda
ι π
0→K−
→N −
→M →0

induce la sucesión exacta de grupos abelianos

A iL ⊗ιi+1 A iL ⊗πi+1
A θi+1
· · · →T ori+1 (L, K) −−−−→ T ori+1 (L, N ) −−−−−→ T ori+1 (L, M ) −−→
i ⊗ι i ⊗π θ
T oriA (L, K) −−
L i
−→ T oriA (L, N ) −L−−→
i
T oriA (L, M ) −
→i
···
i ⊗ι i ⊗π θ
T or1A (L, K) −−
L 1
−→ T or1A (L, N ) −L−−→
1
T or1A (L, M ) −
→1

i ⊗ι i ⊗π
T or0R (L, K) −L−→ T or0R (L, N ) −−
L
−→ T or0R (L, M ) → 0.

Demostración. La prueba completa la dejamos al lector, veamos solamente como se


define θi+1 en (i): consideremos una resolución proyectiva de L

fi+1 fi f1 f0
· · · → Pi+1 −−→ Pi −
→ Pi−1 → · · · → P1 −
→ P0 −
→ L → 0;
90 CAPÍTULO 3. TOR

tenzorizando por K, N y M se obtiene el siguiente diagrama conmutativo:


0 0
 
 
y y
iK ⊗fi+2 iK ⊗fi+1
K i ⊗fi
· · · −−−−−→ K ⊗ Pi+1 −−−−−→ K ⊗ Pi −− −→ · · ·
 
ι⊗iPi+1 y ι⊗iPi y
 

iN ⊗fi+2 iN ⊗fi+1
N i ⊗fi
· · · −−−−−→ N ⊗ Pi+1 −−−−−→ N ⊗ Pi −− −→ · · ·
 
π⊗iPi+1 y π⊗iPi y
 

iM ⊗fi+2 iM ⊗fi+1 i ⊗fi


· · · −−−−−→ M ⊗ Pi+1 −−−−−→ M ⊗ Pi −M
−−→ · · ·
 
 
y y
0 0
las filas son complejos y las columnas son exactas ya que cada Pi es plano. Se tiene
entonces que
A θi+1
T ori+1 (M, L) = ker(iM ⊗ fi+1 )/Im(iM ⊗ fi+2 ) −−−→ T oriA (K, L) = ker(iK ⊗ fi )/Im(iK ⊗ fi+1 )
z 7→ z 0

con z 0 definido de la siguiente manera: como z ∈ ker(iM ⊗fi+1 ) ⊆ M ⊗Pi+1 , entonces


existe z 00 ∈ N ⊗Pi+1 tal que (π⊗iPi+1 )(z 00 ) = z; tenemos que (iN ⊗fi+1 )(z 00 ) ∈ N ⊗Pi ,
pero (π ⊗ iPi )(iN ⊗ fi+1 )(z 00 ) = (iM ⊗ fi+1 )(π ⊗ iPi+1 )(z 00 ) = (iM ⊗ fi+1 )(z) = 0, luego
(iN ⊗ fi+1 )(z 00 ) ∈ ker(π ⊗ iPi ) = Im(ι ⊗ iPi ), por lo tanto, existe z 0 ∈ K ⊗ Pi
tal que (iN ⊗ fi+1 )(z 00 ) = (ι ⊗ iPi )(z 0 ). Notemos que z 0 ∈ ker(iK ⊗ fi ): en efecto,
(ι ⊗ iPi−1 )(iK ⊗ fi )(z 0 ) = (iN ⊗ fi )(ι ⊗ iPi )(z 0 ) = (iN ⊗ fi )(iN ⊗ fi+1 )(z 00 ) = 0, pero
ι ⊗ iPi−1 es inyectiva, luego se obtiene lo afirmado. Se puede demostrar que θi+1
queda bien definida y sujeta entonces a las siguientes condiciones
(π ⊗ iPi+1 )(z 00 ) = z, (iN ⊗ fi+1 )(z 00 ) = (ι ⊗ iPi )(z 0 ).

Teorema 3.1.7. Para cada i ≥ 0 se tiene que

Ns ) ∼
M M M
T oriA ( Mr , = T oriA (Mr , Ns ).

Demostración. Ideas similares a las de la demostración del teorema 2.2.4 pueden ser
usadas en este caso. La prueba se deja al lector (véase también [33], teoremas 6.3.8
y 6.3.9).
Terminamos esta sección con la siguiente generalización de la proposición 1.6.8.
3.2. EJEMPLOS 91

Teorema 3.1.8. Sean R un anillo conmutativo y S un sistema multiplicativo de


R. Entonces, para cada i ≥ 0 y cualesquiera R-módulos M, N se tiene el RS −1 -
isomorfismo
T oriR (M, N )S −1 ∼
−1
= T oriRS (M S −1 , N S −1 ).

Demostración. El caso i = 0 fue probado en la proposición 1.6.8.


Sea i ≥ 1, entonces T oriA (M, N ) = ker(iM ⊗ fi )/Im(iM ⊗ fi+1 ), con
iM ⊗fi+1 i ⊗fi
M ⊗ Pi+1 −−−−−→ M ⊗ Pi −M
−−→ M ⊗ Pi−1 .
Por lo tanto,

T oriA (M, N )S −1 = [ker(iM ⊗ fi )/Im(iM ⊗ fi+1 )]S −1



= [ker(iM ⊗ fi )]S −1 /[Im(iM ⊗ fi+1 )]S −1
= ker[(iM ⊗ fi )S −1 ]/Im[(iM ⊗ fi+1 )S −1 ],

pero se tiene el siguiente diagrama conmutativo


(iM ⊗fi )S −1
(M ⊗ Pi )S −1 −−−−−−−→ (M ⊗ Pi−1 )S −1
 
 
y y
−1 )
(i −1 ⊗fi S
M S −1 ⊗ Pi S −1 −−M−S−−−−−−→ M S −1 ⊗ Pi−1 S −1

donde las flechas verticales son isomorfismos (véase la proposición 1.6.8). En efecto,
se tiene que
m⊗p
s
−−−→ m⊗fsi (p)
 
 
y y
m p m fi (p)
s
⊗ 1
−−−→ s
⊗ 1

El diagrama conmutativo implica que ker[(iM ⊗ fi )S −1 ] ∼ = ker[(iM S −1 ⊗ fi S −1 )] y de


igual manera Im[(iM ⊗ fi+1 )S −1 ] ∼
= Im[(iM S −1 ⊗ fi+1 S −1 )]. Esto completa la prueba
del teorema.

3.2. Ejemplos
Ejemplo 3.2.1. Sea A un anillo hereditario a derecha y sean M un A-módulo
derecho y N un A-módulo izquierdo. Entonces, T oriA (M, N ) = 0 para cada i ≥ 2.
f1 f0
En efecto, consideremos la resolución proyectiva de N , 0 −
→K − → P → N → 0,
entonces iM ⊗ fi = 0 para i ≥ 2, luego T oriA (M, N ) = 0, para cada i ≥ 2 y cada
A-módulo N . En particular,
92 CAPÍTULO 3. TOR

T oriZ (M, N ) = 0, para i ≥ 2 y cualesquiera grupos abelianos M, N .

Por ejemplo,

T oriZ (Zm , Zn ) = 0, para i ≥ 2 y cualesquiera m, n ≥ 0,


T oriZ (R, R) = T oriZ (R, C) = T oriZ (C, C) = 0, para i ≥ 2.

Ejemplo 3.2.2. El ejemplo anterior deja planteado el siguiente problema: calcular


T or1Z (M, N ), con M, N grupos abelianos arbitrarios. Veamos entonces una propiedad
que ayudará a resolver parcialmente el problema. Para A un anillo arbitrario, M
un A-módulo derecho cualquiera, calculemos T oriA (M, A/hx}) para i ≥ 0. Si x = 0,
T oriA (M, A/hx}) = T oriA (M, A) = 0 para i ≥ 1, y T or0A (M, A/hx}) = M ⊗A A ∼= M.
Si x = 1, T oriA (M, A/hx}) = T oriA (M, 0) = 0 para cada i ≥ 0. Sea x 6= 0, 1 tal que
x no es divisor de cero;

T or0A (M, A/hx}) = M ⊗A A/hx} ∼


= M/(M hx}) = M/M x.
f1 f0
Para i ≥ 1, consideremos la resolución proyectiva 0 → A − → A − → A/hx} →
0, con f1 (a) := ax y f0 el homomorfismo canónico en el cociente. Para i ≥ 2,
fi = 0, luego T oriA (M, A/hx}) = 0. Resta pues calcular T or1A (M, A/hx}); tenemos
T or1A (M, A/hx}) = ker(iM ⊗f1 )/Im(iM ⊗f2 ) = ker(iM ⊗f1 ), pero se tiene el siguiente
diagrama conmutativo
iM ⊗f1
M ⊗A A −− −−→ M ⊗A A
 
 
y y
g
M −−−→ M
donde las flechas verticales son los isomorfismos naturales, luego g(m) := m · x.
Resulta, ker(iM ⊗ f1 ) = ker(g) = {m ∈ M |m · x = 0} y entonces

T or1A (M, A/hx}) = {m ∈ M |m · x = 0}.

Resultados análogos a los anteriores se tienen por supuesto para T oriA (A/{xi, M )
con i ≥ 0 y M un A-módulo izquierdo arbitrario.
En particular, para cada grupo abeliano M se tiene que
(
M/M · n, i = 0, n ≥ 2
T oriZ (M, Zn ) =
{m ∈ M |m · n = 0}, i = 1, n ≥ 2.

Por ejemplo,

T or0Z (Zm , Zn ) = Zm ⊗ Zn = Zm /Zm · n = Zd , m, n ≥ 2, con d := m.c.d.(m, n).

El siguiente ejemplo presenta otra propiedad interesante del T or.


3.2. EJEMPLOS 93

Ejemplo 3.2.3. Sean A un anillo, I un ideal derecho y J un ideal izquierdo de A.


Entonces se tiene el siguiente isomorfimso de grupos abelainos:
T or1A (A/I, A/J) ∼
= (I ∩ J)/IJ.
f2 f1 π
Consideremos la siguiente resolución libre de A/J: · · · → F2 −
→ F1 −→A− → A/J →
0, con Fi libre para i ≥ 1 y π el homomorfismo canónico en el cociente; tensorizando
iA/I ⊗f2 iA/I ⊗f1
por A/I obtenemos el complejo · · · → A/I ⊗A F2 −−−−→ A/I ⊗A F1 −−−−→ A/I ⊗A
iA/I ⊗π
A −−−−→ A/I ⊗A A/J → 0, luego T or1A (A/I, A/J) = ker(iA/I ⊗ f1 )/Im(iA/I ⊗ f2 ).
Se tiene el diagrama conmutativo
iA/I ⊗f2 iA/I ⊗f1
A/I ⊗A F2 −−−−→ A/I ⊗A F1 −−−−→ A/I ⊗A A
  
  
y y y
g2 g1
F2 /IF2 −−−→ F1 /IF1 −−−→ A/I

donde las flechas verticales son los isomorfismos naturales, g2 (x) := f2 (x), x ∈ F2 y
g1 (y) := f1 (y), y ∈ F1 . Se obtiene entonces que T or1A (A/I, A/J) ∼
= ker(g1 )/Im(g2 )
y definimos la aplicación
α
ker(g1 ) −
→ (I ∩ J)/IJ, α(y) := f1 (y).
Notemos que α está bien definido: en efecto, si y ∈ ker(g1 ), entonces f1 (y) = 0, de
donde f1 (y) ∈ Im(f1 ) = J y f1 (y) ∈ I, es decir, f1 (y) ∈ I ∩ J; además, si y = 0,
entonces y ∈ IF1 , luego y = a1 · y1 + · · · + at · yt , con ai ∈ I y yi ∈ F1 , 1 ≤ i ≤ t, por
lo tanto, f1 (y) = a1 · f1 (y1 ) + · · · + at · f1 (yt ) ∈ IJ, es decir, fi (y) = 0.
Es claro que α es un homomorfismo de grupos, además, α es sobreyectivo: en
efecto, sea z ∈ (I ∩ J)/IJ con z ∈ I ∩ J, entonces z ∈ I y z ∈ J = Im(f1 ), de donde
z = f1 (y), con y ∈ F1 , luego y ∈ ker(g1 ) y α(y) = f1 (y) = z.
Veamos ahora que ker(α) = Im(g2 ): sea y ∈ Im(g2 ), entonces y = g2 (x), con
x ∈ F2 , se tiene entonces que α(y) = α(g2 (x)) = α(f2 (x)) = f1 (f2 (x)) = 0, es
decir, y ∈ ker(α), hemos pues probado que Im(g2 ) ⊆ ker(α). Sea ahora y ∈ ker(α),
con y ∈ F1 , entonces α(y) = f1 (y) = 0, de donde f1 (y) ∈ IJ; resulta entonces
f1 (y) = a1 z1 + · · · + at zt , con ai ∈ I y zi ∈ J = Im(f1 ), 1 ≤ i ≤ t; luego obtenemos
que y − (a1 · y1 + · · · + at · yt ) ∈ ker(f1 ) = Im(f2 ), con yi ∈ F1 , luego y = a1 · y1 +
· · · + at · yt + f2 (x), con x ∈ F2 , y esto indica que y = f2 (x) = g2 (x) ∈ Im(g2 ).
Por ejemplo,
(
0, m = 0, 1 o n = 0, 1
T or1Z (Zm , Zn ) =
Zd , m ≥ 2, n ≥ 2, con d := m.c.d.(m, n).
La última igualdad se justifica de la siguiente manera: (hmi ∩ hni)/(hmihni) =
hm.c.m(m, n)i/hmni ∼= Zd .
94 CAPÍTULO 3. TOR

Ejemplo 3.2.4. Con los resultados de los ejemplos anteriores podemos calcular
T or0Z (M, N ) y T or1Z (M, N ), con M, N grupos abelianos f.g. Por ejemplo,

(Z2 ⊕ Z6 ⊕ Z10 ) ⊗Z (Z12 ⊕ Z18 ) ∼


= (Z12 )2 ⊕ (Z18 )2 ⊕ Z26 ⊕ Z22 ∼
= Z62 ⊕ Z24 ⊕ Z43 ⊕ Z29 ,
T or1Z (Z2 ⊕ Z6 ⊕ Z10 , Z12 ⊕ Z18 ) ∼
= Z26 ⊕ Z22 ∼
= Z42 ⊕ Z23 .

3.3. Tor y módulos planos


Presentamos ahora otras caracterizaciones de los módulos planos por medio del T or.

Teorema 3.3.1. Sea F un A-módulo derecho. Entonces las siguientes condiciones


son equivalentes:

(i) F es plano.

(ii) Para cada A-módulo N y cada i ≥ 1, T oriA (F, N ) = 0.

(iii) Para cada A-módulo N , T or1A (F, N ) = 0.

(iv) T or1A (F, A/I) = 0, para cada ideal izquierdo I de A.

(v) T or1A (F, A/I) = 0, para cada ideal izquierdo f.g. I de A.

El teorema es también válido en el segundo argumento.

Demostración. (i)⇒(ii): considerando una resolución proyectiva de N y tensorizan-


do por F resulta el complejo
iF ⊗fi+1 F i ⊗fi i ⊗f0
· · · → F ⊗ Pi+1 −−−−−→ F ⊗ Pi −− −→ Pi−1 → · · · → F ⊗ P0 −F−−→ F ⊗ N → 0,

y se tiene entonces que T oriA (F, N ) = ker(iF ⊗ fi )/Im(iF ⊗ fi+1 ); como F es plano,
ι fi
de la sucesión exacta 0 → Im(fi+1 ) −
→ Pi −
→ Im(fi ) → 0 se obtiene la sucesión
exacta
i ⊗ι F i ⊗fi
0 → F ⊗ Im(fi+1 ) −F−→ F ⊗ Pi −− −→ F ⊗ Im(fi ) → 0,
de donde Im(iF ⊗ ι) = ker(iF ⊗ fi ). Pero nótese que Im(iF ⊗ ι) = Im(iF ⊗ fi+1 ),
luego T oriA (F, N ) = 0.
(ii)⇒(iii)⇒(iv)⇒(v): evidentes.
ι
(v)⇒(iv): sea I un ideal izquierdo de A; consideremos la sucesión exacta 0 → I − →
π
A− → A/I → 0; aplicamos la sucesión exacta larga y obtenemos la porción exacta 0 →
i ⊗ι i ⊗ι
T or1A (F, A/I) → F ⊗ I −F−→ F ⊗ A, si probamos que F ⊗ I −F−→ F ⊗ A es inyectivo,
entonces T or1A (F, A/I) = 0. Sea z := f1 ⊗a1 +· · ·+ft ⊗at ∈ ker(iF ⊗ι); consideremos
ι0
el ideal izquierdo I0 := ha1 , . . . , at } y la inclusión I0 −
→ A, con ι0 := ι|I0 ; notemos
3.3. TOR Y MÓDULOS PLANOS 95

que z ∈ ker(iF ⊗ ι0 ) y consideremos la sucesión exacta 0 → I0 → A → A/I0 → 0,


resulta entonces la porción exacta T or1A (F, A/I0 ) → F ⊗ I0 → F ⊗ A, pero por la
iF ⊗ι0
hipótesis (v), T or1A (F, A/I0 ) = 0, luego F ⊗ I0 −− −→ F ⊗ A es inyectivo, de donde
z = 0.
µ
(iv)⇒(i): consideremos un homomorfismo inyectivo K − → L, con K y L f.g.,
iF ⊗µ
queremos demostrar que F ⊗ K −− −→ F ⊗ L es inyectivo (véase el teorema 1.8.3).
Si µ(K) = L, entonces iF ⊗ µ es un isomorfismo, y por lo tanto, inyectivo. Sea
µ(K) L = hx1 , . . . , xt }; sin perder generalidad podemos asumir que x1 ∈ / µ(K),
luego L0 := µ(K) L0 + hx1 } := L1 ; si L1 = L, entonces tenemos la cadena
L0 L1 = L. En caso contrario, L1 L1 + hx2 } := L2 ; continuando de esta manera
µ
se obtiene que el homomorfismo compuesto K − → L 0 → L1 → L2 → · · · → Ls = L
coincide con µ; de igual manera, el homomorfismo compuesto F ⊗ K → F ⊗ L0 →
iF ⊗µ
F ⊗ L1 → F ⊗ L2 → · · · → F ⊗ Ls = F ⊗ L coincide con F ⊗ K −− −→ F ⊗ L. Puesto
que en esta secuencia de homomorfismos F ⊗ K → F ⊗ L0 es inyectivo (biyectivo),
i ⊗ι
entonces para demostrar que el homomorfismo compuesto F ⊗ K −F−→ F ⊗ L es
inyectivo basta probar que para cada i ≥ 0, F ⊗Li → F ⊗Li+1 es inyectivo. Notemos
que Li+1 /Li ∼ = hxi+1 }/(hxi+1 } ∩ Li ) es cı́clico, luego ahora bastará demostrar que si
L /L es un módulo cı́clico, entonces el homomorfismo F ⊗L0 → F ⊗L00 es inyectivo.
00 0

Sea L00 /L0 ∼ = A/I, con I un ideal izquierdo de A, consideremos la sucesión exacta
0 → L → L00 → L00 /L0 → 0, de la sucesión exacta larga resulta la porción exacta
0

T or1A (F, L00 /L0 ) → F ⊗ L0 → F ⊗ L00 , de donde, por la hipótesis (iv), la función
F ⊗ L0 → F ⊗ L00 es inyectiva.
Ejemplo 3.3.2. Puesto que Q es Z-plano, entonces T oriZ (Q, N ) = 0 para i ≥ 1 y
cada grupo abeliano N , y de manera particular,
T oriZ (Q, Q) = T oriZ (Q, R) = T oriZ (Q, C) = 0, para i ≥ 1.

Corolario 3.3.3. Sea F un A-módulo. Entonces las siguientes condiciones son


equivalentes:
(i) F es plano.
(ii) Para cada ideal izquierdo I de A, la sucesión 0 → F ⊗A I → F ⊗A A es exacta.
(iii) Para cada ideal izquierdo f.g. I de A, la sucesión 0 → F ⊗A I → F ⊗A A es
exacta.
Demostración. (i)⇒)(ii): evidente a partir de la definición de módulo plano.
(ii)⇒)(iii): evidente.
(iii)⇒)(i): sea I un ideal izquierdo f.g. del anillo A; aplicamos el teorema 3.1.6
ι
a la sucesión exacta 0 → I − → A → A/I → 0, con ι la inclusión, y obtenemos la
porción exacta
96 CAPÍTULO 3. TOR

i ⊗ι
0 → T or1A (F, A/I) → F ⊗A I −F−→ F ⊗A A.
Según la hipótesis y el teorema 3.3.1, F es plano.

Enseguida demostraremos el recı́proco del corolario 1.8.7 para módulos de pre-


sentación finita. Algunos resultados preliminares son necesarios antes.

Proposición 3.3.4. Sea F un A-módulo. Entonces las siguientes condiciones son


equivalentes:

(i) F es plano.
α
(ii) Para cada ideal izquierdo I de A la función F ⊗A I −
→ F I dada por m⊗a 7→ m·a
es un isomorfismo.
α
(iii) Para cada ideal izquierdo f.g. I de A la función F ⊗A I −
→ F I dada por
m ⊗ a 7→ m · a es un isomorfismo.
ι
Demostración. (i)⇒(ii): si I −
→ A es la inclusión, entonces resulta el homomorfismo
iF ⊗ι θ
compuesto F ⊗A I −−→ F ⊗A A − → F , donde θ es el isomorfismo natural definido por
θ(m ⊗ a) := m · a. Si F es plano entonces el homomorfismo compuesto α := θ(iF ⊗ ι)
α
es inyectivo y su imagen es F I, luego la función F ⊗A I −→ F I es un isomorfismo.
(ii)⇒(iii): evidente.
(iii)⇒(i): Consideremos el siguiente diagrama conmutativo
i ⊗ι
F ⊗ I −F−→ F ⊗ A
α↓ ↓θ
l
FI →
− F
donde l : F I → F es la inclusión. Entonces, ker(iF ⊗ ι) ∼
= ker(l) = 0. Según el
corolario 3.3.3, F es plano.
f g
Proposición 3.3.5. Sea F un A-módulo plano y 0 → K − → F → − M → 0 una
sucesión exacta de A-módulos. Entonces las siguientes condiciones son equivalentes:

(i) M es plano.

(ii) f (K) ∩ F I = f (K)I para cada ideal izquierdo I de A.

(iii) f (K) ∩ F I = f (K)I para cada ideal izquierdo f.g. I de A.

Demostración. (i)⇒(ii): tensorizamos por I la sucesión exacta dada y obtenemos la


f ⊗i
I I g⊗i
sucesión exacta K ⊗A I −−→ F ⊗A I −−→ M ⊗A I → 0; según la proposición 3.3.4 se
3.3. TOR Y MÓDULOS PLANOS 97

tiene el isomorfismo θF : F ⊗A I → F I, θF (f ⊗i) := f ·i; además, θK : K ⊗A I → KI,


θK (k ⊗ i) := k · i es sobreyectivo. Se tiene el siguiente diagrama exacto conmutativo:

If ⊗i I g⊗i
K ⊗A I −−−→ F ⊗A I −−−→ M ⊗A I −−−→ 0
  
γy

θK y

θF y

ι π
KI −−−→ FI −−−→ F I/f (K)I −−−→ 0

donde γ se construye por sobreyectividad, ι es la restricción de f a KI y π es el


homomorfismo canónico del cociente. Veamos que γ es un isomorfismo: por cons-
trucción γ es sobreyectivo; sea z ∈ ker(γ), existe entonces y ∈ F ⊗A I tal que
(g ⊗ iI )(y) = z, resulta γ(g ⊗ iI )(y) = 0 = πθF (y), luego θF (y) ∈ ker(π) = Im(ι) y
entonces existe u ∈ KI tal que ι(u) = θF (y), pero como θK es sobreyectivo, existe
v ∈ K ⊗A I tal que θK (v) = u; de esto resulta ιθK (v) = θF (f ⊗ iI )(v) = θF (y), con
lo cual (f ⊗ iI )(v) = y y finalmente z = (g ⊗ iI )(f⊗ iI (v)) = 0.
Notemos que g(F I) = M I y se tiene el isomorfismo δ : F I/(F I ∩ f (K)) =
F I/(F I ∩ ker(g)) ∼= M I definido por x · i + (F I ∩ f (K)) 7→ g(x · i), con x ∈ F , i ∈ I.
Consideremos entonces el homomorfismo compuesto σ := δ −1 θM γ −1
γ −1 M θ δ −1
F I/f (K)I −−→ M ⊗A I −−
→ M I −−→ F I/(F I ∩ f (K));

de manera explı́cita, σ(x · i + f (K)I) := x · i + (F I ∩ f (K)). Notemos que ker(σ) =


(F I ∩ f (K))/f (K)I, por lo tanto, σ es un isomorfismo si, y sólo si, F I ∩ f (K) =
f (K)I; por otro lado, σ es un isomorfismo si, y sólo si, θM es un isomorfismo. Lo
demostrado hasta ahora es válido para cualquier ideal izquierdo I.
Si M es plano, entonces θM es un isomorfismo (proposición 3.3.4), luego F I ∩
f (K) = f (K)I.
(ii)⇒(iii) es evidente.
(iii)⇒(i): según la hipótesis y lo probado en la primera parte, θM es un isomor-
fismo, luego por la proposición 3.3.4, M es plano.
f g
Lema 3.3.6. Sea 0 → K − →F − → M → 0 una sucesión exacta de A-módulos, donde
F es libre con base Z. Sea z ∈ F y

z = z1 · a1 + · · · + zm · am , con zi ∈ Z, ai ∈ A, 1 ≤ i ≤ m.

Sea Iz := ha1 , . . . , am } el ideal izquierdo generado por las coordenadas de z en la


base Z. Entonces, M es plano si, y sólo si, x ∈ KIf (x) , para cada x ∈ K.

Demostración. ⇒): sea x ∈ K, entonces f (x) ∈ f (K) ∩ F If (x) = f (K)If (x) =


f (KIf (x) ) (aplicamos la proposición 3.3.5 ya que todo módulo libre es plano), pero
como f es inyectivo, entonces x ∈ KIf (x) .
98 CAPÍTULO 3. TOR

⇐): sea I un ideal izquierdo de A, por la proposición 3.3.5, basta demostrar que
f (K)∩F I = f (K)I. La inclusión f (K)I ⊆ f (K)∩F I es trivial; sea f (x) ∈ f (K)∩F I
con x ∈ K, en particular f (x) ∈ F I, luego If (x) ⊆ I, y en consecuencia, KIf (x) ⊆
KI, pero por la hipótesis x ∈ KIf (x) , de donde x ∈ KI, luego f (x) ∈ f (K)I.

Teorema 3.3.7. Cada módulo plano M de presentación finita es proyectivo.


f g
Demostración. Sea 0 → K − → Am − → M → 0 una presentación finita de M ; si
probamos que esta sucesión es hendida, entonces Am ∼ = K ⊕ M , es decir, M es
proyectivo.
Puesto que K es f.g., entonces basta demostrar por inducción sobre r que dados
x1 , . . . , xr ∈ K existe un homomorfismo h : Am → K tal que h[f (xi )] = xi para cada
1 ≤ i ≤ r. Con esto, elegimos un sistema finito de generadores de K y encontramos
un homomorfismo h : Am → K tal que hf = iK , es decir, la sucesión de arriba es
hendida.
r = 1: sea x ∈ K y sea {ej }m m
j=1 la base canónica de A , entonces f (x) =
e1 · a1 + · · · + em · am . Sea I := ha1 , . . . , am }; como M es plano, x ∈ KI (lema 3.3.6),
y se tiene la representación x = k1 · s1 + · · · + kt · st , con kl ∈ K, sl ∈ I, 1 ≤ l ≤ t.
Pero cada sl es de la forma sl = bl1 a1 + · · · + blm am , luego

x = k1 · (b11 a1 + · · · + b1m am ) + · · · + kt · (bt1 a1 + · · · + btm am )


= (k1 · b11 + · · · + kt · bt1 ) · a1 + · · · + (k1 · b1m + · · · kt · btm ) · am .

Definimos

kj0 := k1 · b1j + · · · + kt · btj ; h : Am → K, ej 7→ kj0 , 1 ≤ j ≤ m.


Notemos que h(f (x)) = x.
Supongamos que ya hemos probado lo anunciado para r − 1 elementos de K y
sean x1 , . . . , xr ∈ K; según el primer paso de la inducción, existe hr : Am → K tal
que hr [f (xr )] = xr . Definimos

x0i := xi − hr [f (xi )] ∈ K, 1 ≤ i ≤ r − 1.
Por inducción, existe

h0 : Am → K tal que h0 [f (x0i )] = x0i para 1 ≤ i ≤ r − 1;


definimos

h : Am → K por h(z) := hr (z) + h0 [z − f (hr (z))], z ∈ Am .


Resulta entonces
3.4. MÓDULOS PLANOS Y MATRICES 99

h[f (xr )] = hr [f (xr )] + h0 [f (xr ) − f (hr (f (xr )))] = xr + h0 [f (xr ) − f (xr )] = xr ;


h[f (xi )] = hr [f (xi )] + h0 [f (xi ) − f (hr (f (xi )))] = hr [f (xi )] + h0 [f (xi )] −
h0 [f (hr (f (xi )))] = hr [f (xi )] + h0 [f (x0i ) + f (hr (f (xi )))] − h0 [f (hr (f (xi )))]
= hr [f (xi )] + h0 [f (x0i )] = hr [f (xi )] + x0i = xi , para cada i.

Corolario 3.3.8. Si A es un anillo noetheriano a derecha y M es un A-módulo


plano f.g., entonces M es proyectivo.
Demostración. Como A es noetheriano y M es f.g., entonces M es noetheriano, y
por lo tanto, de presentación finita. El resultado se obtiene entonces del teorema
3.3.7.

3.4. Módulos planos y matrices


En esta sección veremos un teorema de caracterización matricial de los módulos
planos debido a Orlando Villamayor.
Proposición 3.4.1. Sean A un anillo, M un A-módulo derecho, m1 , . . . , mt ∈ M
y N := hn1 , . . . , nt } un A-módulo izquierdo f.g. Entonces,
m1 ⊗ n1 + · · · + mt ⊗ nt = 0 ⇔ existen m01 , . . . , m0s ∈ M y F = [aij ] ∈ Mt×s (A)
T
tales que m1 · · · mt = m01 · · · m0s F T y F T n1 · · · nt = 0.
    

Demostración. ⇒): consideremos la siguiente sucesión exacta:


f1 f0
→ At −
L− → N → 0, f0 (ei ) := ni , 1 ≤ i ≤ t y L libre,
iM ⊗f1 iM ⊗f0
resulta entonces la sucesión exacta M ⊗A L −− −−→ M ⊗A At −− −−→ M ⊗A N → 0 y
m1 ⊗ e1 + · · · + mt ⊗ et ∈ ker(iM ⊗ f0 ) = Im(iM ⊗ f1 ); existen entonces elementos
m01 , . . . , m0s ∈ M y l1 , . . . , ls ∈ L tales que m1 ⊗ e1 + · · · + mt ⊗ et = (iM ⊗ f1 )(m01 ⊗
l1 + · · · + m0s ⊗ ls ), con lj ∈ L, 1 ≤ j ≤ s. En consecuencia, m1 ⊗ e1 + · · · + mt ⊗ et =
m01 ⊗ f1 (l1 ) + · · · + m0s ⊗ f1 (ls ) = m01 ⊗ (a11 · e1 + · · · + at1 · et ) + · · · + m0s ⊗ (a1s · e1 +
· · · + ats · et ) = (m01 · a11 + · · · + m0s · a1s ) ⊗ e1 + · · · + (m01 · at1 + · · · + m0s · ats ) ⊗ et ,
pero se tiene el isomorfismo M t ∼ = M ⊗A At definido por
M t → M ⊗A At M ⊗A At → M t
(m1 , . . . , mt ) 7→ m1 ⊗ e1 + · · · + mt ⊗ et m ⊗ (a1 , . . . , at ) 7→ (m · a1 , . . . , m · at )
luego m1 · · · mt = m01 · · · m0s F T con F := [aij ] ∈ Mt×s (A). Además, para
   

cada j, f1 (lj ) = (a1j , . . . , atj ) ∈ Im(f1 ) = ker(f0 ), luego 0 = f0 (a1j , . . . , atj ) =


 T
a1j ·f0 (e1 )+· · ·+atj ·f0 (et ) = a1j ·n1 +· · ·+atj ·nt = 0, es decir, F T n1 · · · nt = 0.
⇐): m1 ⊗ n1 + · · · + mt ⊗ nt = (m01 · a11 + · · · + m0s · a1s ) ⊗ n1 + · · · + (m01 · at1 + · · · +
m0s ·ats )⊗nt = m01 ⊗(a11 ·n1 +· · ·+at1 ·nt )+· · ·+m0s ⊗(a1s ·n1 +· · ·+ats ·nt ) = 0.
100 CAPÍTULO 3. TOR

Teorema 3.4.2 (Villamayor). Sean A un anillo y M un A-módulo. Entonces, las


siguientes condiciones son equivalentes:
(i) M es plano.
(ii) Si m1 , . . . , mt ∈ M y b1 , . . . , bt ∈ A son tales que m1 · b1 + · · · + mt · bt = 0,
entonces existen m01 , . . . , m0s ∈ M y F = [aij ] ∈ Mt×s (A) tales que
T
m1 · · · mt = m01 · · · m0s F T y F T b1 · · · bt = 0.
    

Demostración. ⇒): consideremos el ideal izquierdo I := hb1 , . . . , bt }; como M es


iM ⊗ι
plano el homomorfismo M ⊗A I −− −→ M ⊗A A es inyectivo, con ι la inclusión
canónica. Luego, m1 ⊗ b1 + · · · mt ⊗ bt = 0 en M ⊗A I, aplicamos entonces la
proposición anterior con N := I.
⇐): sea I un ideal izquierdo f.g. del anillo A; según el corolario 3.3.3, bas-
ta entonces probar que iM ⊗ ι es inyectivo. Sean I = hb1 , . . . , bt } y z = m1 ⊗
b1 + · · · + mt ⊗ bt ∈ ker(iM ⊗ ι), entonces m1 · b1 + · · · + mt · bt = 0, y por
hipótesis, existen m01 , . . . , m0s ∈ M y F = [aij ] ∈ Mt×s (A) tales que m1 · · · mt =
 0 T
m1 · · · m0s F T y F T b1 · · · bt = 0. Resulta, z = (m01 · a11 + · · · + m0s · a1s ) ⊗
 

b1 + · · · + (m01 · at1 + · · · + m0s · ats ) ⊗ bt = m01 ⊗ (a11 b1 + · · · + at1 bt ) + · · · + m0s ⊗


(a1s b1 + · · · + ats bt ) = 0.

3.5. Torsión de un módulo


Estudiaremos en esta sección la torsión para módulos sobre dominios de Ore.
Definición 3.5.1. Sea D un dominio de Ore a derecha y M un D-módulo, el con-
junto
T (M ) := {m ∈ M |m · d = 0, para algún d ∈ D − {0}}
es llamado el submódulo de torsión de M y sus elementos se denominan los
elementos de torsión de M . M es de torsión si T (M ) = M y M es sin
torsión si T (M ) = 0.
El hecho que T (M ) es un D-submódulo de M es una consecuencia de la condición
de Ore a derecha. En efecto, para m1 , m2 ∈ T (M ) y d1 , d2 ∈ D veamos que m1 ·
d1 + m2 · d2 ∈ T (M ): existen p1 , p2 ∈ D − {0} tales que m1 · p1 = 0 y m2 · p2 = 0;
por la condición de Ore aplicada a los elementos d1 , p1 y d2 , p2 , existen r1 , r2 ∈ D y
s1 , s2 ∈ D − {0} tales que d1 s1 = p1 r1 , d2 s2 = p2 r2 ; ahora aplicamos nuevamente la
condición de Ore a s1 , s2 y encontramos elementos no nulos t1 , t2 tales que s1 t1 = s2 t2 ,
resulta entonces que (m1 · d1 + m2 · d2 ) · s1 t1 = m1 · (d1 s1 )t1 + m2 · (d2 s2 )t2 =
(m1 · p1 ) · r1 t1 + ·(m2 · p2 ) · r2 t2 = 0, con s1 t1 6= 0.
3.5. TORSIÓN DE UN MÓDULO 101

Teorema 3.5.2. Sea D un dominio de Ore a derecha y M un D-módulo. Si M es


plano, entonces M es sin torsión. Además, si D es un dominio de ideales izquierdos
principales la afirmación recı́proca es válida.

Demostración. Sean m ∈ M y d ∈ D − {0} tales que m · d = 0, según el teorema


3.4.2, existen m01 , . . . , m0s ∈ M y F = a1 · · · as ∈ M1×s (D) tales que m =
m01 · a1 + · · · + m0s · as y F T [d] = 0, pero D es un dominio, luego a1 = · · · = as = 0,
con lo cual m = 0.
Supongamos ahora que D es un dominio de ideales izquierdos principales y sea
I un ideal izquierdo f.g. de R, entonces I = hd}, por el corolario 3.3.3, basta de-
mostrar que M ⊗D I → M ⊗D D es inyectivo. Sea z un elemento del núcleo de este
homomorfismo, entonces z = m ⊗ d, con m ∈ M , por lo tanto, m · d = 0, se tiene
entonces que m = 0 o d = 0, es decir, z = 0.

Teorema 3.5.3. Sea A un anillo local con radical de Jacobson J y sea M un A-


módulo plano. Si m1 , . . . , mt ∈ M son tales que m1 , . . . , mt son linealmente indepen-
dientes en el A/J-espacio vectorial M/M J, entonces m1 , . . . , mt ∈ M son lineal-
mente independientes. En consecuencia, para un anillo local las siguientes condi-
ciones son equivalentes para cualquier módulo M f.g.:

(i) M es plano.

(ii) M es libre.

(iii) M es proyectivo.

Demostración. La prueba de la primera parte se hace mediante inducción sobre t.


Para t = 1, sea r ∈ A tal  que m · r = 0, según el teorema 3.4.2, existen m01 , . . . , m0s ∈
M y F := r1 · · · rs ∈ M1×s (A) tales que m = m1 · r1 + · · · + m0s · rs y F T [r] = 0.
0

Puesto que m 6= 0, entonces m ∈ / M J, con lo cual existe rk ∈ / J y en consecuencia,



rk ∈ A , luego r = 0. Suponemos por inducción que la afirmación es cierta para t − 1
elementos, y sean r1 , . . . , rt ∈ A tales que m1 · r1 + · · · mt · rt = 0. Según el teorema
3.4.2, existen m01 , . . . , m0s ∈ M y F = [aij ] ∈ Mt×s (A) tales que m1 · · · mt =

 0 T
m1 · · · m0s F T y F T r1 · · · rt = 0. Puesto que mt 6= 0, entonces mt ∈
 
/ MJ

y existe k tal que atk ∈ / J, es decir, atk ∈ A , y como a1k r1 + · · · + atk rt = 0, entonces
rt = b1 r1 +· · ·+bt−1 rt−1 , con bi ∈ A, 1 ≤ i ≤ t−1, luego m1 ·r1 +· · ·+mt−1 ·rt−1 +mt ·
(b1 r1 +· · ·+bt−1 rt−1 ) = 0 = (m1 +mt ·b1 )·r1 +· · ·+(mt−1 +mt ·bt−1 )·rt−1 , pero notemos
que {m1 + mt · b1 , · · · , mt−1 + mt · bt−1 } = {m1 + mt · b1 , · · · , mt−1 + mt · bt−1 } es
linealmente independiente en M/M J, por inducción se tiene entonces que r1 = · · · =
rt−1 = 0, de donde rt = 0.
Finalmente, las implicaciones (ii)⇒(iii)⇒(i) son bien conocidas. Veamos (i)⇒
(ii): si M es plano y {m1 , . . . , mt } es un A/J-base de M/M J, entonces es facil
probar mediante el lema de Nakayama (véase [18]) que m1 , . . . , mt es un sistema
102 CAPÍTULO 3. TOR

de generadores de M , y por la parte ya demostrada, estos vectores son linealmente


independientes, es decir, conforman una base de M .

3.6. Tor y torsión


Veremos ahora en álgebra conmutativa la relación entre el funtor T or y el submódulo
de torsión de un módulo M .

Teorema 3.6.1. Sean D un DI y Q := Q(D) su cuerpo de fracciones.

(i) Si M es un D-módulo de torsión, entonces

T or1D (Q/D, M ) ∼
= M.

(ii) T oriD (Q/D, M ) = 0 para i ≥ 2 y cada D-módulo M .

(iii) Si M es un D-módulo sin torsión, entonces

T or1D (Q/D, M ) = 0.

(iv) T define un funtor covariante


T
D M od −
→ D M od
M 7→ T (M )
f
M−
→ N 7→ T (f ) := f |T (M )

Además, T or1D (Q/D, ) y T son naturalmente equivalentes. En particular,

T or1D (Q/D, M ) ∼
= T (M ).

(v) Para cada D-módulo M se tiene la siguiente sucesión exacta

0 → T (M ) → M → Q ⊗ M → Q/D ⊗ M → 0.

(vi) Sea M un D-módulo. M es de torsión si, y sólo si, Q ⊗ M = 0.

(vii) Si N es un D-módulo de torsión, entonces T oriD (M, N ) es de torsión para


cada D-módulo M e i ≥ 0.

(viii) T oriD (M, N ) es de torsión para i ≥ 1 y cualesquiera D-módulos M y N .


3.6. TOR Y TORSIÓN 103

Demostración. (i) Consideremos la sucesión exacta


l π
0→D−
→Q−
→ Q/D → 0
se obtiene entonces la sucesión exacta larga
π⊗i θ l⊗i π⊗i
· · · → T or1D (Q, M ) −−−−
→ T or1D (Q/D, M ) −→
M 1
D ⊗ M −−−M M
→ Q ⊗ M −−−−
→ Q/D ⊗ M → 0

Pero Q es D-plano, entonces T or1D (Q, M ) = 0, además como M es de torsión,


entonces Q ⊗ M = 0 (Proposición 1.6.3, parte (v). Veamos una prueba directa: dado
m ∈ M , existe a ∈ D, a 6= 0 tal que am = 0, entonces para cada pq ∈ Q y m ∈ M
se tiene que pq ⊗ m = pa
qa
p
⊗ m = qa ⊗ am = 0). Resulta la siguiente porción exacta

θ
0 → T or1D (Q/D, M ) −
→1
D ⊗ M → 0,

es decir, θ1 es un isomorfismo, luego

T or1D (Q/D, M ) ∼
= M.
(ii) Consideremos nuevamente la sucesión exacta larga de (i):
· · · → T oriD (D, M ) → T oriD (Q, M ) → T oriD (Q/D, M ) → T ori−1
D
(D, M ) → · · ·

Pero como D y Q son planos, entonces tenemos la siguiente porción exacta

0 → T oriD (Q/D, M ) → 0

para cada i ≥ 2, es decir, T oriD (Q/D, M ) = 0, para cada D-módulo M .


(iii) La prueba de este numeral hace uso del siguiente resultado: Si M es un D-
módulo sin torsión, entonces M se puede sumergir en un Q-espacio vectorial V . Si
M es además f.g, entonces M puede sumergirse en un D-módulo libre de dimensión
finita.
Con esta propiedad probemos (iii): consideremos la siguiente porción de la suce-
sión larga obtenida de la sucesión exacta de D-módulos 0 → M → V → V /M → 0:
· · · → T or2D (Q/D, V /M ) → T or1D (Q/D, M ) → T or1D (Q/D, V )

Por (ii) el primer grupo es nulo y T or1D (Q/D, V ) = 0 ya que V es D-plano por ser
suma directa de copias del D-módulo plano Q. Ası́ pues, T or1D (Q/D, M ) = 0.
Veamos entonces la prueba de la propiedad enunciada antes. Existe un D-módulo
inyectivo N y un homomorfismo inyectivo φ : M → N ; consideremos la composición
φ j
M−
→N −
→ N/T (N ).

Como M es sin torsión, entonces jφ es inyectiva: en efecto, si jφ(m) = φ(m) = 0,


entonces φ(m) ∈ T (N ) y existe d 6= 0 en D tal que d · φ(m) = 0, es decir, φ(dm) = 0
104 CAPÍTULO 3. TOR

luego d · m = 0 y puesto que M es sin torsión, m = 0. Notemos que N/T (N ) es


sin torsión y además D-divisible ya que N es divisible por ser inyectivo (véase la
proposición 1.7.10). Tenemos entonces que N/T (N ) es sin torsión y D-divisible, pero
esto implica que N/T (N ) es un Q-espacio vectorial.
En efecto, probemos que si L es un D-módulo sin torsión y divisible, entonces L
es un Q-espacio ([27], Ejercicio 3.20): sean x ∈ L y pq ∈ Q, entonces definimos

p
· x := p · x0
q

con x = q · x0 , x0 ∈ L. Para simplificar un poco la notación suprimiremos el punto


entre escalares y vectores. Veamos que este producto está bien definido. Si x = qx0 =
0
qx∗ entonces q(x0 − x∗ ) = 0, ası́ x0 = x∗ . Sea pq = pq0 , con x = qx0 = q 0 x∗ , puesto que
pq 0 = qp0 , entonces q 0 px0 = qp0 x0 = p0 q 0 x∗ , de donde px0 = p0 x∗ , lo que prueba que
el producto está bien definido. Veamos que L con este producto tiene estructura de
Q-espacio:
p p0
 0
pq + qp0
  
+ x= x
q q0 qq 0
se tiene que x = qx0 y x0 = q 0 x∗ , entonces x = (qq 0 )x∗ , luego
 0
pq + qp0

x = (pq 0 + qp0 )x∗ = p(q 0 x∗ ) + qp0 x∗ = px0 + p0 (qx∗ )
qq 0
pero como x = q 0 (qx∗ ), entonces

p p0 p0
 
p
+ 0 x = x + 0 x.
q q q q

También se tiene que


p p p
(x + y) = x + y
q q q
ya que si x = qx0 , y = qy 0 , entonces x+y = q(x0 +y 0 ) y pq (x+y) = p(x0 +y 0 ) = px0 +py 0 .
Además,  
pr pr
x = x,
qs qs
si x = (qs)x0 = q(sx0 ) = s(qx0 ), entonces
 
pr r
x = (pr)x0 y x = rqx0 ,
qs s

luego
p r  p
x = (rqx0 ) = p(rx0 ) = (pr)x0
q s q
3.6. TOR Y TORSIÓN 105

ya que rqx0 = qrx0 . Finalmente, 11 x = x ya que x = 1x. Esto completa la prueba


que L es un Q-espacio.
Ahora solo resta probar la segunda parte de la propiedad enunciada arriba: sea
M = hm1 , . . . , mt } y φ : M → V la inyección de M en el Q-espacio V , con zi :=
φ(mi ), 1 ≤ i ≤ t. Consideremos el Q-espacio W := hz1 , . . . zt i, sea {ω1 , . . . , ωr } una
Q-base de W , entonces M ,→ W . Expandimos los zi como una combinación lineal
de los elementos de la base de W:
(i) (i)
p1 pr
zi = (i)
ω1 + · · · + (i)
ωr .
s1 sr

Entonces existe si 6= 0 tal que si zi ∈ D hω1 , . . . , ωt i, 1 ≤ i ≤ t, luego existe s 6= 0 tal


que sW ⊆ D hω1 , . . . , ωr i, pero sW = W , ası́ pues M ,→ W = sW ⊆ D hω1 , . . . , ωr i.
(iv) Para demostrar este punto necesitamos repasar algunos conceptos en cate-
gorı́as y funtores (véanse [27] y [19]).
Sean B, C dos categorı́as y F, G : B → C funtores covariantes. Una transfor-
mación natural α : F : G es una colección de morfismos

{αX : F (X) → G(X)}X∈Ob(B)

tal que para cada morfismo f : X → X 0 en B, el siguiente diagrama es conmutativo:


αX
F (X) - G(X)

F (f ) G(f )

? ?
αX 0
0 - G(X 0 )
F (X )

La transformación natural α se dice que es una equivalencia natural o que F y


G son funtores naturalmente equivalentes, si para cada X ∈ Ob(B), αX es un
isomorfismo. Un morfismo a : C → C 0 en una categorı́a C es un isomorfismo, si
existe otro morfismo b : C 0 → C tal que ab = iC 0 , ba = iC . Las transformaciones
naturales para funtores contravariantes se definen en forma similar, invirtiendo el
sentido de las flechas verticales.
En (iv) tenemos entonces que B = C = D M od, los funtores en este caso son
F = T : C → C y G = T or1D (Q/D, ) : C → C. Sea X = M ∈ Ob(C) un D-módulo,
entonces
(
F (X) := T (M ), el submódulo de torsión de M
G(X) := T or1D (Q/D, M )
106 CAPÍTULO 3. TOR

Si f : M → M 0 es un D-homomorfismo, entonces F (f ) = T (f ) : T (M ) → T (M 0 );
notemos que si l : T (M ) → M y l0 : T (M 0 ) → M 0 son las inclusiones, en-
tonces l0 T (f ) = f l: en efecto, si m ∈ T (M ), entonces f l(m) = f (l(m)) = f (m);
l0 T (f )(m) = l0 (f (m)) = f (m). G(f ) = T or1D (Q/D, f ) = 1 ⊗ f1 . Debemos entonces
encontrar isomorfismos αM y αM 0 tal que el siguiente diagrama sea conmutativo

αM
T (M ) - T or D (Q/D, M )
1

T (f ) 1⊗f1

? ?
αM 0
0 - T or D (Q/D, M 0 )
T (M ) 1

De ser ası́, entonces T or1D (Q/D, M ) ∼


= T (M ), para todo D-módulo M . Se tienen los
siguientes diagramas

α i⊗l1
T (M ) - T or D (Q/D, T (M )) - T or D (Q/D, M )
1 1

T (f ) i⊗T (f )1 i⊗f1

? ? ?
0 α0 - T or D (Q/D, T (M 0 )) - T or D (Q/D, M 0 )
T (M ) 1 1
i⊗l10

donde α y α0 son los isomorfismos de la parte (i), es decir, α = εθ1 , α0 = ε0 θ10 , con
ε : D⊗M → M , ε0 : D⊗M 0 → M 0 los isomorfismos habituales. La idea es demostrar
que i ⊗ l1 y i ⊗ l10 son isomorfismos y0 que los dos rectángulos conmutan, con lo cual
0
αM := i ⊗ l1 α y αM 0 := i ⊗ l1 α .

Veamos la forma precisa de los homomorfismos involucrados en el diagrama an-


terior. Consideremos resoluciones proyectivas de T (M ), M , M 0 y T (M 0 ):
3.6. TOR Y TORSIÓN 107

s2 s1 s0
··· - T
2
- T
1
- T
0
- T (M ) - 0

l2 l1 l0 l

? ? ? ?
h2 h1 h0
··· - M2 - M1 - M0 - M - 0

f2 f1 f0 f

h02 h01 h00


? ? ? ?
··· - M0 - M0 - M0 - M0 - 0
2 1 0
6 6 6 6
l20 l10 l00 l0

s02 s01 s00


··· - T0 - T0 - T0 - T (M 0 ) - 0
2 1 0

Entonces
i⊗s2 i⊗s1
··· - Q/D ⊗ T2 - Q/D ⊗ T1 - Q/D ⊗ T0 - 0

i⊗l2 i⊗l1 i⊗l0

? ? ?
i⊗h2 i⊗h1
··· - Q/D ⊗ M2 - Q/D ⊗ M1 - Q/D ⊗ M0 - 0

i⊗f2 i⊗f1 i⊗f0

i⊗h02 i⊗h01
? ? ?
··· - Q/D ⊗ M 0 - Q/D ⊗ M 0 - Q/D ⊗ M 0 - 0
2 1 0
6 6 6
i⊗l20 i⊗l10 i⊗l00

i⊗s02 i⊗s01
··· - Q/D ⊗ T 0 - Q/D ⊗ T 0 - Q/D ⊗ T 0 - 0
2 1 0

i⊗l1 i⊗f1
T or1D (Q/D, T (M )) - T or D (Q/D, M ) - T or D (Q/D, M 0 )
1 1

? ? ?
ker(i ⊗ s1 ) -
ker(i ⊗ h1 ) -
ker(i ⊗ h01 )
Im(i ⊗ s2 ) Im(i ⊗ h2 ) Im(i ⊗ h02 )
108 CAPÍTULO 3. TOR

donde

ker(i ⊗ s1 ) i⊗l1
-
ker(i ⊗ h1 )
Im(i ⊗ s2 ) Im(i ⊗ h2 )

t - (i ⊗ l )(t)
1

ker(i ⊗ h1 ) i⊗f1
-
ker(i ⊗ h01 )
Im(i ⊗ h2 ) Im(i ⊗ h02 )

m - (i ⊗ f )(m)
1

ker(i ⊗ s01 ) i⊗l10


-
ker(i ⊗ h01 )
Im(i ⊗ s02 ) Im(i ⊗ h02 )

u - (i ⊗ l0 )(u)
1

También,

s2 s1 s0
··· - T2 - T1 - T0 - T (M ) - 0

T (f )2 T (f )1 T (f )0 T (f )

s02 s01 s00


? ? ? ?
··· - T0 - T0 - T0 - T (M 0 ) - 0
2 1 0

implica

i⊗s2 i⊗s1
··· - Q/D ⊗ T2 - Q/D ⊗ T1 - Q/D ⊗ T0 - 0

i⊗T (f )2 i⊗T (f )1 i⊗T (f )0

i⊗s02 i⊗s01
? ? ?
··· - Q/D ⊗ T 0 - Q/D ⊗ T 0 - Q/D ⊗ T 0 - 0
2 1 0
3.6. TOR Y TORSIÓN 109

luego
i⊗T (f )1
T or1D (Q/D, T (M )) - T or D (Q/D, T (M 0 ))
1

? ?
ker(i ⊗ s1 ) -
ker(i ⊗ s01 )
Im(i ⊗ s2 ) Im(i ⊗ s02 )

t - (i ⊗ T (f ) )(t)
1

Veamos ahora la forma precisa de α y α0 : comencemos recordando la definición de θ1


y θ10 , es decir, los homomorfismos de conexión de la sucesión exacta larga asociada
ι π
la sucesión exacta 0 → D − →Q− → Q/D → 0:

0 0 0

? ? ?
··· - D ⊗ T2 - D ⊗ T1 - D ⊗ T0 - D ⊗ T (M )

ι⊗iT2 ι⊗iT1 ι⊗iT0

? ? ?
··· - Q ⊗ T2 - Q ⊗ T1 - Q ⊗ T0 - D ⊗ T (M )

π⊗iT2 π⊗iT1 π⊗iT0

? ? ?
··· - Q/D ⊗ T2 - Q/D ⊗ T1 - Q/D ⊗ T0 - D ⊗ T (M )

? ? ?
0 0 0

Sea t ∈ ker(i ⊗ s1 ) ⊆ Q/D ⊗ T1 , entonces existe t∗ ∈ Q ⊗ T1 tal que (π ⊗ iT1 )(t∗ ) = t.


Luego, (i ⊗ s1 )(t∗ ) ∈ Q ⊗ T0 , ası́

(π ⊗ iT0 )(i ⊗ s1 )(t∗ ) = (i ⊗ s1 )(π ⊗ iT1 )(t∗ ) = (i ⊗ s1 )(t) = 0,

de donde (i⊗s1 )(t∗ ) ∈ ker(π⊗iT0 ) = Im(ι⊗iT0 ), con lo cual (i⊗s1 )(t∗ ) = (ι⊗iT0 )(t0 )
110 CAPÍTULO 3. TOR

con t0 ∈ D ⊗ T0 . Entonces definimos

θ1 (t) := (iD ⊗ s0 )(t0 ) ∈ D ⊗ T (M ),

con t = (π ⊗ iT1 )(t∗ ), t∗ ∈ Q ⊗ T1 y (i ⊗ s1 )(t∗ ) = (ι ⊗ iT0 )(t0 ) donde t0 ∈ D ⊗ T0 .


Sean t0 := 1 ⊗ t0 , con t0 ∈ T0 , entonces

εT (M ) θ1 (t) = εT (M ) (1 ⊗ s0 (t0 )) = s0 (t0 ),


además
(ι ⊗ iT0 )(t0 ) = (ι ⊗ iT0 )(1 ⊗ t0 ) = 1 ⊗ t0
luego (i ⊗ s1 )(t∗ ) = 1 ⊗ t0 . Ası́ pues, definimos

εT (M ) θ1 (t) := s0 (t0 )

con t = (π ⊗iT1 )(t∗ ), t∗ ∈ Q⊗T1 y (i⊗s1 )(t∗ ) = 1⊗t0 con t0 ∈ T0 . De forma análoga,

εT (M 0 ) θ10 (u) := s00 (u0 )


con u = (π ⊗ iT1 )(u∗ ), u∗ ∈ Q ⊗ T10 y (i ⊗ s01 )(u∗ ) = 1 ⊗ u0 con u0 ∈ T00 .
De esta manera definimos
α := θ1−1 ε−1
T (M )

α0 := θ10−1 ε−1
T (M 0 ) .

Veamos que el rectángulo de la izquierda es conmutativo, es decir, (i ⊗ T (f )1 )α =


α0 T (f ), en otras palabras, veamos que

εT (M 0 ) θ10 (i ⊗ T (f )1 ) = T (f )εT (M ) θ1 .

En efecto, sea t ∈ T or1D (Q/D, T (M )), t ∈ ker(i⊗s1 ) ⊆ Q/D⊗T1 , εT (M ) θ1 (t) = s0 (t0 )


con t0 ∈ T0 , 1 ⊗ t0 = (i ⊗ s1 )(t∗ ) con t∗ ∈ Q ⊗ T1 tal que (π ⊗ iT1 )(t∗ ) = t. Entonces

T (f )(θ1 (t)) = f θ1 (t) = f (s0 (t0 )) ∈ T (M 0 ).




De otra parte, (i ⊗ T (f )1 )(t) = (i ⊗ T (f )1 )(t) con (i ⊗ T (f )1 )(t) ∈ ker(i ⊗ s01 ) ⊆


Q/D ⊗ T10 . Sea u := (i ⊗ T (f )1 )(t), entonces εT (M 0 ) θ10 (u) = s00 (u0 ) donde u0 ∈ T00 ,
1 ⊗ u0 = (i ⊗ s01 )(u∗ ) con u∗ ∈ Q ⊗ T10 tal que (π ⊗ iT10 )(u∗ ) = u. En calidad de u0
podemos tomar
u0 := T (f )0 (t0 )
y

u∗ := (i ⊗ T (f )1 )(t∗ ) ∈ Q ⊗ T10 .
3.6. TOR Y TORSIÓN 111

En efecto, u0 = T (f )0 (t0 ) ∈ T00 ,


(i ⊗ s01 )(u∗ ) = (i ⊗ s01 ) [(i ⊗ T (f )1 )(t∗ )]
= (i ⊗ s01 T (f )1 )(t∗ )
= (i ⊗ T (f )0 s1 )(t∗ )
= (i ⊗ T (f )0 )(1Q ⊗ s1 )(t∗ )
= (i ⊗ T (f )0 )(1 ⊗ t0 )
= 1 ⊗ T (f )0 (t0 )
= 1 ⊗ u0 ,

(π ⊗ iT10 )(u∗ ) = (π ⊗ iT10 ) [(i ⊗ T (f )1 )(t∗ )]


= (πi ⊗ iT10 T (f )1 )(t∗ )
= (π ⊗ T (f )1 )(t∗ )
= (iπ) ⊗ (T (f )1 iT1 )(t∗ )
= (i ⊗ T (f )1 )(π ⊗ iT1 )(t∗ )
= (i ⊗ T (f )1 )(t) = u.
Además,
s00 (u0 ) = s00 (T (f )0 (t0 ))
= (T (f )s0 )(t0 )
= T (f )(s0 (t0 ))
= f (s0 (t0 )).

Hemos pues demostrado que εT (M 0 ) θ10 (i ⊗ T (f )1 ) = T (f )εT (M ) θ1 .


Consideremos ahora el rectángulo de la derecha: para comenzar, repasaremos la
definición de i ⊗ l1 y de i ⊗ l10 :a la sucesión
l j
0 → T (M ) −
→M −
→ M/T (M ) → 0
aplicamos la sucesión exacta larga
D D i⊗l1 D D
· · · → T or2 (Q/D, M/T (M )) → T or1 (Q/D, T (M )) −
−−−
→ T or1 (Q/D, M ) → T or1 (Q/D, M/T (M )),

pero T or2D (Q/D, M/T (M )) y T or1D (Q/D, M/T (M )) son nulos, entonces i ⊗ l1 es
un isomorfismo. De manera análoga, i ⊗ l10 es un isomorfismo.
Veamos entonces la conmutatividad del rectángulo: Sea t ∈ T or1D (Q/D, T (M ))
con t ∈ ker(i ⊗ s1 ) ⊆ Q/D ⊗ T1 , entonces (i ⊗ l1 )(t) = (i ⊗ l1 )(t), luego
(i ⊗ f1 )((i ⊗ l1 )(t)) = (i ⊗ f1 )((i ⊗ l1 )(t))
= (i ⊗ f1 l1 )(t),
112 CAPÍTULO 3. TOR

por el otro lado tenemos (i ⊗ T (f )1 )(t) = (i ⊗ T (f )1 )(t). Por lo tanto

(i ⊗ l10 )((i ⊗ T (f )1 )(t)) = (i ⊗ l10 )((i ⊗ T (f )1 )(t))


= (i ⊗ l10 T (f )1 )(t)

pero como l0 T (f ) = f l, entonces (l0 T (f ))i = (f l)i , para cada i ≥ 0, es decir,


li0 T (f )i = fi li , en particular, l10 T (f )1 = f1 l1 , lo que completa la prueba de (iv).
(v) Consideremos la sucesión exacta

0 → D → Q → Q/D → 0

y la correspondiente sucesión exacta larga para M :


D D D D D D
· · · → T or1 (D, M ) → T or1 (Q, M ) → T or1 (Q/D, M ) → T or0 (D, M ) → T or0 (Q, M ) → T or0 (Q/D, M ) → 0

ahora, como T or1D (Q, M ) = 0, entonces aplicando (iv) queda la siguiente porción
exacta
0 → T (M ) → M → Q ⊗ M → Q/D ⊗ M → 0.
(vi) Parte (v) de la Proposición 1.6.3. Otra manera: como vimos en (i), si M es de
torsión entonces Q ⊗ M = 0. Recı́procamente, si Q ⊗ M = 0, entonces S0−1 M = 0,
con S0 := D − {0}, luego m1 = 0 para cada m ∈ M , es decir, dado m ∈ M existe
d 6= 0 tal que d · m = 0.
(vii) Haremos inducción sobre i. Para i = 0, T or0D (M, N ) = M ⊗N . Dado n ∈ N ,
existe a 6= 0 en D tal que an = 0, entonces
a(m ⊗ n) = am ⊗ n = ma ⊗ n = m ⊗ an = 0.
Ası́ pues, T or0D (M, N ) es de torsión.
i = 1: Para M se tiene siempre una sucesión exacta de la forma

0→L→P →M →0

con P proyectivo, entonces aplicamos la sucesión exacta larga con N y tenemos la


siguiente porción exacta

0 → T or1D (M, N ) → L ⊗ N → P ⊗ N → M ⊗ N → 0.

Por lo probado en el caso i = 0, tenemos que L ⊗ N es de torsión, pero como


T or1D (M, N ) ⊆ L ⊗ N , entonces T or1D (M, N ) es de torsión. Supongamos induc-
tivamente que T oriD (M, N ) es de torsión para todo D-módulo M . Consideremos
nuevamente la sucesión exacta larga:
D D D
· · · → T ori+1 (L, N ) → T ori+1 (L, P ) → T ori+1 (M, N ) → T oriD (L, N ) → · · ·
3.6. TOR Y TORSIÓN 113

D
pero T ori+1 (L, P ) = 0, entonces tenemos la siguiente porción exacta
D
0 → T ori+1 (M, N ) → T oriD (L, N ) → · · ·
D
entonces T ori+1 (M, N ) ⊆ T oriD (L, N ) es de torsión.
(viii) Inducción sobre i. i = 1: Comenzamos asumiendo que N es sin torsión,
según (v) se tiene la siguiente sucesión exacta:
0 → N → Q ⊗ N → Q/D ⊗ N → 0
es exacta, donde Q ⊗ N es un Q-espacio; además,
Q/D ⊗ N = T or0D (Q/D, N )
es de torsión ya que Q/D es de torsión (parte (vii)). Por tanto, aplicamos la sucesión
exacta larga
· · · → T or2D (M, Q/D ⊗ N ) → T or1D (M, N ) → T or1D (M, Q ⊗ N ) → T or1D (M, Q/D ⊗ N ) → · · ·

Según (vii), T or2D (M, Q/D ⊗N ) es de torsión pues Q/D ⊗N es de torsión y además
T or1D (M, Q ⊗ N ) = 0 pues Q ⊗ N es D-plano por ser un Q-espacio, es decir, es suma
directa de copias de Q. Entonces T or1D (M, N ) es cociente del módulo de torsión
T or2D (M, Q/D ⊗ N ), de donde, T or1D (M, N ) es de torsión.
Ahora supongamos que N es arbitrario, consideremos la sucesión
0 → T (N ) → N → N/T (N ) → 0
resulta entonces la sucesión exacta larga
α β
· · · → T or1D (M, T (N )) −
→ T or1D (M, N ) −
→ T or1D (M, N/T (N )) → · · ·

pero T or1D (M, T (N )) es de torsión dado que T (N ) lo es y T or1D (M, N/T (N )) es de


torsión pues N/T (N ) es sin torsión y por lo recien probado. Pero
Im(β) ∼ = T orD (M, N )/Ker(β) = T orD (M, N )/Im(α)
1 1

donde Im(β) e Im(α) son de torsión, esto implica que T or1D (M, N ) es de torsión.
En efecto, se tiene en general la siguiente propiedad: si L/P es de torsión y P es
de torsión, entonces L es de torsión: l ∈ L, entonces existe a 6= 0 tal que al = 0, es
decir, al ∈ P , entonces existe b 6= 0 tal que bal = 0.
Para terminar, suponemos que T oriD (M, N ) es de torsión para cualesquiera
M, N . Entonces,
D D D
· · · → T ori+1 (L, N ) → T ori+1 (L, P ) → T ori+1 (M, N ) → T oriD (L, N ) → · · ·

es exacta a partir de la la sucesión


0→L→P →M →0
como en (vii), entonces
D
T ori+1 (M, N ) ⊆ T oriD (L, N )
es de torsión. Esto completa la prueba del teorema.
114 CAPÍTULO 3. TOR

3.7. Ejercicios
1. Demuestre el teorema 3.1.2.

2. Complete la demostración del corolario 3.1.3.

3. Complete la demostración del teorema 3.1.6.

4. Complete la demostración del teorema 3.1.7.

5. Calcule:

(i) T or1Z (Q2 ⊕ Z6 ⊕ Z10 , Z14 ⊕ Z15 ).


(ii) T or1Z (Q/Z, M ), donde M es un grupo abeliano finitamente generado.

6. Sean M, N módulos proyectivamente equivalentes (véase el ejercicio 5 del


capı́tulo 2). Demuestre que para cada módulo izquierdo L y n ≥ 1,

T ornA (M, L) ∼
= T ornA (N, L).

7. Demuestre la siguiente generalización del teorema 3.1.8. Sean R un anillo con-


mutativo y B una R-álgebra conmutativa plana. Entonces, para i ≥ 0 y cua-
lesquiera R-módulos M, N se tiene el B-isomorfismo

T oriR (M, N ) ⊗R B ∼
= T oriB (M ⊗R B, N ⊗R B).

8. Sea R un anillo conmutativo, J ⊂ R un ideal propio de R y sea B :=


R[x1 , . . . , xn ]/J. Demuestre que para cada i ≥ 0 y cada R-módulo M se tiene
el R[x1 , . . . , xn ]-isomorfismo

T oriR (M, B) ∼
R[x ,...,x ]
= T ori 1 n (M ⊗R R[x1 , . . . , xn ], B).

9. Demuestre que si cada submódulo f.g. de un A-módulo M es plano, entonces


M es plano.
Capı́tulo 4

Dimensiones de módulos y anillos

Aplicaremos en este capı́tulo las técnicas consideradas en los anteriores para estu-
diar algunas de las dimensiones de módulos y anillos usadas con mayor frecuencia
en álgebra homológica, y en general, en álgebra tanto conmutativa como no conmu-
tativa.

4.1. Dimensiones proyectiva, inyectiva y plana de


un módulo
Definición 4.1.1. Sea M un A-módulo no nulo.

(i) Se dice que la resolución proyectiva


fn+1 fn f1 f0
· · · → Pn+1 −−→ Pn −→ Pn−1 → · · · → P1 −
→ P0 −
→M →0 (4.1.1)

de M es finita de longitud n ≥ 0 si Pn 6= 0 y Pi = 0 para i ≥ n + 1.

(ii) Se dice que M tiene dimensión proyectiva finita si M tiene al menos una
resolución proyectiva finita. Si este no es el caso, se dice que la dimensión
proyectiva de M es infinita y se escribe pd(M ) = ∞.

(iii) Sea M de dimensión proyectiva finita, se define

pd(M ) := mı́n{n|n es la longitud de una resolución proyectiva finita de M }.

Observación 4.1.2. (i) Nótese que para un módulo no nulo todas las resoluciones
proyectivas son no nulas. En cambio, el módulo nulo puede tener resoluciones no
iP 0
nulas y la nula: 0 → P −→ P −→ 0 → 0, con P cualquier proyectivo no nulo, y
0
0 → 0 → 0 → 0. Si P es proyectivo no nulo, entonces 0 → P −→ 0 → 0 no es exacta.

115
116 CAPÍTULO 4. DIMENSIONES DE MÓDULOS Y ANILLOS

Esto hace que no podamos definir la dimensión proyectiva de módulo nulo como
igual a cero, en cambio si definimos la longitud de la resolución nula como igual a
−1, entonces es natural definir pd(0) := −1.
(ii) A partir de esta definición es claro que un A-módulo no nulo P es proyectivo
si, y sólo si, pd(P ) = 0. De manera completamente análoga se define la dimensión
inyectiva de un módulo M a través de resoluciones inyectivas, y se denota por
id(M ). Se tiene entonces que un A-módulo no nulo N es inyectivo si, y sólo si,
id(N ) = 0. De igual manera, usando resoluciones planas del módulo M , se define la
dimensión plana y se denota por fd(M ). Se tiene en este caso que M no nulo es
plano si, y sólo si, fd(M ) = 0.
(iii) Puesto que todo módulo proyectivo es plano, entonces fd(M ) ≤ pd(M ).
Algunas caracterizaciones de estas dimensiones en términos de los grupos Ext y
T or se presentan a continuación.
Teorema 4.1.3. Sean M un A-módulo y n ≥ −1. Entonces las siguientes condi-
ciones son equivalentes:
(i) pd(M ) ≤ n.
(ii) ExtkA (M, N ) = 0, para cada A-módulo N y k ≥ n + 1.
(iii) Extn+1
A (M, N ) = 0, para cada A-módulo N .

(iv) Para cada resolución proyectiva


fn fn−1 f1 f0
· · · → Pn −→ Pn−1 −−→ Pn−2 → · · · → P1 −
→ P0 −
→ M → 0.
de M , ker(fn−1 ) es proyectivo.
(v) El funtor ExtnA (M, ) es exacto a derecha.
Demostración. Para n = −1, M = 0 y el teorema se cumple trivialmente. Supon-
gamos entonces que M 6= 0 y n ≥ 0.
(i)⇒(ii): por hipótesis M tiene una resolución proyectiva de la forma
fn fn−1 f1 f0
0 → Pn −→ Pn−1 −−→ Pn−2 → · · · → P1 −
→ P0 −
→ M → 0,
y para cada módulo N se obtiene entonces el complejo

f1 ∗
fn
0 → HomA (M, N ) → HomA (P0 , N ) −−→ HomA (P1 , N ) →
− · · · → HomA (Pn−1 , N ) −−→ HomA (Pn , N ) → 0,

luego para cada k ≥ n + 1, ExtkA (M, N ) = 0 con N arbitrario.


(ii)⇒(iii): evidente.
(iii)⇒(iv): consideremos una resolución proyectiva de M :
fn fn−1 f1 f0
· · · → Pn −→ Pn−1 −−→ Pn−2 → · · · → P1 −
→ P0 −
→ M → 0;
entonces para cada módulo N se tiene
4.1. DIMENSIONES PROYECTIVA, INYECTIVA Y PLANA DE UN MÓDULO 117

Extn+1 n n−1 1
A (M, N ) = ExtA (ker(f0 ), N ) = ExtA (ker(f1 ), N ) = · · · = ExtA (ker(fn−1 ), N ).

En efecto, veamos por ejemplo la primera igualdad: de la resolución proyectiva


de M se obtiene la siguiente resolución proyectiva de ker(f0 ),
gn gn−1 g1 g0
· · · → Qn −→ Qn−1 −−→ Qn−2 → · · · → Q1 −
→ Q0 −
→ ker(f0 ) → 0,

en la cual

Qi := Pi+1 , gi := fi+1 , i ≥ 0.

Por lo tanto,

Extn+1 ∗ ∗ ∗ ∗ n
A (M, N ) = ker(fn+2 )/Im(fn+1 ) = ker(gn+1 )/Im(gn ) = ExtA (ker(f0 ), N ).

Similarmente se obtienen las otras igualdades, y entonces Ext1A (ker(fn−1 ), N ) = 0


para cada N , es decir, ker(fn−1 ) es proyectivo.
(iv)⇒(i): consideremos una resolución proyectiva

fn fn−1 f1 f0
· · · → Pn −→ Pn−1 −−→ Pn−2 → · · · → P1 −
→ P0 −
→ M → 0.

de M ; por hipótesis Kn−1 := ker(fn−1 ) es proyectivo, entonces se obtiene la resolu-


ción proyectiva finita

fn fn−1 f1 f0
0 → Kn−1 −→ Pn−1 −−→ Pn−2 → · · · → P1 −
→ P0 −
→ M → 0,

de donde pd(M ) ≤ n.
(iii)⇒(v): consideremos una sucesión exacta de A-módulos 0 → N 0 → N →
00
N → 0, aplicamos la sucesión exacta larga y obtenemos

· · · → ExtnA (M, N 0 ) → ExtnA (M, N ) → ExtnA (M, N 00 ) → Extn+1 0


A (M, N ) = 0,

esto indica que ExtnA (M, ) es exacto a derecha.


(v)⇒(i): consideremos nuevamente una resolución proyectiva

fn fn−1 f1 f0
· · · → Pn −→ Pn−1 −−→ Pn−2 → · · · → P1 −
→ P0 −
→ M → 0.

de M y la siguiente porción exacta: 0 → Kn−1 → Pn−1 → Kn−2 → 0, con


Kn−1 := ker(fn−1 ), Kn−2 := ker(fn−2 ). Sea N un módulo, de la sucesión exacta larga
estractamos la porción HomA (Pn−1 , N ) → HomA (Kn−1 , N ) → Ext1A (Kn−2 , N ) →
Ext1A (Pn−1 , N ), pero como Pn−1 es proyectivo entonces se obtiene la sucesión exacta

HomA (Pn−1 , N ) → HomA (Kn−1 , N ) → Ext1A (Kn−2 , N ) → 0.


118 CAPÍTULO 4. DIMENSIONES DE MÓDULOS Y ANILLOS

α
→ N 0 es un homo-
Además, Ext1A (Kn−2 , N ) = ExtnA (M, N ); por lo tanto, si N −
morfismo sobreyectivo, entonces resulta el siguiente diagrama conmutativo con filas
exactas:
HomA (Pn−1 , N ) −−−→ HomA (Kn−1 , N ) −−−→ ExtnA (M, N ) −−−→ 0
  
α∗ α∗  ∗
y P y K yαn
HomA (Pn−1 , N 0 ) −−−→ HomA (Kn−1 , N 0 ) −−−→ ExtnA (M, N 0 ) −−−→ 0

Como Pn−1 es proyectivo, entonces αP∗ es sobreyectivo, como ExtnA (M, ) es exac-

to a derecha, entonces αn∗ es sobreyectivo, y entonces es fácil probar que αK es
sobreyectivo. Esto indica que Kn−1 es proyectivo y la sucesión

0 → Kn−1 → Pn−1 → Pn−2 → · · · → P1 → P0 → M → 0

es una resolución proyectiva de M , luego pd(M ) ≤ n.

Teorema 4.1.4. Sean N un A-módulo y n ≥ −1. Entonces las siguientes condi-


ciones son equivalentes:

(i) id(N ) ≤ n.

(ii) ExtkA (M, N ) = 0, para cada A-módulo M y k ≥ n + 1.

(iii) Extn+1
A (M, N ) = 0, para cada A-módulo M .

(iv) Para cada resolución inyectiva


f0 f1 f2 fn fn+1
→ E0 −
0→N − → E1 −
→→ · · · → En−1 −→ En −−→ En+1 → · · · .

de N , Im(fn ) es inyectivo.

(v) El funtor ExtnA ( , N ) es exacto a derecha.

Demostración. Ejercicio para el lector.


Los grupos T or pueden definirse usando resoluciones planas tal como se hizo con
resoluciones proyectivas, mostrando también la independencia de la resolución plana
elegida. Puesto que todo módulo proyectivo es plano, entonces las dos definiciones
coinciden (véase [26], Teorema 7.5). Mediante resoluciones planas se tiene un teorema
análogo a los anteriores para la dimensión plana.

Teorema 4.1.5. Sean M un A-módulo y n ≥ −1. Entonces las siguientes condi-


ciones son equivalentes:

(i) fd(M ) ≤ n.
4.1. DIMENSIONES PROYECTIVA, INYECTIVA Y PLANA DE UN MÓDULO 119

(ii) T orkA (M, N ) = 0, para cada A-módulo izquierdo N y k ≥ n + 1.


A
(iii) T orn+1 (M, N ) = 0, para cada A-módulo izquierdo N .

(iv) Para cada resolución plana


fn fn−1 f1 f0
· · · → Pn −→ Pn−1 −−→ Pn−2 → · · · → P1 −
→ P0 −
→ M → 0.

de M , ker(fn−1 ) es plano.

(v) El funtor T ornA (M, ) es exacto a izquierda.

Demostración. Ejercicio para el lector.

Proposición 4.1.6. Sean A un anillo, 0 → M 0 → M → M 00 → 0 una sucesión


exacta de A-módulos y n ≥ 0. Entonces,

(i) pd(M 00 ) ≤ máx{pd(M 0 ), pd(M )} + 1.

(ii) Si M es proyectivo, entonces M 0 y M 00 son proyectivos ó pd(M 00 ) = pd(M 0 )+1.

(iii) Si pd(M 00 ) ≤ n y pd(M 0 ) ≤ n, entonces pd(M ) ≤ n.

(iv) Si pd(M 00 ) ≤ n y pd(M 0 ) = n, entonces pd(M ) = n.

(v) Si pd(M 0 ) > pd(M ), entonces pd(M 00 ) = pd(M 0 ) + 1.

(vi) pd(M ) ≤ máx{pd(M 00 ), pd(M 0 )}.

Demostración. (i) Si pd(M 0 ) = ∞ o pd(M ) = ∞, entonces claramente (i) se cumple.


Supongamos pues que pd(M ), pd(M 0 ) < ∞ y sea m := máx{pd(M 0 ), pd(M )}. Para
cada A-módulo N y cada k ≥ m + 1 se tiene que ExtkA (M 0 , N ) = 0 = ExtkA (M, N ),
luego en la sucesión exacta larga, aplicada a la sucesión exacta del enunciado de la
proposición, se tiene que

· · · → Extm+1
A (M 0 , N ) −
→ Extm+2
A (M 00 , N ) −
→ Extm+2
A (M, N ) → · · ·

con Extm+1A (M 0 , N ) = 0 = Extm+2A (M, N ), por lo tanto, Extm+2


A (M 00 , N ) = 0 para
cada N , es decir, pd(M 00 ) ≤ m + 1.
(ii) Si M 00 es proyectivo, entonces M ∼ = M 0 ⊕ M 00 y ası́ M 0 es proyectivo. Si M 00
no es proyectivo, entonces M 6= 0 y pd(M 00 ) ≥ 1; además, M 0 6= 0, de lo contrario
00

M 00 ∼
= M serı́a proyectivo. Consideremos dos casos posibles:
Caso 1. pd(M 0 ) = ∞, debemos ver que pd(M 00 ) = ∞. Supongamos lo con-
trario, sea pd(M 00 ) = m, entonces Extm+1A (M 00 , N ) = 0 para cada módulo N , luego
0 0
Extm A (M , N ) = 0, es decir, pd(M ) ≤ m − 1, falso.
120 CAPÍTULO 4. DIMENSIONES DE MÓDULOS Y ANILLOS

Caso 2. pd(M 0 ) es finita, digamos pd(M 0 ) := n ≥ 1. Sea N un A-módulo; se


tiene la sucesión exacta larga resulta

· · · → ExtkA (M, N ) → ExtkA (M 0 , N ) → Extk+1 00 k+1


A (M , N ) → ExtA (M, N ) → · · ·

pero como M es proyectivo entonces ExtkA (M 0 , N ) ∼ = Extk+1 00


A (M , N ) para cada
k ≥ 1, en particular 0 = Extn+1 0 ∼ n+2 00 00
A (M , N ) = ExtA (M , N ), luego pd(M ) ≤ n + 1.
Supongamos que pd(M 00 ) ≤ n, entonces ExtnA (M 0 , N ) ∼ = Extn+1 00
A (M , N ) = 0, es
decir, pd(M ) ≤ n − 1, lo cual es falso. Por lo tanto, pd(M ) = n + 1 = pd(M 0 ) + 1.
0 00

(iii) Por hipótesis, Extn+1 0 n+1 00


A (M , N ) = 0 = ExtA (M , N ) para cada módulo N ;
de la sucesión exacta larga resulta
00 0
· · · → Extn+1 n+1 n+1
A (M , N ) → ExtA (M, N ) → ExtA (M , N ) → · · · ,

luego Extn+1
A (M, N ) = 0, es decir, pd(M ) ≤ n.
(iv) Según (iii) se tiene que pd(M ) ≤ n; puesto que pd(M 0 ) = n, exite un A-
módulo N0 tal que ExtnA (M 0 , N0 ) 6= 0 (de lo contrario pd(M 0 ) ≤ n − 1). Se tiene la
sucesión exacta larga
α
→ ExtnA (M 0 , N0 ) −
→ ExtnA (M, N0 ) −
··· − → Extn+1 00
A (M , N0 ) −
→ ··· ,

pero Extn+1 00 n
A (M , N0 ) = 0 y entonces α es sobreyectivo. Resulta ExtA (M, N0 ) 6= 0,
de donde pd(M ) ≥ n, es decir, pd(M ) = n.
(v) Si pd(M 0 ) > pd(M ), entonces pd(M ) es finita; sea m := pd(M ), según (i)
pd(M 00 ) ≤ pd(M 0 ) + 1. Supongamos que pd(M 00 ) < pd(M 0 ) + 1, entonces pd(M 00 )
es finita. Sea m00 := pd(M 00 ), entonces para cada k ≥ m00 + 1 y cada módulo N se
tiene que ExtkA (M 00 , N ) = 0, luego de la suceción exacta larga

· · · → ExtkA (M 00 , N ) → ExtkA (M, N ) → ExtkA (M 0 , N ) → Extk+1 00


A (M , N ) → · · · ,

resulta ExtkA (M, N ) ∼


= ExtkA (M 0 , N ) para cada k ≥ m00 + 1. En particular, si m0 :=
máx{m, m00 } y k ≥ m0 + 1, entonces ExtkA (M 0 , N ) = 0 para cada N , luego m =
pd(M ) < pd(M 0 ) ≤ m0 = máx{m, m00 }, lo cual solo es posible si m0 = m00 , de donde
pd(M 0 ) ≤ pd(M 00 ) < pd(M 0 ) + 1, es decir, pd(M 0 ) = pd(M 00 ) = m00 , y por (iv),
pd(M ) = m00 , pero esto es falso ya que pd(M ) < pd(M 0 ).
(vi) Si pd(M 0 ) = ∞ o pd(M 00 ) = ∞, entonces pd(M ) ≤ ∞. Supongamos en-
tonces que pd(M 0 ) y pd(M 00 ) son ambas finitas y sea m := máx{pd(M 0 ), pd(M 00 )};
para cualquier A-módulo N se tiene que

· · · → Extm+1
A (M 00 , N ) → Extm+1
A (M, N ) → Extm+1
A (M 0 , N ) → · · · ,

resulta Extm+1
A (M, N ) = 0, y por lo tanto pd(M ) ≤ m.
4.1. DIMENSIONES PROYECTIVA, INYECTIVA Y PLANA DE UN MÓDULO 121

Versiones análogas de las propiedades presentadas en la proposición anterior se


pueden establecer para las dimensiones inyectiva y plana. En particular podemos
enunciar las siguientes.

Proposición 4.1.7. Sean A un anillo, 0 → N 0 → N → N 00 → 0 una sucesión


exacta de A-módulos y n ≥ 0. Entonces,

(i) Si N es inyectivo, entonces N 0 y N 00 son inyectivos ó id(N 0 ) = id(N 00 ) + 1.

(ii) Si id(N 0 ) ≤ n y id(N 00 ) ≤ n, entonces id(N ) ≤ n.

Demostración. Ejercicio para el lector.

Proposición 4.1.8. Sean A un anillo, 0 → M 0 → M → M 00 → 0 una sucesión


exacta de A-módulos y n ≥ 0.

(i) Si fd(M 00 ) ≤ n y fd(M 0 ) ≤ n, entonces fd(M ) ≤ n.

(ii) Si fd(M 00 ) ≤ n y fd(M 0 ) = n, entonces fd(M ) = n.

Demostración. (i) Para cada A-módulo izquierdo L se tiene la sucesión exacta larga
A
· · · → T orn+1 (M 0 , L) → T orn+1
A A
(M, L) → T orn+1 (M 00 , L) → · · · ,

pero T orn+1A
(M 0 , L) = 0 = T orn+1 A
(M 00 , L), luego T orn+1
A
(M, L) = 0, de donde
fd(M ) ≤ n.
(ii) Por (i) se tiene que fd(M ) ≤ n; existe un A-módulo izquierdo L0 para el cual
T ornA (M 0 , L0 ) 6= 0; con la sucesión exacta larga del T or se tiene
A
· · · → T orn+1 (M 00 , L0 ) → T ornA (M 0 , L0 ) → T ornA (M, L0 ) → · · · ,
A
con T orn+1 (M 00 , L0 ) = 0, y como T ornA (M 0 , L0 ) 6= 0 entonces T ornA (M, L0 ) 6= 0. Esto
garantiza que fd(M ) ≥ n, luego fd(M ) = n.

Otras propiedades interesantes de la dimensión proyectiva de un módulo son las


siguientes.

Proposición 4.1.9. Sea M un A-módulo no nulo tal que pd(M ) = n ≥ 1. Entonces


existe un A-módulo libre F tal que ExtnA (M, F ) 6= 0.

Demostración. Supongamos que ExtnA (M, F ) = 0 para cada módulo libre F ; sea N
un A-módulo arbitrario, entonces N ∼
= F/K, con F libre; se tiene la sucesión exacta
0 → K → F → F/K → 0; aplicamos la sucesión exacta larga y obtenemos
· · · → ExtnA (M, K) → ExtnA (M, F ) → ExtnA (M, N ) → Extn+1
A (M, K) → · · · .
122 CAPÍTULO 4. DIMENSIONES DE MÓDULOS Y ANILLOS

Por hipótesis y con el toerema 4.1.3 se tiene que ExtnA (M, F ) = 0 = Extn+1
A (M, K),
n
entonces ExtA (M, N ) = 0, para cada N , en consecuencia pd(M ) ≤ n − 1, lo cual es
falso.
Proposición 4.1.10. Sea {Mi }i∈C una familia de A-módulos. Entonces,
L
pd( i∈C Mi ) = sup{pd(Mi )}i∈C .
Demostración. Consideremos dos casos posibles. L
(i) sup{pd(MLi )}i∈C = ∞, veamos que pd( i∈Cm+1 = ∞. Supongamos lo con-
Mi )L
trario, sea pd( i∈C MQ i ) := m < ∞, entonces ExtA ( i∈C Mi , N ) = 0 para cada
módulo N , luego 0 = i∈C Extm+1 A (M i , N ) = 0, de donde Extm+1
A (Mi , N ) = 0 para
cada N y cada i, esto indica que pd(Mi ) ≤ m para cada i, luego sup{pd(Mi )}i∈C ≤
m, falso. L
(ii) Sea sup{pd(M i )} i∈C := m < ∞. Probemos entonces que pd( Li∈C Mi ) = m:
se tieneL m+1
L ∼ Q m+1
ExtA ( i∈C Mi , N ) = i∈C ExtA (Mi , N ) = 0, luego pd( i∈C Mi ) ≤ m;
sea pd( i∈C Mi ) := n, hemos L entonces demostrado Q que nn+1≤ m.
n+1
Si tiene que ExtA ( i∈C Mi , N ) = 0 = i∈C ExtA (Mi , N ), por lo tanto
n+1
ExtA (Mi , N ) = 0 para cada i y cada N , por lo tanto, pd(Mi ) ≤ n, con lo cual
sup{pd(Mi )}i∈C ≤ n, es decir, m ≤ n, luego m = n.

4.2. Dimensión global de un anillo


Definición 4.2.1. Sea A un anillo, la dimensión global proyectiva a derecha
de A se denota por rpgld(A), y se define por
rpgld(A) := sup{pd(M )| M es un A-módulo a derecha}.
En forma similar se define la dimensión global inyectiva a derecha de A
y se denota por rigld(A).
Teorema 4.2.2. Sea A un anillo. Entonces, rpgld(A) = rigld(A).
Demostración. Consideremos dos casos. Asumamos que rpgld(A) = ∞. Debemos
ver que rigld(A) = ∞, pero supongamos lo contrario, sea rigld(A) := n < ∞,
entonces para cada módulo N se tiene que id(N ) ≤ n con lo cual para cada módulo
M se obtiene que Extn+1A (M, N ) = 0, esto garantiza que pd(M ) ≤ n, de donde
rpgld(A) ≤ n, falso.
Supongamos ahora que rpgld(A) := n < ∞. Sea M un A-módulo arbitrario, en-
tonces pd(M ) ≤ n y para cada módulo N se tiene que Extn+1A (M, N ) = 0. Entonces,
id(N ) ≤ n, con lo cual rigld(A) ≤ n. Sea rigld(A) := m, tenemos pues demostrado
que m ≤ n. Se tiene también que id(N ) ≤ m para cada módulo N , luego para cada
módulo M resulta Extm+1
A (M, N ) = 0, es decir, pd(M ) ≤ m, luego rpgld(A) ≤ m,
es decir, n ≤ m, de donde n = m.
4.2. DIMENSIÓN GLOBAL DE UN ANILLO 123

De este resultado se tiene entonces la siguiente definición.

Definición 4.2.3. Sea A un anillo, la dimensión global a derecha de A se


define por rgld(A) := rpgld(A) = rigld(A).

En forma análoga se definen a izquierda las dimensiones lpgld(R) y ligld(R), y por


supuesto, se tiene también lgld(R) := lpgld(R) = ligld(R), denominada dimensión
global a izquierda de R. Existen anillos R para los cuales rgld(R) 6= lgld(R) (véase
el ejercicio 6). Para anillos en los cuales rgld(R) = lgld(R), se define la dimensión
global simplemente como gld(A) := rgld(R) = lgld(R). Esto se tiene por ejemplo
para anillos que son simultáneamente noetherianos a derecha e izquierda (véase el
teorema 4.4.1 más adelante), y también desde luego para anillos conmutativos.
Probaremos ahora un resultado relevante en álgebra homológica.

Teorema 4.2.4 (Teorema de Auslander). Sea A un anillo. Entonces,

rgld(A) = sup{pd(A/I)|Ies un ideal derecho de A}.

Demostración. Sea I un ideal derecho de A, entonces A/I es un A-módulo y se tiene


que sup{pd(A/I)|I es un ideal derecho de A} ≤ rgld(A). Si este sup es infinito,
entonces rgld(A) = ∞. Sea pues sup{pd(A/I)|I es un ideal derecho de A} := n <
∞; hemos probado que n ≤ rgld(A). Puesto que el sup es n, para cada ideal derecho
I de A y cada módulo N se tiene que Extn+1 A (A/I, N ) = 0. Consideremos una
resolución inyectiva de N
f0 f1 fn fn+1
0→N −
→ E0 −
→ E1 → · · · → En−1 −→ En −−→ En+1 → · · ·
y sea Hn := Im(fn ), entonces
Extn+1 n n−1 1
A (A/I, N ) = ExtA (A/I, H1 ) = ExtA (A/I, H2 ) = · · · = ExtA (A/I, Hn ),

luego Ext1A (A/I, Hn ) = 0, es decir, Ext1A (M, Hn ) = 0 para cada módulo cı́clico M .
Aplicamos el teorema 2.2.3 y obtenemos que Hn es inyectivo. Resulta entonces la
resolución inyectiva
f0 f1 fn
0→N −
→ E0 −
→ E1 → · · · → En−1 −→ Hn → 0,
luego id(N ) ≤ n para cada módulo N . Por el teorema 4.2.2, rgld(A) ≤ n.

Concluimos esta sección con algunas aplicaciones del teorema de Auslander.

Ejemplo 4.2.5. (i) Sea A un anillo. Entonces, A es semisimple a derecha si, y sólo
si, rgld(A) = 0 si, y sólo si, A es semisimple a izquierda si, y sólo si, lgld(A) = 0.
Basta probar la afirmación por el lado derecho ya que un anillo es semisimple a
derecha si, y sólo si, es semisimple a izquierda (véase [18]).
124 CAPÍTULO 4. DIMENSIONES DE MÓDULOS Y ANILLOS

⇒): sea I un ideal derecho de A; si I = A, entonces A/I = 0 y entonces


pd(A/I) = −1. Sea I 6= A, entonces A ∼ = I ⊕ A/I y A/I resulta proyectivo, por lo
tanto, pd(A/I) = 0. El teorema de Auslander garantiza que rgld(A) = 0.
⇐): sea I un ideal derecho de A; si I = A, entonces A es sumando directo
de A. Sea I 6= A, entonces A/I 6= 0 y en consecuencia pd(A/I) = 0, luego A/I
es proyectivo; al considerar el homomorfismo canónico A → A/I encontramos que
A∼ = I ⊕ A/I, es decir, I es sumando directo de A.
(ii) Ası́ pues, la dimensión global de los anillos de división es cero.
(iii) Otra conclusión que se puede sacar de (i) es la siguiente: un anillo A es
semisimple si, y sólo si, cada A-módulo derecho es proyectivo si, y sólo si, cada A-
módulo izquierdo es proyectivo si, y sólo si, cada A-módulo derecho es inyectivo si,
y sólo si, cada A-módulo izquierdo es inyectivo.
Ejemplo 4.2.6. Sea R un dominio de integridad. Entonces, R es un cuerpo si,
y sólo si, gld(R) = 0. En efecto, si R es un cuerpo, entonces R es semisimple, y
por el ejemplo anterior, gld(R) = 0. Recı́procamente, si gld(R) = 0, entonces R
es semisimple, pero un anillo conmutativo semisimple es producto finito de cuerpos
([18]), pero como R no tiene divisores de cero, entonces R es un cuerpo.
Ejemplo 4.2.7. Sea A un anillo. Entonces, A es hereditario a derecha si, y sólo si,
rgld(A) ≤ 1. En particular, si R es un dominio de integridad, R es un dominio de
Dedekind si, y sólo si, gld(R) ≤ 1.
⇒): sea I un ideal derecho de A, entonces I es proyectivo; si A/I es proyectivo,
pd(A/I) = 0, pero si A/I no es proyectivo, aplicamos la proposición 4.1.6 a la
sucesión exacta 0 → I → A → A/I → 0 y obtenemos que pd(A/I) = pd(I) + 1 = 1.
Por el teorema de Auslander, rgld(A) ≤ 1.
⇐): sea I 6= 0 un ideal derecho de A, como A es proyectivo aplicamos nuevamente
la proposición 4.1.6 y obtenemos que I y A/I son proyectivos o pd(A/I) = pd(I) +
1. En el primer caso I es proyectivo y hemos terminado. Para el segundo caso
recordemos que gld(A) ≤ 1, luego por el teorema de Auslander pd(A/I) ≤ 1, es
decir, pd(I) ≤ 0, pero como I 6= 0, entonces pd(I) = 0 e I resulta proyectivo.
En particular, si R es un dominio de ideales principales que no es cuerpo, entonces
rgld(R) = 1. Por ejemplo, gld(Z) = 1 = gld(K[x]), K un cuerpo.

4.3. Dimensión global débil de un anillo


La dimensión global débil a derecha de un anillo, notada rwgld(A), se define
por
rwgld(A) := sup{fd(M )| M es un A-módulo a derecha}.
En forma similar, y por medio de módulos a izquierda, se define la dimensión global
débil a izquierda del anillo A. Se tiene sin embargo la siguiente propiedad.
4.3. DIMENSIÓN GLOBAL DÉBIL DE UN ANILLO 125

Teorema 4.3.1. Sea A un anillo. Entonces, rwgld(A) = lwgld(A).

Demostración. Consideremos dos casos. Asumamos ahora que rwgld(A) = ∞. Debe-


mos ver que lwgld(A) = ∞, pero supongamos lo contrario, sea lwgld(A) := n < ∞,
entonces para cada módulo izquierdo N se tiene que fd(N ) ≤ n con lo cual para cada
A
módulo derecho M se obtiene que T orn+1 (M, N ) = 0, esto garantiza que fd(M ) ≤ n,
de donde rwgld(A) ≤ n, falso.
Supongamos ahora que rwgld(A) := n < ∞. Sea M un A-módulo derecho
arbitrario, entonces fd(M ) ≤ n y para cada módulo izquierdo N se tiene que
A
T orn+1 (M, N ) = 0. Entonces, fd(N ) ≤ n, con lo cual lwgld(A) ≤ n. Sea lwgld(A) :=
m, hemos pues demostrado que m ≤ n. Se tiene además que para cada módulo
módulo izquierdo N , fd(N ) ≤ m, luego para cada módulo derecho M se tiene que
A
T orm+1 (M, N ) = 0, es decir, fd(M ) ≤ m, de donde rwgld(A) ≤ m, es decir, n ≤ m,
con lo cual n = m.

El teorema anterior permite definir la dimensión global débil de un anillo A.

Definición 4.3.2. Sea A un anillo, la dimensión global débil de A se define


por wgld(A) := rwgld(A) = lwgld(A)

Corolario 4.3.3. Sea A un anillo. Entonces, wgld(A) ≤ mı́n{rgld(A), lgld(A)}.

Demostración. Esto se sigue del hecho que todo módulo proyectivo es plano.

Veremos más adelante que las igualdades se tienen en anillos noetherianos a


derecha e izquierda, respectivamente. Cerramos esta sección demostrando la versión
débil del teorema de Auslander.

Teorema 4.3.4. Sea A un anillo. Entonces,

wgld(A) = sup{fd(A/I)|Ies un ideal derecho de A}


= sup{fd(A/I)|Ies un ideal izquierdo de A}.

Demostración. Sea I un ideal derecho de A, entonces A/I es un A-módulo y se tiene


que sup{fd(A/I)|I es un ideal derecho de A} ≤ wgld(A). Si este sup es infinito,
entonces wgld(A) = ∞. Sea pues sup{fd(A/I)|I es un ideal derecho de A} := n <
∞; hemos probado que n ≤ wgld(A). Puesto que el sup es n, para cada ideal derecho
A
I de A y cada módulo izquierdo L se tiene que T orn+1 (A/I, L) = 0. Consideremos
una resolución plana de L
fn+1 fn f1 f0
· · · → Pn+1 −−→ Pn −→ Pn−1 → · · · → P1 −
→ P0 −
→L→0
y sea Kn−1 := ker(fn−1 ), entonces
126 CAPÍTULO 4. DIMENSIONES DE MÓDULOS Y ANILLOS

A
T orn+1 (A/I, L) ∼
= T or1A (A/I, Kn−1 ),
luego T or1A (A/I, Kn−1 ) = 0. Aplicamos el teorema 3.3.1 y obtenemos que Kn−1 es
plano, con lo cual se tiene la resolución plana
f1 f0
0 → Kn−1 → Pn−1 → · · · → P1 −
→ P0 −
→ L → 0,
en consecuencia fd(L) ≤ n para cada módulo L. Por lo tanto, wgld(A) ≤ n.
La prueba por el lado izquierdo es análoga.

4.4. Dimensión global de anillos noetherianos


Teorema 4.4.1. Sea A un anillo. Entonces,
(i) Si A es noetheriano a derecha, entonces wgld(A) = rgld(A).

(ii) Si A es noetheriano a izquierda, entonces wgld(A) = lgld(A).

(iii) Si A es noetheriano a derecha e izquierda, entonces

rgld(A) = wgld(A) = lgld(A).

(iv) Si R es un anillo conmutativo noetheriano, entonces wgld(R) = gld(R).


Demostración. Veamos la prueba de (i), la de (ii) es análoga; (iii) y (iv) resultan de
(i) y (ii). Por el corolario 3.3.8, para cada A-módulo f.g. M se tiene que fd(M ) =
pd(M ). Pero según los teoremas 4.2.4 y 4.3.4, el cálculo de las dimensiones global
derecha y débil basta realizarlo sobre módulos cı́clicos, luego el resultado se obtiene
inmediatamente.
Corolario 4.4.2. Sea A un anillo. Entonces,
(i) Si A es artiniano a derecha, entonces wgld(A) = rgld(A).

(ii) Si A es artiniano a izquierda, entonces wgld(A) = lgld(A).

(iii) Si A es artiniando a derecha e izquierda, entonces

rgld(A) = wgld(A) = lgld(A).

(iv) Si R es un anillo conmutativo artiniano, entonces wgld(R) = gld(R).


Demostración. Esto es consecuencia directa del teorema anterior y del teorema de
Hopkins-Akizuki el cual establece que cada anillo artiniano de derecha (izquierda)
es noetheriano a derecha (izquierda), véase [18].
4.4. DIMENSIÓN GLOBAL DE ANILLOS NOETHERIANOS 127

Ejemplo 4.4.3. Sea n ≥ 2, recordemos que Zn es semisimple si, y sólo si, n es libre
de cuadrados (véase [18]). En este caso se tiene que gld(Zn ) = wgld(Zn ) = 0.

El ejemplo anterior plantea una preguna muy elemental: si n no es libre de


cuadrados, ¿cuál es la dimensión global de gld(Zn )?. Cerramos esta sección con un
par de resultados que permiten responder esta pregunta.

Definición 4.4.4. Sea R un anillo conmutativo; se dice que R es autoinyectivo


(también denominado quasi-Frobenius) si R es noetheriano y RR es un módulo
inyectivo.

Teorema 4.4.5. Sea R un anillo conmutativo noetheriano. R es autoinyectivo si,


y sólo si, cada R-módulo proyectivo es inyectivo.

Demostración. ⇒): sea P un R-módulo proyectivo, entonces existe un módulo libre


R(X) tal que P es sumando directo de R(X) . Como RR es inyectivo y R es noetheriano,
el corolario 1.7.7 garantiza que R(X) es inyectivo, luego por la proposición 1.7.2, P
resulta inyectivo.
⇐): evidente ya que RR es proyectivo.

Corolario 4.4.6. Sea R un anillo autoinyectivo. Entonces, gld(R) = 0 ó ∞. En el


primer caso R es semisimple.

Demostración. Se presentan dos casos: existe un R-módulo M tal que pd(M ) = ∞,


entonces gld(R) = ∞; o la otra opción es que cada R-módulo tenga dimensión
proyectiva finita. Supongamos que existe un módulo M tal que pd(M ) := n ≥ 1.
Entonces, M 6= 0 y, por la proposición 4.1.9, existe un módulo libre F tal que
ExtnA (M, F ) 6= 0, pero esto es contradictorio ya que la hipótesis y el teorema 4.4.5
garantizan que F es inyectivo. En conlusión, pd(M ) ≤ 0 para cada M , es decir,
gld(R) = 0.

Ejemplo 4.4.7. Sea R un dominio de ideales principales y sea I := hai un ideal no


nulo de R; entonces R/I es autoinyectivo. En efecto, notemos que R/I es un anillo
noetheriano; probemos que R/I es un R/I-módulo inyectivo. Sea J/I un ideal de
R/I, con J := hbi ⊇ hai, resulta a = bc, con c ∈ R; sea f : J/I → R/I un
R/I-homomorfismo, entonces se tiene el siguiente diagrma conmutativo

R/I
p
6 Ip p
f p pg
pp
pp
J/I ι
- R/I
128 CAPÍTULO 4. DIMENSIONES DE MÓDULOS Y ANILLOS

donde ι es la inclusión y g se define de la siguiente manera: sea f (b) := s, tenemos


f (b c) = f (b)c = f (a) = 0, luego s c = 0 y en consecuencia sc ∈ I, de donde
sc = ar = bcr, con r ∈ R; definimos g(1) := r y se tiene que gι = f . En efecto,
notemos que c 6= 0 ya que a 6= 0, por lo tanto s = rb, es decir, s = b r, con lo cual
gι(b) = g(b) = g(1b) = g(1)b = rb = s = f (b).

Ejemplo 4.4.8. Sea n ≥ 2, entonces Zn es autoinyectivo, por lo tanto gld(Zn ) = 0


ó ∞. Si n es libre de cuadrados, entonces Zn es semisimple y gld(Zn ) = 0; en otro
caso gld(Zn ) = ∞. Por ejemplo, gld(Z4 ) = ∞.

4.5. Anillos con dimensión débil ≤ 1


A continuación se caracterizan los anillos con dimensión global débil ≤ 1. Comence-
mos con los de dimensión cero.

Definición 4.5.1. Un anillo A es regular de von Neumann a derecha si cada


A-módulo derecho es plano.

Se tiene entonces que A es regular de von Neumann a derecha si, y sólo si,
wgld(A) = 0, y por lo tanto, A es regular de von Neumann a derecha si, y sólo si, A
es regular de von Neumann a izquierda.

Ejemplo 4.5.2. Todo anillo semisimple A es regular de von Neumann ya que A es


artiniano y por lo tanto 0 = gld(A) = wgld(A).

Teorema 4.5.3. Sea A un anillo. Entonces las siguientes condiciones son equiva-
lentes:

(i) wgld(A) ≤ 1.

(ii) Cada submódulo de un A-módulo plano es plano.

(iii) Cada ideal derecho de A es plano.

(iv) Cada ideal izquierdo de A es plano.

(v) Para cada A-módulo derecho M y cada A-módulo izquierdo N , T or2A (M, N ) =
0.

Demostración. (i) ⇒(ii): sea M un A-módulo plano y sea M 0 un submódulo de


M ; como wgld(A) ≤ 1, fd(M/M0 ) ≤ 1. Consideremos la resolución plana de M/M 0
f0
· · · → M 00 → M − → M/M 0 → 0; según el teorema 4.1.5, ker(f0 ) es plano, es decir,
0
M es plano.
(ii) ⇒(iii): puesto que AA es plano, entonces cada ideal derecho de A es plano.
4.6. DIMENSIÓN GLOBAL Y EXTENSIONES DE ANILLOS 129

(iii) ⇒(i): sea I un ideal derecho de A, por hipótesis I es plano y se tiene la


resolución plana 0 → I → A → A/I → 0, luego fd(A/I) ≤ 1, y por el teorema 4.3.4,
wgld(A) ≤ 1.
(i)⇔(iv): la prueba es análoga a la anterior pero usando la versión derecha del
teorema 4.3.4.
(i)⇔(v): si wgld(A) ≤ 1, entonces fd(M ) ≤ 1 para cada módulo M , luego
2
T orA (M, N ) = 0 para cada módulo izquierdo N . Recı́procamente, la condición en
(v) dice que wgld(A) ≤ 1.
Ejemplo 4.5.4. Sea R un dominio de integridad; se dice que R es un dominio de
Prüfer si cada ideal f.g. de R es proyectivo. En [27] se prueba que R es un dominio
de Prüfer si, y sólo si, wgld(R) ≤ 1 (véase también [5]).

4.6. Dimensión global y extensiones de anillos


Sea ι : R → A un homomorfismo de anillos, en esta sección revisaremos la relación
entre las dimensiones de los anillos R y A. Los resultados que probemos podrán por
lo tanto ser aplicados al caso particular en el cual ι es inyectivo, es decir, cuando A
es una extensión de R.
Para comenzar debemos observar que no siempre rgld(R) ≤ rgld(A) ni tampoco
rgld(A) ≤ rgld(R).
Ejemplo 4.6.1. (i) Z → Q, con gld(Z) = 1 y gld(Q) = 0.
(ii) Tenemos el homomorfismo canónico Z → Z4 con gld(Z) = 1 y gld(Z4 ) = ∞.
Proposición 4.6.2. Sea ι : R → A un homomorfismo de anillos y MA un módulo.
Entonces,
(i) pd(MR ) ≤ pd(MA ) + pd(AR ).

(ii) fd(MR ) ≤ fd(MA ) + fd(AR ).


Demostración. (i) Si pd(MA ) = ∞, entonces no hay algo que se deba demostrar. Sea
pues pd(MA ) = n < ∞. La pueba entonces se hace por inducción sobre n. Si n = −1
entonces MA = 0 = MR y la desigualdad se tiene trivialmente. Si n = 0, entonces
MA es proyectivo y por tanto es sumando directo de un A-módulo libre FA = A(I) ,
(I)
digamos MA ⊕ MA0 = A(I) , con lo cual MR ⊕ MR0 = AR . Sabemos que la dimensión
proyectiva de una suma directa es el sup de las dimensiones proyectivas de los
(I)
sumandos, luego pd(MR ) ≤ pd(AR ) = pd(AR ) = 0 + pd(AR ) = pd(MA ) + pd(AR ).
Supongamos que n > 0, entonces MA no es proyectivo; existe un módulo libre
FA = A(I) y una sucesión exacta 0 → KA → FA → MA → 0. Según la proposición
4.1.6 (ii), pd(KA ) = n − 1. Por inducción, pd(KR ) ≤ n − 1 + pd(AR ); pero pd(AR ) =
(I)
pd(AR ) = pd(FR ), entonces podemos aplicar la proposición 4.1.6 (i) y obtenemos
130 CAPÍTULO 4. DIMENSIONES DE MÓDULOS Y ANILLOS

que pd(MR ) ≤ máx{pd(KR ), pd(FR )} + 1 = máx{pd(KR ), pd(AR )} + 1 ≤ máx{n −


1 + pd(AR ), pd(AR )} + 1 = n − 1 + pd(AR ) + 1 = n + pd(AR ) = pd(MA ) + pd(AR ).
(ii) La prueba completa de este punto requiere de varios preliminares que nos
apartan demasiado del tema que nos ocupa y se puede consultar en [23].

Definición 4.6.3. Un módulo plano R F es fielmente plano si para cada módulo


LR se cumple que

L ⊗R F = 0 ⇔ L = 0.

Por ejemplo, todo módulo libre no nulo es fielmente plano. Una caracterización
de los módulos fielmente planos se presenta en la siguiente proposición.

Proposición 4.6.4. Sea R F plano. Entonces, las siguientes condiciones son equi-
valentes:

(i) RF es fielmente plano

(ii) IF 6= F para cada ideal derecho propio I de R

(iii) JF 6= F para cada ideal maximal derecho J de R.

Demostración. (i)⇒(ii): Como R F es fielmente plano y R/I es no nulo, entonces


(R/I) ⊗ F ∼ = F/IF es no nulo, luego IF 6= F .
(ii)⇒(i): sean LR 6= 0 y L0 cualquier submódulo cı́clico no nulo de L. Entonces,
L0 ∼
= R/I, con I ideal derecho propio de R. Por hipótesis, IF 6= F , luego (R/I)⊗F 6=
0, es decir, L0 ⊗ F 6= 0. Como F es plano y L0 ,→ L es inyectiva, entonces entonces
L ⊗ F 6= 0. Esto demuestra que F es fielmente plano.
(ii)⇒(iii) es evidente.
(iii)⇒(ii): sea I ideal derecho propio de R, entonces existe J maximal derecho
tal que I ⊆ J, luego IF ⊆ JF . Si IF = F , entonces F = JF .

Corolario 4.6.5. Sea R F plano. Entonces, R F es fielmente plano si, y sólo si, para
cada módulo simple MR , M ⊗ F 6= 0.

Demostración. ⇒): evidente.


⇐): sea J un ideal maximal derecho de R, entonces R/J es un R-módulo sim-
ple, luego (R/J) ⊗R F ∼= F/JF 6= 0, es decir, JF 6= F . El resultado es entonces
consecuencia de la proposición anterior.

Corolario 4.6.6. Sean ι : R → A un homomorfismo de anillos y R A plano. R A es


fielmente plano si, y sólo si, cada módulo simple MR se tiene que M ⊗R A 6= 0.

Demostración. Consecuencia directa del corolario anterior.


4.6. DIMENSIÓN GLOBAL Y EXTENSIONES DE ANILLOS 131

Proposición 4.6.7. Sean ι : R → A un homomorfismo de anillos, R A fielmente


plano y MR un módulo. Entonces, la función canónica M → M ⊗R A, m 7→ m ⊗ 1,
es inyectiva.

Demostración. Sea α : M → M ⊗R A la función canónica y sea K := ker(α). Sean


x ∈ K y a ∈ A, entonces α(x) = x⊗1 = 0 y (x⊗1)a = x⊗a = 0, resulta K ⊗A = 0,
de donde K = 0, es decir, α es inyectiva.

Teorema 4.6.8. Sean ι : R → A un homomorfismo de anillos, rgld(R) < ∞ y R A


fielmente plano.

(i) Si AR es proyectivo, entonces rgld(R) ≤ rgld(A).

(ii) Si R es noetheriano a derecha y AR es plano, entonces rgld(R) ≤ rgld(A).

Demostración. (i) Sea n := rgld(R); elegimos un módulo MR tal que pd(MR ) = n,


según la proposición anterior podemos construir la sucesión exacta de R-módulos
0 → M → M ⊗R A → C → 0, con C := (M ⊗R A)/M . Para esta sucesión se tiene
que pd(M ) = n y pd(C) ≤ n ya que C es un R-módulo; podemos entonces aplicar la
proposición 4.1.6 (iv) y concluir que pd((M ⊗R A)R ) = n, pero de la proposición 4.6.2
(i) se obtiene que n = pd((M ⊗R A)R ) ≤ pd((M ⊗R A)A ) + pd(AR ) = pd((M ⊗R
A)A ) + 0 = pd((M ⊗R A)A ). Ası́ pues, al menos un A-módulo tiene dimensión
proyectiva mayor o igual a n, esto demuestra que rgld(A) ≥ n = rgld(R).
(ii) Sea n := wgld(R) = rgld(R); elegimos un módulo MR tal que fd(MR ) = n.
Según la proposición anterior podemos construir la sucesión exacta de R-módulos
0 → M → M ⊗R A → C → 0, con C := (M ⊗R A)/M . Para esta sucesión se
tiene que fd(M ) = n y fd(C) ≤ n ya que C es un R-módulo; podemos entonces
aplicar la proposición 4.1.8 (ii) y concluir que fd((M ⊗R A)R ) = n, pero de la
proposición 4.6.2 (ii) se obtiene que n = fd((M ⊗R A)R ) ≤ fd((M ⊗R A)A )+fd(AR ) =
fd((M ⊗R A)A )+0 = fd((M ⊗R A)A ). Ası́ pues, al menos un A-módulo tiene dimensión
plana mayor o igual a n, esto demuestra que wgld(A) ≥ n = wgld(R). Se tiene que
rgld(A) ≥ wgld(A), luego rgld(A) ≥ n = rgld(R).

Teorema 4.6.9. Sean R ⊆ A anillos tales que R es sumando directo de A como


R − R-bimódulo. Entonces, rgld(R) ≤ rgld(A) + pd(AR ). En particular, si AR es
proyectivo, entonces rgld(R) ≤ rgld(A).

Demostración. Es claro que A y R son R − R-bimódulos; existe I ⊆ A, R − R-


bimódulo, tal que A = R ⊕ I. Sea MR un módulo, entonces M ⊗R A ∼
= (M ⊗R R) ⊕

(M ⊗R I) = M ⊕ (M ⊗R I), de donde pd(MR ) ≤ pd(M ⊗R A)R ≤ pd(M ⊗R A)A +
pd(AR ) ≤ rgld(A) + pd(AR ).
Habitualmente el siguiente resultado se prueba usando herramientas categóricas.
Veamos una prueba a partir del teorema anterior y con herramientas elementales.
132 CAPÍTULO 4. DIMENSIONES DE MÓDULOS Y ANILLOS

Corolario 4.6.10. Sea A un anillo, entonces para cada n ≥ 1,

rgld(Mn (A)) = rgld(A).

Demostración. Al considerar el homomorfimso inyectivo de anillos A → Mn (A),


a 7→ diag(a, . . . , a), podemos concluir que A ⊆ Mn (A); además A es sumando
directo de Mn (A) ∼
2
= An como A−A-bimódulo. Según el teorema anterior, rgld(A) ≤
rgld(Mn (A)). Si rgld(A) = ∞, se tiene la igualdad.
Supongamos entonces que rgld(A) := m < ∞ y probemos que rgld(Mn (A)) ≤
rgld(A). Esto lo haremos en dos pasos.
Paso 1. Sea B := Mn (A) y sea M un B-módulo derecho arbitrario; asumamos
que existe un A-módulo derecho NA y un A − B-bimódulo L de tal forma que A L
es plano, LB es proyectivo y NA ⊗A L ∼ = MB (B-isomorfismo). Entonces, puesto que
pd(N ) ≤ m, N tiene una A-resolución proyectiva en la forma
0 → Pm → Pm−1 → · · · → P1 → P0 → N → 0;
tensorizamos la resolución anterior por L y obtenemos una B-resolución proyectiva
de M
0 → Pm ⊗A L → Pm−1 ⊗A L → · · · → P1 ⊗A L → P0 ⊗A L → M → 0;
(la sucesión es exacta ya que L es A-plano a izquierda, además cada Pi ⊗A L es
B-proyectivo). Resulta de esto que pd(M ) ≤ m, por lo tanto, rgld(B) ≤ m.
Paso 2. Probemos lo que hemos asumido en el paso 1. Sea An×1 el conjunto de
vectores columna sobre A de longitud n, se tiene que An×1 es un B −A-bimódulo. En
calidad de NA tomamos entonces NA := M ⊗B An×1 . Ahora consideremos el conjunto
de vectores fila sobre A de longitud n, A1×n , entonces L := A1×n es un A − B-
bimódulo. A L es plano ya que es libre; LB es proyectivo ya que BB ∼
= L ⊕ L0 , donde
L es el conjunto de matrices de B con primera fila nula. Veamos que NA ⊗A L ∼
0
= MB :
NA ⊗A L = (M ⊗B An×1 ) ⊗A (A1×n ) ∼ = M ⊗B (An×1 ⊗A (A1×n )) ∼ = M ⊗B B ∼ = MB ,
ya que An×1 ⊗A (A1×n ) ∼
=B BB . Veamos la demostración de este último isomorfismo:
la función
α : An×1 × (A1×n ) → B, ((a1 , . . . , an )T , (b1 , . . . , bn )) 7→ X := [ai bj ], 1 ≤ i, j ≤ n,
es bilineal y A-balanceada; se induce entonces un homomorfismo de grupos abelianos
e : α : An×1 ⊗A A1×n → B, (a1 , . . . , an )T ⊗A (b1 , . . . , bn ) 7→ X.
α
Recı́procamente, se tiene el homomorfismo
βe : B → An×1 ⊗A A1×n , Y := [yij ] → Y (1) ⊗ e1 + · · · + Y (n) ⊗ en ,
con Y (j) la j-ésima columna de Y y ej el j-ésimo vector cannónico de A1×n . Notemos
que α
eβe = iB , βeα e = iAn×1 ⊗A A1×n , y además, βe es un B − B-homomorfismo.
4.6. DIMENSIÓN GLOBAL Y EXTENSIONES DE ANILLOS 133

Para los anillos de fracciones ninguno de los dos teoremas anteriores aplica, pero
se tiene el siguiente resultado.

Teorema 4.6.11. Sean A un anillo y S un subconjunto multiplicativo de A tal que


AS −1 existe. Entonces, rgld(AS −1 ) ≤ rgld(A).

Demostración. Sea M un AS −1 -módulo, entonces pd(M ⊗A AS −1 ) ≤ pd(MA ): en


primer lugar recordemos que a través del homomorfismo canónico A → AS −1 , M es
un A-módulo; consideremos una resolución proyectiva de MA , entonces tensorizan-
do por AS −1 obtenemos una resolución proyectiva de M ⊗A AS −1 , esto prueba lo
afirmado.
Como M ∼ = M ⊗A AS −1 (proposición 1.6.3), entonces se tiene que pd(MAS −1 ) =
pd(M ⊗A AS −1 ) ≤ pd(MA ), luego rgld(AS −1 ) ≤ rgld(A).

Pasamos ahora a considerar el caso de un producto finito de anillos. Comenzamos


con el siguiente resultado preliminar.

Proposición 4.6.12. Sea A un anillo y sean I1 , . . . , In ideales biláteros propios de


A tales que I1 I2 · · · In = 0. Entonces,

rgld(A) ≤ máx{rgld(A/Ii ) + pd((A/Ii )A )}ni=1 .

Demostración. Dado un módulo MA consideremos los submódulos Mi := M I1 · · · Ii


y los cocientes Mi−1 /Mi , 1 ≤ i ≤ n, con M0 := M . Notemos que Mi−1 /Mi es un
A/Ii -módulo y, por la proposición 4.6.2, se tiene que
pd((Mi−1 /Mi )A ) ≤ pd((Mi−1 /Mi )A/Ii ) + pd((A/Ii )A ) ≤ rgld(A/Ii ) + pd((A/Ii )A ).
Entonces,
máx{pd((Mi−1 /Mi )A )}ni=1 ≤ máx{rgld(A/Ii ) + pd((A/Ii )A )}ni=1 := k.
Consideremos la siguiente sucesión exacta de A-módulos:
0 → Mn−1 → Mn−2 → Mn−2 /Mn−1 → 0;
según la proposición 4.1.6 (vi), pd(Mn−2 ) ≤ máx{pd(Mn−1 ), pd(Mn−2 /Mn−1 )} ≤ k
(nótese que Mn−1 = Mn−1 /Mn ). De manera similar, se tiene la suceción exacta
0 → Mn−2 → Mn−3 → Mn−3 /Mn−2 → 0,
y entonces pd(Mn−3 ) ≤ máx{pd(Mn−2 ), pd(Mn−3 /Mn−2 )} ≤ k. Continuando de esta
manera llegamos a
0 → M1 → M0 → M0 /M1 → 0,
con lo cual pd(M ) = pd(M0 ) ≤ máx{pd(M1 ), pd(M0 /M1 )} ≤ k. Como MA es un
módulo cualquiera, entonces rgld(A) ≤ k.
134 CAPÍTULO 4. DIMENSIONES DE MÓDULOS Y ANILLOS

Teorema 4.6.13. Sea A := A1 × · · · × An un producto finito de anillos. Entonces,


rgld(A) ≤ máx{rgld(Ai )}ni=1 .
Demostración. Basta demostrar la afirmación para n = 2. Notemos que I1 :=
(A1 , 0) := {(a1 , 0)|a1 ∈ A1 } es un ideal bilátero propio de A; lo mismo se tiene
para I2 . Además, I1 I2 = 0; I1 , I2 son ideales derechos de A luego son A-submódulos
de A y se tiene la descomposición A = I1 ⊕I2 , por tanto, I1 , I2 son A-proyectivos y se
tienen los A-isomorfismos A/I1 ∼ = I2 , A/I2 ∼
= I1 . Además se tienen los isomorfismos
de anillo A/I1 ∼
= 2 A , A/I ∼
2 = A 1 . La afirmación del teorema es entonces consecuencia
de la proposición anterior.

4.7. Dimensión de Krull de un módulo


Mostramos a continuación los principales resultados de la teorı́a de dimensión de
Krull de módulos y anillos. El desarrollo lo haremos para el caso de módulos a
derecha, pero desde luego que todos los resultados son también válidos a izquierda.
Definición 4.7.1. Sea A un anillo. Para cada ordinal α ≥ −1 se definen las si-
guientes clases Kα de A-módulos:
(i) K−1 := {0}.
(ii) Se asume que la clase Kβ está definida para cada β < α y se define Kα de
la siguiente manera: M ∈ Kα si, y sólo si, para
S cada cadena de submódulos
M0 ⊇ M1 ⊇ · · · de M se tiene que Mi /Mi+1 ∈ β<α Kβ , para casi todo i.
(iii) Si M ∼
= M 0 , entonces M ∈ Kα si, y sólo si, M 0 ∈ Kα .
Se dice que la dimensión de Krull de M existe, o que está definida, si existe α tal
que M ∈ Kα , y en tal caso se escribe Kdim(M ) ≤ α. El menor α tal que M ∈ Kα
se denomina la dimensión de Krull de M y se escribe Kdim(M ) = α. Si para
cada α, M ∈ / Kα , entonces se dice que M no tiene dimensión de Krull.
Proposición 4.7.2. Sean α ≥ 0 y M un módulo. Entonces, Kdim(M ) ≤ α si, y
sólo si, para cada cadena descendente M0 ⊇ M1 ⊇ · · · de submódulos de M se tiene
que Kdim(Mi /Mi+1 ) < α, para casi todo i.
Demostración. ⇒): sea Kdim(M ) ≤ α, entonces para casi todo i, Mi /Mi+1 ∈
S
β<α Kβ , y para esos valores de i se tiene que Kdim(Mi /Mi+1 ) está definida y
Kdim(Mi /Mi+1 ) < α.
⇐): la condición impuesta indica que para casi todo i los cocientes Mi /Mi+1
tienen dimensión de Krull, digamos,
S Kdim(Mi /Mi+1 ) := αi < α, luego Mi /Mi+1 ∈
Kαi , de donde Mi /Mi+1 ∈ β<α Kβ , es decir, M ∈ Kα , o en forma equivalente,
Kdim(M ) ≤ α.
4.7. DIMENSIÓN DE KRULL DE UN MÓDULO 135

La siguiente proposición establece que la dimensión de Krull de un módulo M


mide que tan cerca está M de ser artiniano.

Proposición 4.7.3. Kdim(M ) = 0 si, y sólo si, M 6= 0 y M es artiniano.

Demostración. ⇒): M 6= 0 ya que de lo contrario Kdim(M ) = −1; sea M0 ⊇


M1 ⊇ · · · una cadena descendente de submódulos de M , entonces por la proposición
anterior, para casi todo i, Kdim(Mi /Mi+1 ) < 0, es decir, Mi /Mi+1 = 0, luego la
cadena se estabiliza.
⇐): como M 6= 0, M ∈ / K−1 ; puesto que cada cadena descendente de submódulos
de M se estabiliza, entonces M ∈ K0 , es decir, Kdim(M ) = 0.

Ejemplo 4.7.4. (i) Kdim(ZZ ) = 1: ya que ZZ no es artiniano, entonces Kdim(ZZ ) ≥


1. Sea hm0 i ⊇ hm1 i ⊇ · · · una cadena de ideales de Z; para cada i, hmi i/hmi+1 i es
artiniano, luego hmi i/hmi+1 i ∈ K0 , y entonces Kdim(ZZ ) = 1.
(ii) Puesto que Zp∞ es artiniano Kdim(Zp∞ ) = 0, notemos que Zp∞ no es un
Z-módulo noetheriano.

Teorema 4.7.5. Sean N ⊆ M módulos. Entonces, Kdim(M ) existe si, y sólo si,
Kdim(N ) y Kdim(M/N ) existen. En tal caso,

Kdim(M ) = máx{Kdim(N ), Kdim(M/N )}.

Demostración. ⇒): sea Kdim(M ) = α, veamos que Kdim(N ), Kdim(M/N ) ≤ α.


Para esto probemos que N, M/N ∈ Kα . Sea N0 ⊇ N1 ⊇ · · · una cadena de submódu-
los de N , luego es también
S una cadena descendente de submódulos de M y por
lo tanto Ni /Ni+1 ∈ β<α Kβ , para casi todo i. Esto indica que N ∈ Kα , es de-
cir, Kdim(N ) ≤ α. Sea ahora M0 /N ⊇ M1 /N ⊇ · · · una cadena descendente de
submódulos de M/N ,Sresulta en M la cadena M0 ⊇ M1 ⊇ · · · y para casi todo i se
tiene que Mi /Mi+1 ∈ Sβ<α Kβ , pero como (Mi /N )/(Mi+1 /N ) ∼
= Mi /Mi+1 , entonces
(Mi /N )/(Mi+1 /N ) ∈ β<α Kβ para casi todo i, de donde Kdim(M/N ) ≤ α.
Lo probado adicionalmente significa que
Kdim(M ) ≥ máx{Kdim(N ), Kdim(M/N )}.
⇐): supongamos ahora que Kdim(N ) y Kdim(M/N ) existen, y sea
α := máx{Kdim(N ), Kdim(M/N )}.
Entonces Kdim(N ) ≤ α y Kdim(M/N ) ≤ α; probaremos por inducción sobre α
que Kdim(M ) ≤ α. Si α = −1, entonces N = 0, M/N = 0, de donde M = 0 y
Kdim(M ) = −1 ≤ α. Sea α ≥ 0 y sea M0 ⊇ M1 ⊇ · · · una cadena descendente de
submódulos de M , entonces resulta en M/N la cadena (M0 +N )/N ⊇ (M1 +N )/N ⊇
· · · , luego para casi todo i,
Kdim[(Mi + N/N )/(Mi+1 + N/N )] = Kdim[(Mi + N )/(Mi+1 + N )] < α.
136 CAPÍTULO 4. DIMENSIONES DE MÓDULOS Y ANILLOS

De igual manera resulta en N la cadena de submódulos M0 ∩ N ⊇ M1 ∩ N ⊇ · · · , y


para casi todo i,
Kdim(Mi ∩ N/Mi+1 ∩ N ) < α.
Notemos que para cada i se tiene el A-homomorfismo sobreyectivo
fi
Mi /Mi+1 −
→ (Mi + N )/(Mi+1 + N )
mi 7→ m
fi
con ker(fi ) ∼
= (Mi ∩ N )/(Mi+1 ∩ N ). Veamos la prueba de esta última afirmación: si
mi ∈ Mi ∩ N , entonces fi (mi ) = m
fi = e0 y definimos
gi
Mi ∩ N −
→ ker(fi ), mi 7→ mi .
gi es claramente un A-homomofismo, además es sobreyectivo porque si mi ∈ ker(fi ),
entonces m 0, de donde mi = mi+1 + n, con mi ∈ Mi , mi+1 ∈ Mi+1 , n ∈ N , luego
fi = e
n = mi − mi+1 ∈ Mi ∩ N y de esta manera gi (n) = n = mi . Ahora observemos
que ker(gi ) = Mi+1 ∩ N pues si mi ∈ ker(gi ), entonces mi ∈ N y mi = 0, de donde
mi ∈ Mi+1 , es decir, mi ∈ Mi+1 ∩ N .
Con lo probado se tiene que Kdim(ker(fi )) < α para casi todo i y también
que Kdim[(Mi /Mi+1 )/(ker(fi ))] = Kdim[(Mi + N )/(Mi+1 + N )] < α. Aplicamos
inducción y encontramos que para casi todo i
Kdim(Mi /Mi+1 ) ≤ máx{Kdim(ker(fi )), Kdim[(Mi + N )/(Mi+1 + N )]} < α,
pero esto significa que Kdim(M ) ≤ α. Esto completa la prueba del teorema.
Corolario 4.7.6. Sean M1 , . . . , Mk módulos. Entonces, Kdim(M1 ⊕· · · ⊕Mk ) existe
si, y sólo si, cada Kdim(Mi ) existe, 1 ≤ i ≤ k. En tal caso,
Kdim(M1 ⊕ · · · ⊕ Mk ) = máx{Kdim(Mi )}ki=1 .
Demostración. Basta probar el corolario para k = 2. Si Kdim(M1 ⊕ M2 ) existe,
entonces cada Kdim(Mi ) existe ya que Mi es un submódulo de M1 ⊕ M2 . Recı́pro-
camente, si Kdim(M1 ), Kdim(M2 ) existen, entonces Kdim[(M1 ⊕ M2 )/M1 ] existe y
se puede aplicar el teorema anterior.
Proposición 4.7.7. Sea M 6= 0 un módulo tal que para cada submódulo 0 6= N ⊆
M , Kdim(M/N ) ≤ α. Entonces, Kdim(M ) ≤ α + 1.
Demostración. Si α = −1, entonces para cada 0 6= N ⊆ M , N = M , es decir, M
es simple, luego artiniano, de donde Kdim(M ) = 0 = α + 1. Sea pues α ≥ 0; sea
M0 ⊇ M1 ⊇ · · · una cadena de submódulos de M ; si existe n tal que Mn = 0,
entonces para cada i ≥ n se tiene que Kdim(Mi /Mi+1 ) = −1 < α < α + 1; si cada
Mi es no nulo, entonces Kdim(Mi /Mi+1 ) ≤ Kdim(M/Mi+1 ) ≤ α < α + 1. Resulta,
Kdim(M ) ≤ α + 1.
4.7. DIMENSIÓN DE KRULL DE UN MÓDULO 137

Teorema 4.7.8. Si M es un módulo noetheriano, entonces Kdim(M ) está definida.

Demostración. Si M = 0, entonces Kdim(M ) = −1. Supongamos entonces que


M es no nulo. Mediante inducción noetheriana (véase [7]) se puede demostrar que
para cada submódulo 0 6= N ⊆ M la dimensión de Krull de M/N está definida.
Ası́ pues, sea αN := Kdim(M/N ). Puesto que todo conjunto de ordinales tiene
supremo, sea α := sup{αN |0 6= N ⊆ M }, entonces la proposición 4.7.7 garantiza
que Kdim(M ) ≤ α + 1, es decir, M tiene dimensión de Krull.

Proposición 4.7.9. Sean R y A anillos con módulos MR , NA tales que Kdim(MR ) y


Kdim(NA ) están definidas. Si existe una función g del retı́culo L(M ) de submódulos
de MR en el retı́culo L(N ) de submódulos de NA que preserva la inclusión estricta,
entonces Kdim(MR ) ≤ Kdim(NA ).

Demostración. Dividimos la demostración en dos pasos.


Paso 1. Probemos por inducción que para cada ordinal α,
si L0 L en L(M ) con Kdim(L/L0 ) > α, entonces Kdim(g(L)/g(L0 )) > α.
En efecto, sea α = −1, como g(L)/g(L0 ) 6= 0, entonces Kdim(g(L)/g(L0 )) > −1 = α.
Sea α = 0, entonces Kdim(L/L0 ) > 0 y por lo tanto L/L0 no es artiniano. Existe
una cadena estricta descendente infinita L0 /L0 > L1 /L0 > · · · de submódulos de
L/L0 con lo cual se tiene también en g(L)/g(L0 ) la cadena descendente infinita
g(L0 )/g(L0 ) > g(L1 )/(L0 ) > · · · , es decir, g(L)/g(L0 ) no es artiniano, de donde
Kdim(g(L)/g(L0 )) > 0. Supongamos que para cada β < α se cumple la propiedad
que estamos demostrando. Sea Kdim(L/L0 ) > α; por la proposición 4.7.2 existe
una cadena descendente L0 /L0 ⊇ L1 /L0 ⊇ · · · de submódulos de L/L0 tal que
Kdim(Li /L0 /Li+1 /L0 ) = Kdim(Li /Li+1 ) ≥ α > 0 para casi todo i, luego para casi
todo i, Li+1 Li ; puesto que Kdim(Li /Li+1 ) > β podemos aplicar inducción y resul-
ta Kdim(g(Li )/g(Li+1 )) > β para casi todo i y todo β < α, es decir, en g(L)/g(L0 )
existe una cadena descendente tal que casi todas las secciones tienen dimensión de
Krull > β para cada β < α, luego por la proposición 4.7.2, Kdim(g(L)/g(L0 )) > α.
Paso 2. Sean α := Kdim(M ) y β := Kdim(N ). Si α = −1, entonces claramente
β ≥ α; si α = 0, M 6= 0, pero como g preserva la inclusión estricta, entonces N 6= 0
y β ≥ α. Sea pues α ≥ 1; usando nuevamente el hecho que g preserva la inclusión
estricta se tiene que β ≥ 0. Consideremos en M una cadena arbitraria descendente
de submódulos M0 ⊇ M1 ⊇ M2 ⊇ · · · ; si probamos que para casi todo i se cumple
que Kdim(Mi /Mi+1 ) < β, entonces por la proposición 4.7.2 Kdim(M ) ≤ β, es decir,
α ≤ β y la proposición estarı́a demostrada.
Ası́ pues, sea αi := Kdim(Mi /Mi+1 ); para los valores de i con αi = −1 claramente
αi < β; sea entonces αi ≥ 0, luego Mi+1 Mi , y como Kdim(Mi /Mi+1 ) > αi − 1,
por lo probado en el paso 1 resulta βi := Kdim(g(Mi )/g(Mi+1 )) > αi − 1. En N
tenemos entonces la cadena de submódulos g(M0 ) ⊇ g(M1 ) ⊇ g(M2 ) ⊇ · · · y para
138 CAPÍTULO 4. DIMENSIONES DE MÓDULOS Y ANILLOS

casi todo i se tiene β > βi = Kdim(g(Mi )/g(Mi+1 )), luego β > βi > αi − 1, es decir,
β > βi ≥ αi para casi todo i. Esto concluye la demostración.
Proposición 4.7.10. Sea M 6= 0 un A-módulo tal que Kdim(M ) existe. Sea f ∈
EndA (M ) inyectivo. Entonces, Kdim(M ) ≥ Kdim(M/Im(f )) + 1.
Demostración. Como M 6= 0, Kdim(M ) ≥ 0; sabemos que α := Kdim(M/Im(f ))
existe. Si α = −1 no hay algo que demostrar; sea pues α ≥ 0. Consideremos la cadena
M ⊇ Im(f ) ⊇ Im(f 2 ) ⊇ · · · , se tiene que Im(f i )/Im(f i+1 ) ∼
= M/Im(f ) para cada
i i+1
i, es decir, Kdim(Im(f )/Im(f )) = Kdim(M/Im(f )) = α, luego Kdim(M ) > α,
es decir, Kdim(M ) ≥ α + 1 = Kdim(M/Im(f )) + 1.
Definición 4.7.11. Sean M 6= 0 un módulo y α ≥ 0.
(i) Se dice que M es α-crı́tico si Kdim(M ) = α y para cada submódulo no nulo
N de M se tiene que Kdim(M/N ) < α.

(ii) Se dice que M es crı́tico si es α-crı́tico para algún α ≥ 0.

(iii) Una serie de composición crı́tica de M es una cadena finita de submódulos

0 = M0 M1 · · · Mn = M

tal que cada cociente Mi /Mi−1 es crı́tico y además

Kdim(M1 ) ≤ Kdim(M2 /M1 ) ≤ · · · ≤ Kdim(Mn−1 /Mn−2 ) ≤ Kdim(M/Mn−1 ).

Proposición 4.7.12. Si M es α-crı́tico y 0 6= N ≤ M , entonces N también es


α-crı́tico.
Demostración. Sea 0 6= N 0 ≤ N , entonces N 0 es un submódulo no nulo de M
y Kdim(M/N 0 ) < α, pero N/N 0 ≤ M/N 0 , entonces Kdim(N/N 0 ) < α. Según el
teorema 4.7.5, α = Kdim(M ) = máx{Kdim(N ), Kdim(M/N )}, pero Kdim(M/N ) <
α, entonces Kdim(N ) = α.
Teorema 4.7.13. Sea M un módulo noetheriano no nulo. Entonces M tiene una
serie de composición crı́tica.
Demostración. Dividimos la prueba en tres pasos.
Paso 1. Como M es noetheriano, M y cada uno de sus submódulos tienen
dimensión de Krull. Sea α1 := min{Kdim(N )|0 6= N ⊆ M } ≥ 0. Veamos que
M tiene al menos un submódulo que es α1 -crı́tico: en efecto, elegimos N0 no nu-
lo con Kdim(N0 ) = α1 . Si N0 no es crı́tico, entonces existe un submódulo no
nulo N1 en N0 tal que Kdim(N0 /N1 ) = α1 ; por el teorema 4.7.5 sabemos que
α1 ≥ Kdim(N1 ), pero por la minimalidad de α1 se debe tener que α1 = Kdim(N1 ).
4.8. DIMENSIÓN DE KRULL DE UN ANILLO 139

Continuando de esta forma resulta la cadena N0 ≥ N1 ≥ N2 ≥ · · · de tal for-


ma que Kdim(Ni ) = Kdim(Ni−1 /Ni ) = α1 . Si para cada i, Ni no es crı́tico, en-
tonces tendrı́amos una cadena descendente con Kdim(Ni−1 /Ni ) = α1 , es decir,
Kdim(Ni−1 /Ni ) ≥ α1 , pero según la propsición 4.7.2 se tendrı́a que Kdim(N0 ) > α1 ,
lo cual es falso. Ası́ pues, algún Ni es α1 -crı́tico.
Paso 2. Sea M1 el submódulo α1 -crı́tico encontrado en el paso anterior. Probemos
que M/M1 no contiene submódulos no nulos con dimensión de Krull menor que α1 :
supongamos que M 0 /M1 es un submódulo no nulo de M/M1 tal que Kdim(M 0 /M1 ) <
α1 , por el teorema 4.7.5 es claro que Kdim(M 0 ) = α1 (el máximo entre Kdim(M 0 /M1 )
y Kdim(M1 )); por la escogencia de α1 , cada submódulo no nulo de M 0 debe tener
dimensión de Krull mayor o igual que α1 ,
Paso 3. Si M1 = M hemos terminado; sea M1 6= M ; M/M1 es noetheriano no
nulo y podemos aplicar nuevamente los pasos 1 y 2 a este cociente encontrando
un ordinal α2 y un submódulo M2 /M1 α2 -crı́tico tal que M/M1 /M2 /M1 ∼ = M/M2
no tiene submódulos no nulos con dimensión de Krull menor que α2 . Notemos que
M1 M2 ya que M2 /M1 es no nulo por ser crı́tico; además α2 ≥ α1 ya que de lo
contrario M/M1 contendrı́a como submódulo no nulo a M2 /M1 con dimensión de
Krull α2 < α1 . Si M2 6= M podemos continuar en la misma forma y, puesto que M
es noetheriano, el proceso termina en una cadena finita de la forma 0 = M0 M1
M2 · · · Mn = M ; por construcción, el cociente Mi /Mi−1 es αi -crı́tico y además
α1 ≤ α2 ≤ · · · ≤ αn , es decir, tenemos una serie de composición crı́tica para M .

4.8. Dimensión de Krull de un anillo


Definición 4.8.1. Sea A un anillo. rKdim(A) := Kdim(AA ), si esta última existe.

Ejemplo 4.8.2. En forma análoga se define la dimensión de Krull de A a izquierda.


Parece ser un problema todavı́a abierto si la dimensión de Krull a derecha de un
anillo A coincide siempre con su dimensión de Krull a izquierda, en caso que ambas
existan (véase [7]). Mostramos en este ejemplo un anillo con dimensión de Krull
a derecha pero que no tiene dimensión de Krull a izquierda: sea A el subanillo de
M2 (R) definido por
 
Q R
A := ,
0 R
notemos que AA es artiniano (y por lo tanto noetheriano), pero A A no es arti-
niano (tampoco noetheriano), véase [18]. Luego rKdim(A)
P∞ = 0 y lKdim(A) 6= 0. En
realidad notemos que lKdim(A) no existe: A A ⊇ i=1 Asi , con
 
0 xi
si := , {xi }i≥1 ⊂ R linealmente independiente sobre Q;
0 0
140 CAPÍTULO 4. DIMENSIONES DE MÓDULOS Y ANILLOS

notemos que para cada i, Asi ∼


= As1 . Resulta entonces en A A la cadena descendente
M1 ) M2 ) M3 ) · · · , con

M1 :=As1 ⊕ As2 ⊕ As3 ⊕ As4 ⊕ · · ·


M2 :=As2 ⊕ As4 ⊕ As6 ⊕ As8 ⊕ · · ·
M3 :=As4 ⊕ As8 ⊕ As12 ⊕ As16 ⊕ · · ·
..
.

y que satisface Mi /Mi+1 ∼ = M1 para cada i ≥ 1. Esto implica que la dimensión de


Krull de A A no está definida. En efecto, sea M un módulo que contiene una cadena
de submódulos M1 M2 · · · tal que Mi /Mi+1 ∼ = M1 para cada i. Entonces M no
tiene dimensión de Krull: supongamos que lKdim(M ) ≤ α; sea lKdim(M1 ) := β ≤ α;
M1 6= 0 ya que se tiene la cadena de submódulos propios anunciada, luego β ≥ 0 y
por la proposición 4.7.2, lKdim(Mi /Mi+1 ) < β para cada casi todo i ≥ 2, lo cual es
una contradicción ya que Mi /Mi+1 ∼ = M1 para cada i.
Ejemplo 4.8.3. (i) Es claro que si R es un anillo conmutativo, la dimensión a
derecha coincide con la dimensión a izquierda, en caso de que exista, y se denota
simplemente por Kdim(R). Ası́ pues, por la proposición 4.7.3, si K es un cuerpo,
entonces Kdim(K) = 0.
(ii) Del ejemplo 4.7.4 se tiene que Kdim(Z) = 1 = gld(Z).
(iii) Puesto que Z4 es artiniano, entonces Kdim(Z4 ) = 0, sin embargo gld(Z4 ) =
∞, es decir, la dimensión de Krull y la dimensión global no siempre coinciden.
Proposición 4.8.4. Sean A un anillo e I un ideal bilátero propio de A. Si rKdim(A)
existe, entonces, rKdim(A/I) existe y rKdim(A/I) ≤ rKdim(A).
Demostración. Según el teorema 4.7.5, Kdim[(A/I)A ] existe. Puesto que los A/I-
submódulos de A/I coinciden con sus A-submódulos, rKdim(A/I) existe y se tiene
que rKdim(A/I) = Kdim((A/I)A/I ) = Kdim[(A/I)A ] ≤ Kdim(AA ) = rKdim(A).

Proposición 4.8.5. Sean A un anillo noetheriano a derecha y M un A-módulo f.g.


Entonces, Kdim(M ) ≤ rKdim(A).
Demostración. Por el teorema 4.7.8, Kdim(M ) y rKdim(A) están definidas. Exis-
te t ≥ 1 tal que M ∼ = At /N , con N un submódulo de At , y por el corolario
4.7.6, Kdim(At ) = Kdim(AA ) = rKdim(A). Por el teorema 4.7.5, Kdim(M ) =
Kdim(At /N ) ≤ Kdim(At ) = rKdim(A).
Proposición 4.8.6. Sea ι : R → A un homomorfismo de anillos noetherianos a
derecha tal que R A es fielmente plano. Si MR es un módulo f.g., entonces
Kdim(M ) ≤ Kdim(M ⊗R A). En particular, rKdim(R) ≤ rKdim(A).
4.8. DIMENSIÓN DE KRULL DE UN ANILLO 141

Demostración. En primer lugar observemos que MR y (M ⊗R A)A son módulos


noetherianos, luego las dimensiones Kdim(M ) y Kdim(M ⊗R A) existen. Conside-
remos los retı́culos de submódulos de M y M ⊗R A; sean M1 M2 submódulos
de M , entonces se tiene la inclusión canónica M1 ,→ M2 , y como R A es plano,
entonces la aplicación M1 ⊗ A ,→ M2 ⊗ A es inyectiva. Pero puede ocurrir que
M1 ⊗ A = 0 = M2 ⊗ A y la aplicación anterior continue siendo inyectiva, pero
como R A es fielmente plano tal situación no se presenta. Ası́ pues podemos aplicar
la proposición 4.7.9 y obtenemos el resultado.
Ejemplo 4.8.7. Sea R un anillo noetheriano a derecha y sea R[x] su anillo de
polinomios. Entonces rKdimR ≤ rKdim(R[x]). En realidad se tiene el siguiente
resultado más completo: rKdim(R[x]) = rKdimR + 1 (véase [?])
Teorema 4.8.8. Sean A un anillo noetheriano a derecha y S un subconjunto mul-
tiplicativo de A tal que AS −1 existe. Entonces,
rKdim(AS −1 ) ≤ rKdim(A).
Demostración. Según la proposición 1.5.14 las dos dimensiones involucradas existen
y existe una función que preserva la inclusión estricta entre los ideales derechos
de AS −1 y los ideales derechos de A. El teorema es entonces consecuencia de la
proposición 4.7.9.
Sea rad(A) el nilradical de A, conocido también como el radical primo de
A, y definido como la intersección de todos los ideales primos de A (véase [18]). La
dimensión de Krull de A y la de su anillo cociente A/rad(A) coniciden en el caso
noetheriano. Para demostrarlo debemos probar un par de resultados clásicos.
Proposición 4.8.9. Sea A un anillo y sea P un ideal primo. Entonces P contiene
un ideal primo minimal.
Demostración. Recordemos que un ideal bilátero propio Q de A es primo si dados
dos ideales biláteros I, I 0 de A con II 0 ⊆ Q se cumple que I ⊆ Q o I 0 ⊆ Q.
Veamos que esta definición es equivalente a la siguiente: si a, b ∈ A son tales que
aAb ⊆ Q, entonces a ∈ Q o b ∈ Q. En efecto, supongamos la primera definición y
sea aAb ⊆ Q, entonces haihbi ⊆ Q ya que cada elemento de este producto de ideales
biláteros es una suma finita de productos en la forma xayx0 by 0 = x(ayx0 b)y 0 ∈ Q.
En consecuencia, hai ⊆ Q o hbi ⊆ Q, es decir, a ∈ Q o b ∈ Q. Asumamos ahora la
segunda definición y sean I, I 0 biláteros de A tales que II 0 ⊆ Q; si I * Q, existe
x ∈ I, x ∈ / Q; sea y ∈ I 0 , entonces xy = x1y ∈ xAy ⊆ Q, por la condición de la
segunda definición, x ∈ Q o y ∈ Q, es decir, I 0 ⊆ Q.
Sea M la colección de primos contenidos en P , como P ∈ M se tiene que
M= 6 ∅; podemos ordenar M por inclusión y usar el lema de Zorn en su versión de
acotamiento inferior (versión desde luego equivalente a la versión usual). Sea C un
142 CAPÍTULO 4. DIMENSIONES DE MÓDULOS Y ANILLOS

subconjunto no vacı́o totalmente ordenado de M; sea Q := P 0 ∈C P 0 , es claro que


T
Q es un ideal propio de A contenido en P . Veamos que Q es primo: sean x, y ∈ A
tales que xAy ⊆ Q pero x ∈ / Q, existe P 0 ∈ C tal que x ∈ / P 0 . Para cualquier P 00 ∈ C
con P 00 ⊆ P 0 se tiene que x ∈/ P 00 pero xAy ⊆ Q ⊆ P 00 , por lo tanto y ∈ P 00 ; en
particular, y ∈ P . Ahora bien, si P 00 ∈ C pero P 00 * P 0 , entonces P 0 ⊆ P 00 , luego de
0

nuevo y ∈ P 00 . Esto demuestra que y ∈ Q.


Ası́ pues, Q es una cota inferior de C en M, luego por el lema de Zorn existe un
elemento minimal P ∗ en M, es decir, P ∗ es un ideal primo de A contenido en P y
es tal que cada primo contenido en P ∗ pertenence a M, luego por la minimalidad
se conluye que P ∗ es un ideal primo minimal.
Proposición 4.8.10. Sea A un anillo noetheriano a derecha. Entonces
(i) Existe una colección finita P1 , . . . , Pn de ideales primos minimales (no nece-
sariamente distintos) tales que P1 · · · Pn = 0.

(ii) La colección de primos minimales de A es finita.


Demostración. (i) Probemos primero que existe una colección finita de ideales pri-
mos P10 , . . . , Pn0 tales que P10 · · · Pn0 = 0. Supongamos contrariamente que cada pro-
ducto finito de ideales primos de A es no nulo (en particular, el ideal nulo no es
primo). Sea C la colección de ideales biláteros I de A que no contienen productos
finitos de ideales primos (en particular, I no es primo). C 6= ∅ ya que 0 ∈ C. Como
A es noetheriano a derecha (en realidad basta suponer que la colección de biláteros
de A es noetheriana) en C hay elemento maximal I. Ası́ pues, en A := A/I cada
producto finito de ideales primos es no nulo, y por la maximalidad de I, en A cada
ideal no nulo contiene al menos un producto finito de ideales primos. En particular,
0 no es primo, por lo tanto, existen ideales no nulos K := K/I y J := J/I tales
que K J = 0 y además existen ideales primos P 1 , . . . , P m , Q1 , . . . , Qn en A tales que
P 1 · · · P m ⊆ K y Q1 · · · Qn ⊆ J. Resulta, P 1 · · · P m Q1 · · · Qn = 0, lo cual es una
contradicción. Ası́ pues, existen P10 , . . . , Pn0 primos de A tales que P10 · · · Pn0 = 0.
Según la proposición anterior, sea Pi ⊆ Pi0 minimal. Entonces, P1 . . . Pn ⊆
P1 . . . Pn0 = 0, luego P1 . . . Pn = 0.
0

(ii) Sea P un primo minimal de A, entonces 0 = P1 · · · Pn ⊆ P , de donde Pi = P


para algún 1 ≤ i ≤ n.
Teorema 4.8.11. Sea A un anillo noetheriano a derecha. Entonces,

rKdim(A) = rKdim(A/rad(A)) = máx{rKdim(A/P )|P es primo minimal de A}.

Demostración. Como A es noetheriano a derecha todas las dimensiones involucradas


en el enunciado están definidas. Sea N := rad(A), entonces

α := rKdim(A) ≥ rKdim(A/N ) := β.
4.8. DIMENSIÓN DE KRULL DE UN ANILLO 143

Veamos ahora que β ≥ γ, con γ := máx{rKdim(A/P )|P es primo minimal de A}.


Sea P un ideal primo cualquiera de A, entonces rKdim(A/N ) ≥ rKdim(A/P ): en
efecto, puesto que A/P ∼
= (A/N )/(P/N ), entonces

rKdim(A/P ) = Kdim[(A/P )A/P ] = Kdim[((A/N )/(P/N ))(A/N )/(P/N ) ]


= Kdim[((A/N )/(P/N ))A/N ] ≤ Kdim[(A/N )A/N ] = rKdim(A/N ).

Resta ver que γ ≥ α: como A es noetheriano a derecha existe una colección fini-
ta de ideales primos minimales, no necesariamente distintos, P1 , . . . , Pn tales que
P1 · · · Pn = 0 (en realidad la colección de primos minimales de A es finita, véase la
proposición 4.8.10). Por la proposición 4.8.5, para cada 1 ≤ i ≤ n se tiene que

Kdim[(P1 · · · Pi−1 /P1 · · · Pi )A ] = Kdim[(P1 · · · Pi−1 /P1 · · · Pi )A/Pi ] ≤ rKdim(A/Pi ).

Veamos que con esto podemos completar la demostración de γ ≥ α; con tres casos
particulares que examinemos podremos más adelante inducir una prueba general y
completar la demostración: si n = 1, entonces P1 = 0 y entonces Kdim(A/0)A =
rKdim(A/P1 ), es decir, rKdim(A) = rKdim(A/P1 ); si n = 2, entonces P1 P2 = 0, por
lo probado arriba resulta Kdim[(P1 /P1 P2 )A ] ≤ rKdim(A/P2 ), es decir, Kdim(P1 )A ≤
rKdim(A/P2 ), además Kdim[(A/P1 )A ] = rKdim(A/P1 ), luego por el teorema 4.7.5,

rKdim(A) = Kdim(AA ) = máx{Kdim(P1 )A , Kdim(A/P1 )A }


≤ máx{rKdim(A/P2 ), rKdim(A/P1 )}.

Para n = 3 tenemos P1 P2 P3 = 0 y entonces

Kdim[((P1 P2 )/(P1 P2 P3 ))A ] ≤ rKdim(A/P3 ) ⇒ Kdim[(P1 P2 )A ] ≤ rKdim(A/P3 );


Kdim[(P1 /P1 P2 )A ] ≤ rKdim(A/P2 );
Kdim[(A/P1 )A ] = rKdim(A/P1 ),

de donde

rKdim(A) = Kdim(AA ) = máx{Kdim(P1 P2 )A , Kdim(A/P1 P2 )A }


= máx{Kdim(P1 P2 )A ,
máx{Kdim[(P1 /P1 P2 )A ], Kdim[((A/P1 P2 )/P1 /P1 P2 )A ]}}
≤ máx{rKdim(A/P3 ), rKdim(A/P2 ), rKdim(A/P1 )}.

Ası́ pues, si P1 · · · Pn = 0 podemos aplicar inducción:

rKdim(A) = Kdim(AA ) = máx{Kdim(P1 · · · Pn−1 )A , Kdim(A/P1 · · · Pn−1 )A }


≤ máx{rKdim(A/Pn ), rKdim(A/Pn−1 ), . . . , rKdim(A/P1 )}.
144 CAPÍTULO 4. DIMENSIONES DE MÓDULOS Y ANILLOS

Corolario 4.8.12. Sea A un anillo noetheriano a derecha. Entonces, existe un pri-


mo minimal P en A tal que rKdim(A) = rKdim(A/P ).

Demostración. Consecuencia directa del teorema 4.8.11 y de la proposición 4.8.10.

Veremos enseguida que la dimensión de Krull de un producto finito de anillos


noetherianos a derecha es menor que el máximo de las dimensiones de los factores.

Proposición 4.8.13. Sea A un anillo noetheriano a derecha y sean I1 , . . . , In ideales


biláteros propios de A tales que I1 I2 · · · In = 0. Entonces,

rKdim(A) ≤ máx{rKdim(A/Ii )}ni=1 .

Demostración. Consideremos los productos Pi := I1 · · · Ii y los cocientes Pi−1 /Pi ,


1 ≤ i ≤ n, con P0 := A. Notemos que Pi−1 /Pi es un A/Ii -módulo, por la proposición
4.8.5 se tiene que
Kdim((Pi−1 /Pi )A ) = Kdim((Pi−1 /Pi )A/Ii ) ≤ rKdim(A/Ii ).
Entonces,
máx{Kdim((Pi−1 /Pi )A )}ni=1 ≤ máx{rKdim((A/Ii )}ni=1 := k.
Consideremos la siguiente sucesión exacta de A-módulos:
0 → Pn−1 → Pn−2 → Pn−2 /Pn−1 → 0;
según el teorema 4.7.5, Kdim(Pn−2 ) = máx{Kdim(Pn−1 ), Kdim(Pn−2 /Pn−1 )} ≤ k
(nótese que Pn−1 = Pn−1 /Pn ). De manera similar, se tiene la suceción exacta
0 → Pn−2 → Pn−3 → Pn−3 /Pn−2 → 0,
y entonces Kdim(Pn−3 ) = máx{Kdim(Pn−2 ), Kdim(Pn−3 /Pn−2 )} ≤ k. Continuando
de esta manera llegamos a
0 → P1 → P0 → P0 /P1 → 0,
con lo cual rKdim(A) = Kdim(P0 ) = máx{Kdim(P1 ), Kdim(P0 /P1 )} ≤ k.

Teorema 4.8.14. Sea A := A1 × · · · × An un producto finito de anillos noetherianos


a derecha. Entonces, rKdim(A) ≤ máx{Kdim(Ai )}ni=1 .

Demostración. Basta demostrar la afirmación para n = 2. Notemos que I1 :=


(A1 , 0) := {(a1 , 0)|a1 ∈ A1 } es un ideal bilátero propio de A; lo mismo se tiene
para I2 . Además, se tienen los isomorfismos de anillo A/I1 ∼ = A2 , A/I2 ∼= A1 . La
afirmación del corolario es entonces consecuencia de la proposición anterior.
4.9. EJERCICIOS 145

4.9. Ejercicios
1. Demuestre la proposición 4.1.7.

2. Sean A un anillo, n ≥ 0 y Extn+1A (M, N ) = 0, con M, N módulos derechos


A
arbitrarios. Demuestre que T orn+1 (P, Q) = 0 para cada A-módulo derecho P
y cada A-módulo izquierdo Q.

3. Demuestre que si R es un dominio de integridad von Neumann regular, en-


tonces R es un cuerpo (sugerencia: demuestre que un anillo A es von Neumann
regular si, y sólo si, dado a ∈ A existe a0 ∈ A tal que aa0 a = a).

4. Sea R un dominio de integridad. Demuestre que R es un cuerpo si, y sólo si,


wgld(R) = 0.

5. Sean R un dominio de Dedekind y M un R-módulo. Demuestre que M es


proyectivo si, y sólo si, Ext1A (M, F ) = 0 para cada módulo libre F .

6. Existen anillos R para los cuales rgld(R) 6= lgld(R). Presente un ejemplo


(consulte [9] o también [10]).

7. Muestre un ejemplo en el cual no se cumple el teorema 4.6.8 si rgld(R) = ∞.


Bibliografı́a

[1] Bell, A., Notes on Localizations in Noncommutative Noetherian Rings, Depar-


tamento de Álgebra y Fundamentos, Universidad de Granada, España, 1989.

[2] Cartan, H. and Eilenberg, S., Homological Algebra, Princeton University


Press, 1956.

[3] Dixmier, J., Enveloping Algebras, GSM 11, AMS, 1996.

[4] Faith, C., Algebra I: Rings, Modules and Categories, Springer, 1981.

[5] M. Fontana, J.A. Huckaba and I.J. Papick, Prüfer Domains, Marcel
Dekker, New York, 1997.

[6] Gallego, C. and Lezama, O., Gröbner bases for ideals of σ−P BW extensions,
Comminications in Algebra, 39 (1), 2011, 50-75.

[7] Goodearl, K. and Warfield, R. Jr., An Introduction to Noncommutative


Noetherian Rings, London Mathematical Society, ST 61, 2004.

[8] Humphreys, J. E., Introduction to Lie Algebras and Representation Theory,


GTM 9, Springer, 1980.

[9] Jategaonkar, A. V., A counter-example in ring theory and homological algebra,


J. Algebra, 12, 1969.

[10] Kaplansky, I., On the dimension of rings and modules, X, Nagoya Math. J,
13, 1958.

[11] Kunz, E., Introduction to Commutative Algebra and Algebraic Geometry,


Birkhäuser, 1991.

[12] Lam, T.Y., Serre’s Problem on Projective Modules , Springer Monographs in


Mathematics, Springer, 2006.

[13] Lang, S., Algebra, Springer, 2004.

146
BIBLIOGRAFÍA 147

[14] Lezama, O. and Villamarı́n, G., Anillos, Módulos y Categorı́as, Facultad


de Ciencias, Universidad Nacional de Colombia, 1994.

[15] Lezama, O., Cuadernos de Álgebra, No. 2: Anillos, SAC2 , Departamen-


to de Matemáticas, Universidad Nacional de Colombia, sede de Bogotá,
sites.google.com/a/unal.edu.co/sac2

[16] Lezama, O., Cuadernos de Álgebra, No. 3: Módulos, SAC2 , Departamen-


to de Matemáticas, Universidad Nacional de Colombia, sede de Bogotá,
sites.google.com/a/unal.edu.co/sac2

[17] Lezama, O., Cuadernos de Álgebra, No. 4: Álgebra lineal, SAC2 , Departa-
mento de Matemáticas, Universidad Nacional de Colombia, sede de Bogotá,
sites.google.com/a/unal.edu.co/sac2.

[18] Lezama, O., Cuadernos de Álgebra, No. 6: Anillos y módulos, SAC2 , Depar-
tamento de Matemáticas, Universidad Nacionalde Colombia, sede de Bogotá,
sites.google.com/a/unal.edu.co/sac2

[19] Lezama, O., Cuadernos de Álgebra, No. 7: Categorı́as, SAC2 , Departa-


mento de Matemáticas, Universidad Nacionalde Colombia, sede de Bogotá,
sites.google.com/a/unal.edu.co/sac2

[20] Lezama, O., Cuadernos de Álgebra, No. 9: Álgebra no conmutativa, SAC2 , De-
partamento de Matemáticas, Universidad Nacionalde Colombia, sede de Bogotá,
sites.google.com/a/unal.edu.co/sac2

[21] Lezama, O. and Reyes, M., Some homological properties of skew P BW


extensions, Communications in Algebra, 42, 2014, 1200-1230.

[22] Li, H., Noncommutative Gröbner Bases and Filtered-Graded Transfer, Lecture
Notes in Mathematics, Vol. 1795, Springer, 2002.

[23] McConnell, J. and Robson, J., Noncommutative Noetherian Rings, Grad-


uate Studies in Mathematics, AMS, 2001.

[24] Quillen, D., Proyective modules over polynomial rings, Invent. Math., 36,
1976, 167-171.

[25] Rocuts, S., Dominios de Prüfer: caracterizaciones, subclases y ejemplos, Tra-


bajo de Grado, Universidad Nacional de Colombia, 1999.

[26] Rotman, J., An Introduction to Homological Algebra, Springer, 2009.

[27] Rotman, J., An Introduction to Homological Algebra, Academic Press, 1979.


148 BIBLIOGRAFÍA

[28] Rowen, L., Graduate Algebra: Noncommutative View, Graduate Studies in


Mathematics, Vol. 91, AMS, 2008.

[29] Serre, J. P., Faisceaux algébriques cohérents, Ann. Math., 61, 1955, 191-278.

[30] Stenström, B., Rings of Quotients: An Introduction to Methods of Ring The-


ory, Springer, 1975.

[31] Suslin, A.A., Proyective modules over polynomial rings are free, Soviet Math.
Dokl., 17, 1976, 1160-1164.

[32] Suslin, A.A., On the structure of the special linear group over polynomial
rings, Math. USSR-Izv. 11, 1977, 221-238.

[33] Vermani, L.R., An Elementary Approach to Homological Algebra, Chapman


& Hall/CRC, Monographs and Surveys in Pure and Applied Mathematics 130,
2003.

[34] Weibel, C., An Introduction to Homological Algebra, Cambridge University


Press, 1997.

Vous aimerez peut-être aussi